Главная Юзердоски Каталог Трекер NSFW Настройки

Космос и астрономия

Ответить в тред Ответить в тред
Check this out!
<<
Назад | Вниз | Каталог | Обновить | Автообновление | 521 124 156
[ТТВ] Тред Тупых Вопросов №178 Аноним # OP 05/05/23 Птн 15:17:34 754527 1
image.png 731Кб, 3800x2800
3800x2800
image.png 1532Кб, 959x1774
959x1774
image.png 257Кб, 600x450
600x450
image.png 512Кб, 800x600
800x600
[ТТВ] Тред Тупых Вопросов №178 - неправильноземельный эдишон

Тред вопросов о жизни, Вселенной и всём таком.

Спрашиваем то, за что в других местах выдают путёвку в биореактор. Здесь анонимные учёные мирового уровня критически рассмотрят любые гениальные идеи и нарисованные в Paint схемы.

Если повстречаете сообщения о "плотноприжатых" фотонах или о "секте" Эйнштейна, то не спешите отвечать на них. Это бредни шизика aka любителя чужих мам, которые были детально разобраны и опровергнуты нейросетью (начиная с >>749760 →): https://arhivach.top/thread/878713/

Прошлый тред >>750203 (OP) уходит за горизонт событий.
Аноним 05/05/23 Птн 15:44:35 754529 2
>>754527 (OP)
Почему ракеты не разбиваются об небесную твердь?
Аноним 05/05/23 Птн 15:58:05 754530 3
Что будет если телепортировать 1см^3 вещества нейтронной звезды на поверхность Земли? Оно взорвется, или останется сжатым?
Аноним 05/05/23 Птн 16:41:16 754533 4
>>754529
С чего ты взял? Они регулярно бьются, нужно не менее девяти ракет чтоб пробить твердь и десятая смогла пролететь.
Аноним 05/05/23 Птн 17:20:57 754534 5
>>754533
А почему ядерными бомбами не пробивают небесный купол?
Аноним 05/05/23 Птн 17:49:22 754536 6
>>754529
Ракеты снабжены двигателями, которые изменяют их траекторию и позволяют им избегать столкновения с небесной твердью. Они могут быть оснащены навигационными системами, которые позволяют им следовать определенному маршруту в пространстве. Кроме того, ракеты имеют механизмы, которые помогают им избегать столкновений и мониторить их путь

>>754530
Я бы предположил, что в этом случае вещество взорвется, поскольку имеет очень большую плотность и при перемещении на небольшое расстояние может достичь предельной точки и взорваться

>>754534
Ядерные бомбы не могут пробить небесный купол, поскольку они просто не достаточно сильны для этого. Для пробивания небесного купола необходимо использовать гораздо более мощные силы
Аноним 05/05/23 Птн 18:46:34 754537 7
Почему в Австралии не ходят верх ногами или под углом?
Аноним 05/05/23 Птн 19:17:27 754538 8
>>754530
Смотря откуда брать кубик.

С поверхности или с внешних слоем там все еще обычное вещество и при снятие давления оно ебанет. В добавок оно ОЧЕНЬ горячее и светит тепловым излучением с пиком аж в гамма области, которое способно наводить радиацию в веществе. Мало будет просто сильного бабаха больше мегатонны, потом будет люто фонить радиацией(правда не долго).

Со средних слоев уже идет нейтронизация вещества - тупо нейтронный газ с примесями тяжелых ядер и иногда мюонов. При снятие давления нейтроны тупо разлетятся, частично рассеиваясь в среде, наводя в ней лютую радиоактивность. Бабах будет, но на порядки слабее, ибо нейтроны электронейтральны и взаимодействуют только с ядрами вещества. Ну еще нейтронное вещество из-аз нейтринного охлаждения холоднее вещества на поверхности.

С недр и ядра не ясно. До сих пор не известно уравнение состояние вещества при таких условиях.
По идеи вещество должно становиться "странным" и становиться метаустойчивым при снятие давления, тогда кубик просто провалится под поверхность и упадет на ядро, ибо оно будет без электронным и электронейтральным, только ядра будут хоть как-то тормозить его.
Ну и другая популярный кандидат это кварковое вещество, находящееся в цветной сверхпроводимости. Тут совсем непонятно, ибо даже на калькуляторах не посчитаешь, а в природе такое не встречается.
По грубым оценкам это вещество не только стабильно при снятие давления, но барионному веществу энергетически выгодно находится в нем. Так при встречи с ядрами, оно будет захватывать их с энерговыделением(преимущественно в виде нейтрино). Но оно опять электронейтрально и удар на ядро, медленно, но верно пожирая его.
Аноним 05/05/23 Птн 19:20:44 754539 9
>>754537
В Австралии ходьба верх ногами или под углом не является популярной, поскольку это не безопасно и нельзя предсказать, как это может привести к непредвиденным последствиям. Люди в Австралии обычно используют наиболее приемлемый и безопасный путь для того, чтобы достичь их целей. В Австралии принято идти по часовой стрелке. Это обусловлено в основном историческими причинами, такими как противоречие на перекрестках, а также одной из причин повышения безопасности движения – это необходимость поворота налево на красный свет
Аноним 05/05/23 Птн 20:32:40 754542 10
>>754538
>снятие давления нейтроны тупо разлетятся
А они не будут удерживаться силами сильного или еще какого-то взаимодействия?
Аноним 05/05/23 Птн 20:44:13 754543 11
>>754529
Потому что небесной тверди не существует.
Аноним 05/05/23 Птн 20:46:02 754544 12
>>754537
А почему ты сам не ходишь на голове?
Аноним 05/05/23 Птн 21:28:25 754546 13
С какой высоты можно ссать вниз, чтобы моча приобрела кинетическую убойную силу взрывного эквивалента?
Или для этого нужна планета с g под сотню, вместо обычных для Земли без малого десяти?
Аноним 05/05/23 Птн 22:03:58 754549 14
>>754546
Согласно физики, для достижения максимального взрывного эквивалента кинетической энергии мочи, ей необходимо находится на высоте около 500 метров. Для Земли это обычно от 67 - 70 км
Аноним 05/05/23 Птн 22:22:15 754551 15
>>754538
>но барионному веществу энергетически выгодно находится в нем. Так при встречи с ядрами, оно будет захватывать их с энерговыделением(преимущественно в виде нейтрино). Но оно опять электронейтрально и удар на ядро, медленно, но верно пожирая его.

Будь так, то все вещество нейтронной звезды превращалось бы в эту странную материю, не?
Аноним 05/05/23 Птн 22:40:47 754552 16
Когда полнолуние закончится?
Аноним 05/05/23 Птн 22:59:26 754553 17
Аноним 06/05/23 Суб 04:44:57 754558 18
скажите пожалуйста, гороскопы и знаки зодиака здесь обсуждаются?
Аноним 06/05/23 Суб 09:15:13 754559 19
>>754551
Таки да, но для этого нужно время. Очень больше время.
Дело в том, что нейтронные звезды релятивистские объекты - гравитационная энергия сопоставима с массой покоя. И эффект отрицательной теплоемкости выражены очень сильно, что подавляет течение многих процессов. В добавок нейтронные звезды ОЧЕНЬ горячие, настолько, что они находятся даже находиться в равновесие с нейтрино. В таких условиях наоборот идет разрушение кварковой материи. Чтоб она начала захватывать, нужно нейтронной звезде остыть. Из-за малой площади поверхности и огромной теплоемкостью нейтронные звезды не могут эффективно охлаждаться. Белые карлики и то быстрее охлаждаются.
Ну еще на нейтронные звезды постоянно что падает с большим энерговыделением, что не способствует падение температуры.
Аноним 06/05/23 Суб 09:51:17 754560 20
>>754558
Здесь. Чего хотел?
Аноним 06/05/23 Суб 10:26:58 754561 21
>>754560
Цереру, Эриду и Седну надо учитывать в нотальной карте?
Аноним 06/05/23 Суб 10:30:30 754562 22
1683358219467.jpeg 381Кб, 790x1000
790x1000
1683358219496.jpg 31Кб, 596x380
596x380
1683358219500.png 641Кб, 1024x1750
1024x1750
1683358219511.png 87Кб, 605x829
605x829
>>754527 (OP)
Если земля круглая и имеет гравитацию то тогда почему на ней нет доминирующий жизни в виде чёрной распластавшейся слизи радиусом в километр и толщиной в 1 миллиметр способную питаться бактериями, солнцем и другой органикой. По сути это была бы самая живучая форма жизни которая была бы сложна в поедании для других животных так она бы еще могла поедать и остальных, просто налипать сверху начиная поедать всё живое.

Как же заебал Абу, посылает мне в мозг 25 кадр через капчу с целью сделать из меня диверсанта.
Аноним 06/05/23 Суб 11:01:17 754563 23
show.png 5Кб, 270x120
270x120
>>754561
Нужно учитывать все, даже все чайники Рассела летающие по своим орбитам, и тогда ты сможешь предсказать судьбу с величайшей точностью. Но можно провести усреднение, но тогда и точность прогнозов упадет до статистической погрешности.
Аноним 06/05/23 Суб 11:03:59 754564 24
>>754562
Если бы земля была абсолютно гладкой, то такая штука возможно бы и существовала.
Аноним 06/05/23 Суб 11:09:21 754565 25
1683360551131.png 38Кб, 556x658
556x658
>>754564
Так если бы земля была гладкая то от такой штуки можно легко избавиться, а когда земля пористая ты хуй как её достанешь, она в каждой дырочке будет сидеть и поедать разрастаясь.
Аноним 06/05/23 Суб 11:24:19 754566 26
>>754565
Держать такое огромное тело все части которого равноценны просто не имеет смысла, оно будет распадаться на отдельные самостоятельные части. Ну а существа поменьше типа слизневиков существуют.
Аноним 06/05/23 Суб 11:32:16 754567 27
1683361925637.mp4 2391Кб, 720x1280, 00:00:15
720x1280
>>754566
А зачем держать его, ну вот немертина распадается а потом снова соединяется, распадаться оно будет но с целью распространения, такому существу было бы достаточно просто оказаться на шерсти любого животного а уже через время оно было съедено и просто получилась бы массивная слизь площадью в километр которая бы просто хаотично лежала и ждала когда в неё наступит другой объект.

Интересно кстате что будет если немертина всосёт обратно не свой желудок а желудок другой немертины, они могут друг с другом желудками обмениваться или нет.
Аноним 06/05/23 Суб 11:40:53 754568 28
image.png 5Кб, 270x120
270x120
Аноним 06/05/23 Суб 11:41:09 754569 29
>>754567
>которая бы просто хаотично лежала и ждала когда в неё наступит другой объект.
Ее бы давно пожрали другие существа - чего это тут биомасса бесхозная валяется.
Аноним 06/05/23 Суб 11:42:01 754570 30
>>754568
Тоже об этом подумал.
Аноним 06/05/23 Суб 11:52:53 754571 31
1683363162212.gif 1917Кб, 480x270
480x270
1683363162291.png 908Кб, 1391x686
1391x686
>>754569
> Ее бы давно пожрали другие существа
Ну а кто муравьи, они бы просто застряли в ней. Какой нибудь лесной лось, ну часть слизал бы но очень сильно заебался так как площадь большая, ему еще и на шерсть возле ебла эта хрень налипла и в последствии сожрала ему ебало и стала только больше.
Аноним 06/05/23 Суб 11:55:11 754572 32
>>754571
Между муравьем и лосем есть еще куча всяких типоразмеров.
Аноним 06/05/23 Суб 12:02:38 754573 33
1683363747947.jpg 144Кб, 736x981
736x981
1683363747961.jpg 311Кб, 865x1080
865x1080
>>754572
Я назвал самых прожорливых животных.

И кто сможет её съесть, ну типа даже у муравьеда не получится такое сожрать. Любое соприкосновение с ней приведет тебя к тому что она на тебе налипнет а после размножится на тебе же и сожрет. Ну максимум ей может противостоять новый эволюционный вид который как раз был бы полностью гладким и на котором та не смогла бы закрепиться. Но такой вид был бы не конкурентен среди других животных так как приспособлен для поедания лишь одного животного "черная сильзь".

Черная слизь всё равно бы доминировала.
Аноним 06/05/23 Суб 12:07:48 754574 34
>>754573
> она на тебе налипнет
Довольно урчишь и слизываешь с себя вкусняшку, лапкой чистишь мордочку, ушки и усики, и тоже слизываешь.
Аноним 06/05/23 Суб 12:13:44 754575 35
1683364413334.png 989Кб, 1015x567
1015x567
>>754574
> Довольно урчишь и слизываешь с себя вкусняшку, лапкой чистишь мордочку, ушки и усики, и тоже слизываешь.
Типа как на картинке?

Загривок сама себе не почистит, и под шерстью тоже, в волосах застрянет.
Аноним 06/05/23 Суб 12:16:13 754576 36
>>754575
Всегда можно попросить чтоб тебя облизал кто-то еще.
что-то у меня фантазии разыгрались...
Аноним 06/05/23 Суб 12:22:23 754577 37
> Всегда можно попросить чтоб тебя облизал кто-то еще.
Ну если только так.
Аноним 06/05/23 Суб 13:59:56 754581 38
>>754562
>виде чёрной распластавшейся слизи
Есть, бактериальные маты, появились миллиарды лет назад
>>754562
>способную питаться бактериями, солнцем и другой органикой
Такого не бывает, либо в примитивном виде ты фотосинтетик автотрофный, либо ты гетеротроф и жрешь других. Если ты растительноядный тебе надо жрать овердохуя и двигаться в поисках еды, если ты хищник, то тем более, и еды у тебя меньше и тебя самого меньшинство в биосфере так как располагаешься на самом верху пищевой цепи.
>>754562
>была бы самая живучая форма жизни
Нету никаких самых живучих, больше всего на земле бактерий и вирусов, каждый день половина популяции всех бактерий выпиливается вирусами бактериофагами. А вирусы это так - полужизнь, неполноценная жизнь. Самые живучие как раз высшие животные - рыбы, птицы, пресмыкающиеся, млекопитающие. Именно такие в процессе эволюции получили способность жить долго.
Аноним 06/05/23 Суб 17:55:15 754592 39
1683384903627.mp4 8669Кб, 640x360, 00:02:55
640x360
1683384903629.mp4 806Кб, 240x240, 00:00:38
240x240
>>754581
> бактериальные маты, появились миллиарды лет назад
От них легко избавиться, а от живой слизи сложнее.
> Если ты растительноядный тебе надо жрать овердохуя и двигаться в поисках еды
Ты всеядный в прямом смысле слова, ты черный как нигер для того чтобы впитывать свет и получать тепло, ты покрыт слизью чтоб не пересохнуть на солонце и ты липки, сюбой кто тебя коснётся просто заберёт частичку тебя. Тебе даже двигаться не обязательно, животное просто само наступит в тебя и ты начнёшь его медленно переваривать.
> А вирусы это так - полужизнь, неполноценная жизнь.
Хуйня полная вирусы это тоже жизнь.
Аноним 06/05/23 Суб 19:06:05 754603 40
>>754592
Ну вот твои грибы и медузы - это максимум. Никакой суперслизи тут нет и в помине.
Аноним 06/05/23 Суб 19:21:05 754604 41
1683390060747.jpg 156Кб, 1023x1280
1023x1280
1683390060753.mp4 4487Кб, 540x960, 00:00:16
540x960
Эволюционологи тут? Как вы объясните пикрилы? Ведь нельзя просто взять и слепить части тела, как пластилин блять. Оно вообще не должно работать, так как для появления сиамских близнецов или пятой ноги у кота должны произойти миллионы мутаций, это почти невозможно. Любая из миллиона неправильная - и плод нежизнеспособен.
Аноним 06/05/23 Суб 19:48:53 754609 42
>>754604
Ответ прост: на пикрилах инопланетяне.

Мимо эволюционолог из спейсача
Аноним 06/05/23 Суб 20:18:11 754612 43
>>754604
>Как вы объясните пикрилы?
Мутации.
Лунные вопросы Аноним 06/05/23 Суб 23:00:22 754620 44
Creepy-tan.png 73Кб, 810x975
810x975
Вопрос 1: На какой высоте Луна перестанет быть приливнозахваченной, если мы будем увеличивать/уменьшать высоту её орбиты?

Вопрос 2: Как бы повлияла на жизнь на Земле и на саму Луну появление на орбите Земли второй Луны, находящейся в резонансе с Луной-1, но при этом вращающейся на полярной орбите? А если то же самое, но на полярной орбите, одновременно перпендикулярной и орбите Луны-1, и орбите Луны-2?

Вопрос 3: Если мы за Луной-1 поставим Луну-2, которые теперь вращаются в одной плоскости, но не в резонансе, то, если в отличие от почти круглой орбиты Луны-1, сделаем орбиту Луны-2, при каких параметрах орбиты Луны-2 (а Луна-1 по всем параметрам как Луна на Земле) при взаимовлиянии с Луной-2 приведут к орбитальной рокировке Луны-2 на Луну-1? А при каких Луна-1 от сближения Луны-2 будет просто слегка прокручиваться? Будет ли прогрев Луны-1 приближением Луны-2 настолько существенным, что сможет держать околоземные температуры на Луне-1?
Аноним 06/05/23 Суб 23:50:11 754624 45
Положим тебе за щеку что на расстояние 100 световых лет внезапно обнаружился аналог Земли, но на ней живут динозавры и теоретически могут жить люди, но людей там нет - НАШИ ДЕЙСТВИЯ?
Аноним 07/05/23 Вск 00:51:39 754627 46
>>754604
>>754612
Нет там никаких мутаций. Это просто два эмбриона-близнеца на ранней стадии развития слиплись. Человеческий организм очень универсален -- можно отрезать ухо и пришить его на ногу, и оно будет там жить и потреблять кровь (так делают при операциях). Тут тот же принцип.
Аноним 07/05/23 Вск 05:24:29 754633 47
show.png 6Кб, 270x120
270x120
>>754624
Изучаем ее дистанционно, допиливаем пузырь альбукерке.
Аноним 07/05/23 Вск 08:22:42 754638 48
>>754603
> Никакой суперслизи тут нет и в помине.
А когда появится нас всех ждёт пиздец. Слизь она идеальна, может просто распластавшись лежать и впитывать солнце а когда в неё кто-то наступит то тутже заберёт её клетки на себя. Условный лось наступил, к шерсти прилипла, сначала съест шерсть а потом в течени 2-3 месяцев и всего лося.

Ну типа максимум к чему она была бы не приспособлена это к перепадам минусовых температур, но это ни на что не повлияет, у нас 70-80% видов к этим температурам не устойчивы.
Аноним 07/05/23 Вск 09:36:16 754641 49
>>754624
>НАШИ ДЕЙСТВИЯ
Далеко, только зоонаблюдать. Если бы на Альфецентавре то можно было бы начинать строить взрыволет.
Аноним 07/05/23 Вск 09:52:43 754642 50
scale120085.jpg 277Кб, 833x1200
833x1200
>>754527 (OP)
>неправильноземельный эдишон
Где полая Земля?
Аноним 07/05/23 Вск 10:02:42 754643 51
vom-wachsenden-[...].jpg 27Кб, 511x700
511x700
Без имени.jpg 7Кб, 259x194
259x194
Почему Земля растёт?
Аноним 07/05/23 Вск 10:16:20 754644 52
Земля растет.mp4 12893Кб, 640x480, 00:02:26
640x480
>>754643
В том смысле - почему она постоянно разбухает?
Аноним 07/05/23 Вск 10:53:53 754646 53
>>754643
>>754644
Она не растёт и не разбухает. Иначе, если бы она таки росла, откуда на ней взялся океан, которого не было? За счёт каких процессов рост?
Теория растущей Земли зиждится на теории того, что раньше континенты были одним континентом, что хотя и главенствующая сейчас теория, но тоже теория, причём не неоспоримая.
Аноним 07/05/23 Вск 11:24:25 754647 54
>>754646
>если бы она таки росла, откуда на ней взялся океан, которого не было? За счёт каких процессов рост?
Может быть в центре Земли немного не то, что подразумевает официально признанная гипотеза?
Вдруг в центре Земли зарождается материя за счёт захвата материальной среды на полюсах?
Маленькое Солнце там - >>754642
Аноним 07/05/23 Вск 11:36:55 754648 55
1683448615038.jpeg 175Кб, 864x1002
864x1002
>>754646
> Иначе, если бы она таки росла
Так она действительно растёт, получает энергию от солнца, за счёт этого растут растения, потом умирают, потом их съедают бактерии и цикл повторяется вновь.
> откуда на ней взялся океан
Чат бот говорит что в начале своего существование земля была просто раскалённой лавой, и что она постепенно остывала выбрасывая пары газа и таким образом формировалась атмосфера, а сама вода появилась за счёт вулканической деятельности, комет и астероидов.
>>754647
Не в центре земли у нас находится тот элемент что самый тяжелый, ну а выброс магмы может происходить из-за того что сверху на это все давит гигантская толща поверхности но из более легких материалов. А так просто классика, в низу самое тяжелое, в верху самое легкое, какой нибудь газ формирующий атмосферу и прочее.
Аноним 07/05/23 Вск 11:44:49 754649 56
>>754648
>Так она действительно растёт, получает энергию от солнца, за счёт этого растут растения, потом умирают, потом их съедают бактерии и цикл повторяется вновь.

Это неоспоримый факт, но анон под "ростом" имел в виду шизогипотезу о том, что радиус Земли увеличивается в размерах.
Аноним 07/05/23 Вск 11:57:47 754650 57
>>754649
>радиус Земли увеличивается в размерах.
Почему нет?
Вся сложность гипотезы в том - откуда берется масса, но это легко объяснимо в рамках гипотез о мироустройстве с физической материальной средой вместо вакуума. В том же фильме на ТВ Кульура (>>754644 ) этого объяснения избегали и объясняли рост массы захватом нейтрино - пусть даже так. Какая разница?
Зато Динозавров объяснить гораздо проще и куда они исчезли - возросла гравитация.
Аноним 07/05/23 Вск 12:00:51 754652 58
>>754648
>Не в центре земли у нас находится тот элемент что самый тяжелый
>А так просто классика, в низу самое тяжелое, в верху самое легкое
Ну хз - что там у тебя за классика)
Возьми воздушны шарик, кинь туда подшипник, залей водой и раскрути. Где оказался подшипник?
Аноним 07/05/23 Вск 12:03:27 754653 59
1683450206896.png 95Кб, 1200x845
1200x845
1683450206904.png 17Кб, 662x189
662x189
>>754646
> раньше континенты были одним континентом
Ну раньше не земле не было воды следовательно и вся земля была одним большим континентом.
>>754649
> Это неоспоримый факт, но анон под "ростом" имел в виду шизогипотезу о том, что радиус Земли увеличивается в размерах.
Так радиус земли и увеличивается, на поверхности есть растения питающиеся солнцем, а в воде есть бактерии и водоросли, когда их цикл оканчивается они умирают и формируют дно из своих останков и цикл повторяется вновь, земля растёт. Там один меловой слой увеличил радиус океанического дна на 1 км, а сколько у нас всего разных слоев, штук 10 и каждый из них прибавляет к радиусу на 1-2 км.
Аноним 07/05/23 Вск 12:03:36 754654 60
>>754650
Потому что ни расширения Земли, ни наличие манясреды не подтверждается экспериментально.
Аноним 07/05/23 Вск 12:06:29 754655 61
>>754652
> Где оказался подшипник?
В том месте на которое будет воздействовать центробежная сила.
> Возьми воздушны шарик, кинь туда подшипник, залей водой и раскрути.
Твой эксперемент хуйня, по твоей логике получается что мы должны каждую секунду смываться океаническими волнами высотою в 200 метров как в интерстелларе?
Аноним 07/05/23 Вск 12:07:45 754656 62
>>754652
> Возьми воздушны шарик, кинь туда подшипник, залей водой и раскрути. Где оказался подшипник?
Можешь еще взять ведро с водой и начать его раскручивать, и где окажется вода?

тот же самый эксперимент.
Аноним 07/05/23 Вск 12:19:03 754657 63
>>754653
>Так радиус земли и увеличивается, на поверхности есть растения питающиеся солнцем, а в воде есть бактерии и водоросли, когда их цикл оканчивается они умирают и формируют дно из своих останков и цикл повторяется вновь, земля растёт. Там один меловой слой увеличил радиус океанического дна на 1 км, а сколько у нас всего разных слоев, штук 10 и каждый из них прибавляет к радиусу на 1-2 км.

Размеры планеты определяются её массой. А при круговороте жизни масса планеты не изменяется, так как все останки организмов используются другими организмами в качестве источника питания и энергии.
Аноним 07/05/23 Вск 12:21:43 754658 64
>>754657
> А при круговороте жизни масса планеты не изменяется
Земля энергию от солнца получает, солнцем питаются многие бактерии и растения, эта энергия берётся из вне а не только формируется внутри планеты.
Аноним 07/05/23 Вск 12:28:33 754659 65
>>754658
Я не спорю, что Земля получает значительную часть энергии от Солнца. Я тебе про массу говорил.
Аноним 07/05/23 Вск 12:32:43 754661 66
>>754659
> Я не спорю, что Земля получает значительную часть энергии от Солнца. Я тебе про массу говорил.
Да, вот бактерии и растения едят солнце, после умирают и формируют земную кору и массу земли.
Аноним 07/05/23 Вск 12:33:30 754662 67
>>754658
Так масса откуда берется? На нас не падает каждый день по миллиарды тонн вещества? Более того Земля теряет массу из диссипации атмосферы.
Аноним 07/05/23 Вск 12:38:27 754663 68
>>754662
А она берется? Не считая небольшого количества того что сыпется из космоса.
Аноним 07/05/23 Вск 12:41:25 754664 69
>>754663
Она изначально была на Земле с момента формирования.
Всякие геологические слои это просто глобальная перетасовка вещества.
Аноним 07/05/23 Вск 13:02:32 754666 70
1683453752038.webp 544Кб, 1600x1200
1600x1200
1683453752042.jpg 236Кб, 900x691
900x691
>>754662
> Так масса откуда берется?
Пиздец ну и вопросы у тебя, что-то уровня откуда берётся пространство. Он не берётся, оно есть изначально.
> На нас не падает каждый день по миллиарды тонн вещества?
А нам и не нужно чтобы каждый день падало по миллиардам тон вещества. Достаточно просто по сотне тонн в год. Просто возьми сто тон умнож на условную цифру в 250 миллионов лет, ровно столько сколько длился триасовый период. Просто 100 тонн в год.
Аноним 07/05/23 Вск 13:04:15 754667 71
>>754661
Растениям и бактериям помимо солнечной энергии необходимы органические вещества, минералы из почвы и другие питательные элементы. Но даже если организм питается только за счёт фотосинтеза, для зарождения такого организма необходимы химические элементы, которые невозможно получить из солнечного излучения. А после смерти останки организма будут преобразованы в питательные вещества для других организмов и в элементы для образования новой жизни, так что новая масса никак не образуется.
Аноним 07/05/23 Вск 13:10:49 754668 72
>>754666
> Просто 100 тонн в год.
100 000 кг / 365 = 273,97 кг в день.
>>754667
Ну видишь солнечная энергия есть и растения её потребляют следовательно и есть масса.

И вон я уже под рассчитал, наша земля в треассовый период в среднем росла на 273 кг в день просто за счёт солнечной энергии, но это условно. Я условно выбрал массу но это не та охуевшая масса которую выбрал ты про сотни миллардов тонн в день.
Аноним 07/05/23 Вск 13:16:47 754670 73
>>754666
При условии, что ежегодно на Землю падает 100 тонн всякой космической пыли, за 250 миллионов лет Земля получила 2.5х1013 кг вещества. А масса Земли приблизительно 6х1024 кг. То есть за весь триасовый период Земля получила 4,2х10-10 процента своей массы - это меньше чем капля в море.
Аноним 07/05/23 Вск 13:21:37 754671 74
>>754668
Земля не получает новую массу за счёт солнечной энергии. Это уже существующие на Земле химические элементы собираются в живые организмы, подпитываясь энергией. А когда эти организмы умирают, их вещество снова используется для образования жизни или для питания других организмов. Поэтому это и называется КРУГОВОРОТОМ жизни
Аноним 07/05/23 Вск 13:22:58 754673 75
>>754668
>273,97 кг в день

Это ничтожно мало по сравнению с массой планеты >>754670
Аноним 07/05/23 Вск 13:31:23 754674 76
>>754670
Масса земли примерно равна 6-ти триллионам тонн.
Лет ей 4.5 миллиардов.

100 тонн одних метеоров и астероидов даже без учёта поедания бактериями и растениями солнца.

4,5 миллиарда x 100 тонн = 450 миллиардов тонн соотношение 1 к 13 и это только от метеоритов астероидов и прочего мусора.

>>754673
> При условии, что ежегодно на Землю падает 100 тонн всякой космической пыли
> Это ничтожно мало по сравнению с массой планеты
Это одна тринадцатая от целой массы планеты.
Аноним 07/05/23 Вск 13:52:59 754676 77
Screenshot20230[...].png 48Кб, 1080x2340
1080x2340
Screenshot20230[...].png 51Кб, 1080x2340
1080x2340
Screenshot20230[...].png 67Кб, 1080x2340
1080x2340
image.png 55Кб, 494x573
494x573
>>754674
>Масса земли примерно равна 6-ти триллионам тонн.
1 триллион это 1012. А масса Земли примерно равна 6х1021 тонн или 6х1024 кг - твоя ошибка номер 1

>Это одна тринадцатая от целой массы планеты.
Ты изначально ошибся с массой Земли, поэтому у тебя получилось такое большое соотношение. - твоя ошибка номер 2

Смотри пикрилы:
1 пик - масса упавшего вещества в кг за 4.5 миллиарда лет
2 пик - масса этого же вещества в кг, умноженная на 13 - это гораздо меньше массы Земли (6х1024 кг)
3 пик - процентное соотношение массы упавшего вещества к массе Земли.

За всё время существования Земли на неё упало 7,5х10-9 процента её массы.
Аноним 07/05/23 Вск 13:55:16 754677 78
>>754674
>поедания бактериями и растениями солнца
Чего?
Аноним 07/05/23 Вск 13:57:40 754678 79
>>754677
Анон считает, что растения и бактерии получают энергию от солнца и преобразуют её в новую массу
Аноним 07/05/23 Вск 14:09:14 754680 80
>>754678
Helicobacter Einstaini - преобразует энергию в массу.
Аноним 07/05/23 Вск 14:14:33 754681 81
1683458073208.png 4Кб, 423x42
423x42
>>754676
> 1 триллион это 1012. А масса Земли примерно равна 6х1021 тонн или 6х1024 кг - твоя ошибка номер 1
Преобразовывай значения в тонны, ты полюбому сам гдето проебался, мне похуй на степени потомучто в этих степенях можно проебаться.
> Смотри пикрилы:
И что мне твои пикрилы?

Вот смотри мой пикрил 5,9736⋅1024 кг в кг, в килограммах понимаешь да, а мне в тоннах нужно...
Аноним 07/05/23 Вск 14:21:06 754682 82
>>754681
Подели на тысячу - будут тебе тонны
Аноним 07/05/23 Вск 14:21:27 754683 83
1683458487573.jpg 219Кб, 1200x839
1200x839
>>754676
6 триллионов тонн = 5,9736⋅1024 кг
Я все верно вывел, это ты проебался и начал мне в килограммах массу нести.

и 1/13 это только то что мы подсчитываем, ну типа как ты сказал что это масса космической пыли, тоесть это даже не масса метеоритов а лишь космическая пыль.
Аноним 07/05/23 Вск 14:23:01 754684 84
>>754680
чатгпт не знает такую:
>Я не могу найти подтвержденных научных данных о существовании Helicobacter Einstaini. Возможно, вы имели в виду Helicobacter pylori - бактерию, которая может вызывать язвенную болезнь желудка и 12-перстной кишки.
Аноним 07/05/23 Вск 14:24:37 754685 85
>>754682
> Подели на тысячу - будут тебе тонны

6 триллионов тонн = 6⋅1024 кг.

Масса земли примерно равна 6-ти триллионам тонн.
Лет ей 4.5 миллиардов.

100 тонн одних метеоров и астероидов даже без учёта поедания бактериями и растениями солнца.

4,5 миллиарда x 100 тонн = 450 миллиардов тонн соотношение 1 к 13 и это только от метеоритов астероидов и прочего мусора.

= 1/13
Аноним 07/05/23 Вск 14:27:17 754686 86
>>754683
тонна = 103 кг
триллион = 1012
6х103х1012 = 6х1015
это ты проебался или покажи как ты считал
Аноним 07/05/23 Вск 14:27:55 754687 87
1683458875397.png 22Кб, 690x168
690x168
Есть бактерии еще которые хранят в себе заряд, типа встроенная батарейка с энергией, им когда надо они освобождают эту батарейку и там два химических вещества начинают выдавать энергию этой бактерии.
Аноним 07/05/23 Вск 14:28:28 754688 88
>>754685
Где ты нашёл, что масса Земли 6 триллионов тонн?
Аноним 07/05/23 Вск 14:32:58 754690 89
>>754684
Это редкие бактерии, изначально они жили в жерлах вулканов, однако позже их стали находить на ядерных полигонах и внутри ядерных реакторов. Именно их бесконтрольный рост привел к авариям на Маяке, Чернобыле и Тримайл-айленде.
Аноним 07/05/23 Вск 14:34:44 754691 90
>>754690
> Это редкие бактерии
Очень, даже у нас в желудке нашли.
> изначально они жили в жерлах вулканов
Изначально всё живое в жерле вулканов жило...
Аноним 07/05/23 Вск 14:34:53 754692 91
>>754690
давай ссылку на статью
Аноним 07/05/23 Вск 14:41:21 754693 92
>>754692
Ссылку точную не дам но статья за авторством Тазиева, Зельдовича и Даррелла с названием что-то "О энергопоглощающих бактериях в свете теории относительности"
Аноним 07/05/23 Вск 14:46:32 754694 93
>>754693
пусто
>К сожалению, мне не удалось найти статью с таким названием и авторами, возможно, она была выдумана для задания. Если у вас есть более точная информация, я могу попробовать помочь с поиском.
Аноним 07/05/23 Вск 14:49:01 754695 94
1683460141649.png 16Кб, 745x367
745x367
>>754688
Слово из спам листа.
Аноним 07/05/23 Вск 15:09:22 754697 95
Screenshot20230[...].png 65Кб, 1080x2340
1080x2340
Screenshot20230[...].png 63Кб, 1080x2340
1080x2340
>>754695
Всё равно при пересчёте в тоннах получается малая доля процента
А MЗемли / mупавшего вещества примерно равно 133333333333, то есть масса упавшего вещества составляет 1/133333333333 часть от массы планеты

Помимо этого у тебя есть ещё одна ошибка:
ты говоришь, что MЗемли / mупавшего вещества = 13 333,333 и что это соотношение 1/13
Вот только это соотношение 1/13 333,333
Аноним 07/05/23 Вск 15:16:52 754698 96
>>754697
> Вот только это соотношение 1/13 333,333
Ну да это мой проёб в подсчётах, только ты не учел одно но, мы изначально то подсчитывали массу какой то космо пыли и не учитывали массу других космических объектов.

Но еще стоит упомянуть что солнце постоянно теряет массу, от того что она теряет протоны электроны и тепло. Но это всё получает и кое кто другой, например те кто питается протонами и те кто хладнокровные и получают энергию в виде тепла от солнышка, а получаем мы её все. Хз но наверное и электроны тоже кому-то на нашей планете тоже нужны.
Аноним 07/05/23 Вск 15:20:33 754699 97
1683462032710.png 7Кб, 652x95
652x95
1683462032723.png 11Кб, 676x151
676x151
>>754698
Вот еще про космо парашу, хз сколько еще от астероидов и комет мы получили массы, я инфы не нашёл.
Аноним 07/05/23 Вск 15:30:41 754700 98
>>754627
>Это просто два эмбриона-близнеца на ранней стадии развития слиплись
А с хуя ли они слиплись? Господь так повелел?
Там же на уровне генов, очевидно, происходит какой-то сбой и в результате получается такая хуйня. По-научному это и зовется мутацией.
>>754643
Ниггерский хуй в пизде у твоей мамаши растет, а Земля имеет постоянный объем.
Аноним 07/05/23 Вск 15:35:14 754702 99
Чем занимались динозавры на Земле?
Аноним 07/05/23 Вск 15:37:52 754703 100
>>754700
>происходит какой-то сбой
А с хуя ли он происходит? Господь так повелел?
Иногда бывает что один близнец полностью поглощает другого, и у него внутри могут быть некоторые части своего брата/сестры.

>>754702
В основном пьянством, развратом и наркотиками, посему и вымерли.
Аноним 07/05/23 Вск 15:45:41 754705 101
>>754702
Думаю если растение и увеличивали свою массу и массу планеты.
Аноним 07/05/23 Вск 16:01:20 754710 102
1683464480350.webp 226Кб, 2000x1333
2000x1333
>>754527 (OP)
А как вы относитесь к моей теории постоянного усиления гравитации больших космических объектов следовательно и всей массы планеты.

Условно где самая сильная гравитация на земле? Всё верно, в её центре. Что делает гравитация с веществом? Притягивает его к себе. А что делает притянувшиеся вещество? Сдавливает другое вещество ещё сильнее. А что случается с планетой когда внутри неё начинается сильное сдавливание? Всё правильно! Термоядерные ядерные реакции которые увеличивают массу и гравитацию этой же самой планеты.
Аноним 07/05/23 Вск 16:04:40 754712 103
Аноним 07/05/23 Вск 16:06:19 754713 104
>>754655
>Твой эксперемент хуйня, по твоей логике получается что мы должны каждую секунду смываться океаническими волнами высотою в 200 метров как в интерстелларе?

Где ты в воздушном шарике, полностью заполненным водой видел волны?
Наоборот это лучшая симуляция Земли - ибо натяжение шарика имитирует земную гравитацию.

>В том месте на которое будет воздействовать центробежная сила.
Ну то есть в лучшем случае подшипник прижмется к стенке - в худшем он пробьет стенку и лава вода вытечет наружу.
Но ни в одном варианте подшипник не останется в центре вращающегося с водой шарика.
Аноним 07/05/23 Вск 16:07:14 754714 105
>>754656
>Можешь еще взять ведро с водой и начать его раскручивать, и где окажется вода?
>тот же самый эксперимент.

нет не тот же самый - натяжение шарика имитирует гравитацию
Аноним 07/05/23 Вск 16:09:03 754715 106
>>754666
>Просто 100 тонн в год.
Откуда?
Раз где-то прибыло, значит где-то убыло. Где убыло?
Аноним 07/05/23 Вск 16:18:40 754716 107
>>754713
> Где ты в воздушном шарике, полностью заполненным водой видел волны?
Ну да, воздушных потоков не существует, водных потоков тоже. В полностью заполненном шарике будут водные потоки, водный поток это тоже что-то типа такой волны понимаешь.
> лучшая симуляция Земли - ибо натяжение шарика имитирует земную гравитацию.
Ты же вкурсе что земная гравитация она притягивает к себе и лучшей её имитацией является магнит.
> Ну то есть в лучшем случае подшипник прижмется к стенке - в худшем он пробьет стенку и лава вода вытечет наружу.
В твоём ебанном эксперементе я хз что должно произойти но по сути на подшипник начнёт влиять центробежная сила и тот начнёт бегать по стенкам шарика в зависимости от оси центробежности.
> залей водой и раскрути
Вот возьми ведро, залей его водой на половину и раскрути, можешь крутить как угодно, хоть вдоль хоть поперёк, можешь еще веревку к ручке привязать и крутить, и о чудо, вода просто прибьётся к днищу ведра вне зависимости в каком положении будет ведро, хоть вверх тармашками хоть в лево и в право.
Аноним 07/05/23 Вск 16:22:15 754718 108
1683465735133.png 180Кб, 768x768
768x768
>>754715
> Откуда?
> Раз где-то прибыло, значит где-то убыло. Где убыло?
Наверное из пояса астероидов.
Аноним 07/05/23 Вск 16:54:49 754729 109
>>754710
>Условно где самая сильная гравитация на земле? Всё верно, в её центре
В центре Земли гравитация самая маленькая. Буде она идеальным шаром равномерной плотности, там была бы невесомость.
Аноним 07/05/23 Вск 17:05:11 754731 110
1683468310875.jpg 263Кб, 2560x1436
2560x1436
1683468310900.jpeg 122Кб, 1200x1015
1200x1015
>>754729
> В центре Земли гравитация самая маленькая.
> невесомость.
Там невесомость в следствии того что это центр но на деле там не невесомость, с твоей же логикой можно сказать что и в центре черной дыры тоже невесомость, но по факту ты просто притянулся к центру где ты "якобы в невесомости" то как ты себя формально будешь ощущать, а на деле ты от туда даже выбраться не сможешь. И это ровно то что делает гравитация, центр земли притягивает в себя материю а та материя что оказалась в центре находится в неком коллапсирующим состоянии производящем термоядерные реакции на которые со всех сторон давит материя желающая притянуться к центру. Отсутствие гравитации в центре это всего лишь иллюзия.
Аноним 07/05/23 Вск 17:10:44 754733 111
>>754703
>А с хуя ли он происходит?
Мутации случайны по определению.
>>754710
>А как вы относитесь к моей теории постоянного усиления гравитации
Как к бреду сумасшедшего. Теорией такую хуйню назвать нельзя.
Аноним 07/05/23 Вск 17:15:19 754734 112
1683468919518.jpg 560Кб, 1440x863
1440x863
>>754733
> Как к бреду сумасшедшего. Теорией такую хуйню назвать нельзя.
Черные дыры так и появляются, начинают сжиматься под собственной гравитацией.

Но с чего все взяли что так может только черная дыра? Конкретно все крупные объекты в нашей вселенной на это способны, только черной дырой они в такой ситуации превратятся оооооооочень не скоро.
Аноним 07/05/23 Вск 17:16:56 754735 113
>>754734
только в черные дыры они в такой ситуации превратятся оооооооочень не скоро. фикс.
Аноним 07/05/23 Вск 17:18:55 754736 114
>>754731
>центр земли притягивает в себя материю
Центр Земли не может к себе ничего притягивать, это геометрическая точка. Масса притягивается не к центру, а к другой массе.
Аноним 07/05/23 Вск 17:26:07 754737 115
1683469567211.jpg 55Кб, 1000x1000
1000x1000
>>754736
> Центр Земли не может к себе ничего притягивать, это геометрическая точка. Масса притягивается не к центру, а к другой массе
> которая находится в центре земле
фикс

Я тебе уже писал что отсутствие гравитации в центре это иллюзия, если бы твоё сознание могло бы там выжить то ты ощущал себя как в невесомости.

А еще ты бы ощущал себя в невесомости если бы находился на очень глубокой глубине под водой и на тебя бы давили сотни метров воды, ты бы просто умер под водой находясь в одной точке, все потому что давление воды не дало тебе всплыть.
Аноним 07/05/23 Вск 18:45:37 754742 116
>>754710
>Термоядерные ядерные реакции которые увеличивают массу и гравитацию этой же самой планеты
Только термоядерные и ядерные реакции не увеличивают массу, это противоречит законам сохранения массы и энергии.
Аноним 07/05/23 Вск 18:51:58 754744 117
Screenshot20230[...].png 69Кб, 1080x2340
1080x2340
>>754699
Вот только чтобы масса Земли увеличилась хотя бы на 1% за 4,5 миллиарда лет, на неё должно падать 13333333333 тонн вещества в год.
Аноним 07/05/23 Вск 19:06:44 754745 118
1683475603836.jpg 295Кб, 1280x868
1280x868
>>754742
> Только термоядерные и ядерные реакции не увеличивают массу, это противоречит законам сохранения массы и энергии.
А что если масса это пространство и время, ну типа гравитация становится настолько сильной что начинает притягивать пространственное полотно внутрь себя. Теория относительности объясняет искривление пространства и времени под действием гравитации. Масса и энергия создают кривизну в пространстве-времени, а гравитация является следствием этой кривизны. Моя теория заключается в том, что под действием гравитации пространство и время могут искажаться настолько сильно, что создаются новые области с высокой плотностью энергии которые могут восприниматься как новые массы. Это может объяснять некоторые неясные явления в космосе например, почему некоторые галактики содержат так много темной материи. Возможно, эта темная материя формируется в результате искажения пространства-времени вблизи массивных объектов, таких как черные дыры или галактики.
Аноним 07/05/23 Вск 19:19:47 754746 119
1683476387110.jpg 170Кб, 1280x905
1280x905
>>754744
Ну а протоны и тепло от солнца электроны тоже хз нужны ли для жизни электроны которые выбрасывает солнце или они только полярное сияние создают, но тепло это энергия это то что помогает нам сохранять массу, ну а протоны это протоны, их жрут бактерии и растения в следствии чего их жрем мы а потом какаем и формируем массу всего.

Ну типа сам суди, что дает нам тепло от солнца, позволяет меньше тратить массы, следовательно больше накапливать, тепло это то что приходит к нам из вне.
Аноним 07/05/23 Вск 19:40:37 754747 120
>>754746
>тепло это энергия это то что помогает нам сохранять массу
Типа мы дров меньше сжигаем? Но тогда новая масса все равно не образуется... Или как в твоём понимании "тепло помогает сохранять массу"?

>ну а протоны это протоны, их жрут бактерии и растения
Более 90% заряженных частиц из солнечного ветра отклоняются от Земли магнитным полем. А в фотосинтезе протоны не участвуют, растения и бактерии используют именно энергию солнечного света чтобы расщеплять воду на кислород и водород, а затем из водорода и СО2 синтезировать глюкозу. Новая масса при этом не образуется.

>тепло от солнца, позволяет меньше тратить массы, следовательно больше накапливать
Но эта масса уже изначально была на Земле, она не появилась из вне как в случае космической пылью и метеоритами.
Аноним 07/05/23 Вск 20:00:25 754748 121
>>754747
> Типа мы дров меньше сжигаем?
И это тоже, но помимо этого мы и кушаем меньше следовательно больше и остается, есть же всякие еще и хладнокровные животные и также мы тратим часть энергии на поддержания температуры тела, а тепло от солнца помогает нам это энергию не тратить а запасать. Тепло это энергия из вне на которую мы не расходуем ресурсов а только получаем её.
> Новая масса при этом не образуется.
А какже накопления органических веществ, органики больше становится.
> расщеплять воду на кислород и водород
Ну по такой логике у нас должны были уже моря и океаны испариться.
> Но эта масса уже изначально была на Земле, она не появилась из вне как в случае космической пылью и метеоритами
Всмысле изначально, убери солнце и этой массы не будет. И эта масса и энергия как раз таки появилась из вне, её солнце сюда на нас посылает.
Аноним 07/05/23 Вск 20:01:16 754749 122
>>754734
>начинают сжиматься под собственной гравитацией
Не черные дыры, а ядра массивных звезд. А коллапс ядра начинается из-за остановки термоядерных реакций, которые поддерживали давление внутри звезды и противостояли гравитации. Чтобы Земля схлопнулась в черную дыру сама собой, нужно, чтобы ее масса составляла 20 масс Солнца. Только это будет уже не планета, а звезда - голубой гигант. Поэтому не надо тут нести хуйню.
Аноним 07/05/23 Вск 20:05:42 754750 123
>>754749
> Чтобы Земля схлопнулась в черную дыру сама собой, нужно, чтобы ее масса составляла 20 масс Солнца. Только это будет уже не планета, а звезда - голубой гигант. Поэтому не надо тут нести хуйню.
Так я и пишу что это произойдет очень не скоро и не факт что это будет наша земля, скорее всего нас поглотит солнце.
> А коллапс ядра начинается из-за остановки термоядерных реакций,
А может термоядерные реакции останавливаются из-за очень сильного давления.
Аноним 07/05/23 Вск 20:50:42 754752 124
>>754748
>А какже накопления органических веществ, органики больше становится.
Но массы больше не становиться. Вещество, из которого образуются новые организмы, заменяется веществом из уже умерших организмов и так по кругу.

>Ну по такой логике у нас должны были уже моря и океаны испариться.
Так они именно что испаряются, а потом вода снова конденсируется и наполняет моря. И тоже самое с водой внутри живых организмов: вода+СО2 превращаются в глюкозу, потом растение умирает и его поедают бактерии, выделяя водород, углекислый газ, метан и тд, потом эти вещества проходят ещё кучу этапов и вновь собираются в воду, но при этом общая масса всего вещества на планете не изменяется.

>Всмысле изначально, убери солнце и этой массы не будет
Убрав Солнце, убрав только излучение Солнца, но сохранив его гравитационное поле, чтобы солнечная система не рассыпалась мы получаем необитаемую планету с той массой вещества, которую оно собрало из протопланетного диска, и эта масса практически такой же как и у Земли сейчас.
"практически" потому что тепло от Солнца влияет на улетучивание атмосферы с планеты и на кинетическую энергию молекул вещества, которое необходимо захватить гравитации зарождающейся планеты, в протопланетном диске.
Аноним 07/05/23 Вск 21:22:40 754753 125
>>754752
> Вещество, из которого образуются новые организмы, заменяется веществом из уже умерших организмов и так по кругу.
Ну так организмы еще и тепло кушают, твой организм он вынужден поддерживать определённую температуру тела а солнце в этом ему помогает, то-есть халявная энергия, без этого тепла ты был бы вынужден больше хавать с целью получения тепла. Тепло от солнца это ресурс извне.
> Так они именно что испаряются, а потом вода снова конденсируется
> вода+СО2 превращаются в глюкозу, потом растение умирает и его поедают бактерии,
По такой логике жизнь вобще должна вымереть ведь по сути все полезные вещества со временем вымываются в глубь поверхности земли где ни растение ни бактерии их достать не смогут, а в нашем же случаее жизнь процветает, это значит что мы берем ресурсы из вне.
> потому что тепло от Солнца влияет на улетучивание атмосферы
Если убрать излучение Солнца, то это приведет к значительному изменению условий на планете, так как источник тепла и света исчезнет. Тепло это калории, тепло это энергия.
Аноним 07/05/23 Вск 21:31:38 754754 126
>>754753
При питании и пищеварении ни один миллиграмм съеденной пищи не пропадает. Вся пища либо выходит из организма в форме отходов и перерабатывается бактериями, либо преобразуется в строительный материал для роста и уже после смерти используется другими организмами. Так что даже если бы Земля была вечной антарктидой и организмы потребляли десятки кг пищи в день, общая масса всего вещества на планете была бы неизменной. Просто было бы меньше животных, чтобы им хватало еды.
Аноним 07/05/23 Вск 21:53:07 754756 127
>>754754
Ладно ты победил.
Аноним 07/05/23 Вск 22:05:41 754760 128
Раз вы закончили массо-спор, прошу ответить на луно-вопросы >>754620
Аноним 07/05/23 Вск 22:41:08 754762 129
>>754750
>Так я и пишу что это произойдет
С хуя ли это произойдет? Ты говна въебал, что ли?
>А может термоядерные реакции останавливаются из-за очень сильного давления
Из-за давления ниггерского хуя в пизде твоей мамаши у нее заканчивается сквирт, а термоядерные реакции в ядре массивных звезд прекращаются из-за исчерпания там запасов водорода.
Аноним 07/05/23 Вск 23:23:13 754766 130
>>754760
>Раз вы закончили массо-спор, прошу ответить на луно-вопросы
Будет исполнено...

>>754620
1. Расстояние приливного захвата рассчитывается по формуле Роша:
d = R ( 2M / m ) / 3
где:
d - расстояние от центра гравитирующего тела до спутника
R - радиус планеты
M - масса планеты
m - масса спутника
Для системы Земля-Луна d ≈ 384400 - это среднее расстояние от Земли до Луны. Вообще радиус Лунной орбиты варьируется от 363 104 до 405 696 км, вследствие чего наблюдаются Лунные либрации: https://ru.wikipedia.org/wiki/%D0%9B%D0%B8%D0%B1%D1%80%D0%B0%D1%86%D0%B8%D1%8F
Так образом, если средний радиус Лунной орбиты будет равен 390~400 тыс. км и более, Луна уже не будет приливно захвачена. И чем больше будет радиус её орбиты, тем сильнее будут отличаться её собственный и орбитальный периоды вращения.

2. >Как бы повлияла на жизнь на Земле и на саму Луну появление на орбите Земли второй Луны, находящейся в резонансе с Луной-1, но при этом вращающейся на полярной орбите?
В предыдущих ТТВ уже был подобный вопрос. Расчёт орбитального движения для трёх тел сравнимой массы - так ещё и с перпендикулярными плоскостями орбит - это крайне сложная штука и называется она "задача трёх тел".
Если в общих чертах, такая система будет очень-очень нестабильной, и с большой вероятностью всё закончиться столкновением космических тел. Но пока это не произошло, Луна-2 будет создавать на Земле дополнительные приливы, которые ощутимо повлияют на климат.

>А если то же самое, но на полярной орбите, одновременно перпендикулярной и орбите Луны-1, и орбите Луны-2?
Вот тут не совсем понял. Как орбита Луны-2 может быть перпендикулярна своей собственной орбите?

3. Орбитальная рокировка происходит когда период обращения одного спутника равен удвоенному периоду второго. Период обращения рассчитывается по формуле:
T = 2 π √( a³ / GM )
где:
T - сидерический период обращения спутника
a - радиус его орбиты
G - гравитационная постоянная
М - масса гравитирующего тела
Сидерический период Луны-1 ≈ 27,3 дня, значит для Луны-2 сидерический период должен быть равен 54,6 дней. (так же период Луны-2 может быть и в 2 раза меньше периода Луны-1, но тогда, Луна-2 точно упадёт на Землю)
Подставляем циферки в формулу и получаем, что радиус орбиты Луны-2 ≈ 607870 км

>А при каких Луна-1 от сближения Луны-2 будет просто слегка прокручиваться?
Вот это уже очень сложно рассчитать. Радиусы орбит Луны-1 и Луны-2 должны не сильно отличаться, но при этом их орбиты не должны находиться слишком близко друг к другу, потому что тогда луны просто столкнуться.

>Будет ли прогрев Луны-1 приближением Луны-2 настолько существенным, что сможет держать околоземные температуры на Луне-1?
Точно нет. Для этого у Луны-1 должно быть горячее ядро, в реальности же у Луны оно практически остыло. Так же поверхность Луны очень плохо сохраняет температуры из-за отсутствия атмосферы. + Луна-2 будет лишь периодически взаимодействовать с Луной-1, столь малого времени не достаточно для прогрева Луны.
Аноним 07/05/23 Вск 23:28:49 754767 131
>>754766
>>754620
>А при каких Луна-1 от сближения Луны-2 будет просто слегка прокручиваться?
Кстати Луна и так слегка прокручивается, причем по двум осям, это и есть лунные либрации, и второй спутник для этого не нужен
Аноним 08/05/23 Пнд 05:57:03 754781 132
>>754762
> С хуя ли это произойдет? Ты говна въебал, что ли?
Это происходит с любым крупным объектом, каждому суждено стать чёрной дырой но это займёт очень много времени. Просто у массивных звёзд это быстрее всего получается, ну а пронаблюдать такое тупо не возможно, мы не можем через телескоп увидеть как планета становится массивной звёздой а она в свою очередь чёрной дырой.
> Из-за давления ниггерского хуя в пизде твоей мамаши у нее заканчивается сквирт
Хуя се куколд с фантазиями, мысли только о чёрных хуях.
> а термоядерные реакции в ядре массивных звезд прекращаются из-за исчерпания там запасов водорода.
Тогда почему термоятерные реакции не прекращаются а начинаются другие с другими более тяжелыми элементами?
И почему так получилось что у чёрных дыр сильная гравитация? Хотя как мы знаем гравитация берётся из за массы планеты, а чёрная дыра не только она одна так может создаёт массу из ниоткуда, просто гравитацией прижимает очень сильно этим самым и останавливает термоядерные процессы.
Аноним 08/05/23 Пнд 09:06:44 754783 133
>>754766
>Вот тут не совсем понял. Как орбита Луны-2 может быть перпендикулярна своей собственной орбите?
Луна-1, стандартная, врщается по экватору, Луна-2 вращается через нулевый меридиан, то есть её плоскость орбиты проходит через Мелкобританию и Камчатку, ну а Луна-3, перпендикулярная плоскостям орбит Луны-1 и Луны-2 тоже полярная, но проходит плоскостью своей через Сибирь и Канаду по меридианам E90 и W90
Аноним 08/05/23 Пнд 09:16:22 754784 134
>>754767
>Кстати Луна и так слегка прокручивается, причем по двум осям, это и есть лунные либрации, и второй спутник для этого не нужен
Не. Прокручивалась она если бы при каждом новом цикле приближения и отдаления Луны, она хотя бы на 1 градус проворачивалась в одну и ту же сторону каждый лунный месяц, таким образом мы бы уж давно видели с Земли всю поверхность Луны как на ближней, так и на дальней для нас стороне, потому что она - дальняя сторона - медленно, периодически и постоянно становилась бы ближней стороной, чтобы много позже снова стать дальней.
Вот это и есть проворачивание в контексте вопроса.
Либрация же не приводит к такому эффекту, а значит это не прокручивание. Тут, скорее, более уместным будет слово дрожь, или колебания, которое подразумевает некое визуальное сподвижение, но не подразумевает кардинальных, даже малых изменений
Аноним 08/05/23 Пнд 09:46:07 754785 135
>>754716
>В полностью заполненном шарике будут водные потоки, водный поток это тоже что-то типа такой волны понимаешь.
Откуда?
Если воздушный шарик с водой будет вращаться с постоянной угловой скоростью, то никаких потоков не будет.

>Ты же вкурсе что земная гравитация она притягивает к себе и лучшей её имитацией является магнит.
Для симуляции Земли не нужен магнит - нужен воздушный шарик, вода и подшипник.
Подшипник - это ядро Земли, ты его никогда не удержишь в центре при вращении шарика с водой.
Вода- это жидкая магма, в которой плавает тяжелое ядро.
А шарик - земная кора, который своим натяжением ещё и земную гравитацию симулирует, сжимая воду/магму до окружности.
Нахуй тебе в этой симуляции магнит блядь?

>В твоём ебанном эксперементе я хз что должно произойти
Подшипник как аналог тяжелого металлического ядра Земли должен удерживаться в центре - согласно ебанутым оф гипотезам.
Конечно такого не будет - потому что только ебанутые слоупоки могут верить, что тяжелое ядро Земли в жидкой магме сможет удерживаться в центре при вращении Земли. Про это и разговор бля.

>Вот возьми ведро, залей его водой на половину и раскрути, можешь крутить как угодно, хоть вдоль хоть поперёк, можешь еще веревку к ручке привязать и крутить, и о чудо, вода просто прибьётся к днищу ведра вне зависимости в каком положении будет ведро, хоть вверх тармашками хоть в лево и в право.
Ну да - тяжелое металлическое ядро Земли в океане жидкой магмы уже давно бы прижало к земной коре и возможно даже ядро бы пробило земную кору.
Но олигофрены этого не понимают - и даже заявляют на оф уровне, что тяжелое мет ядро находится в центре Земли - придурки, сэр!
Аноним 08/05/23 Пнд 10:27:51 754787 136
>>754785
Во-первых твой эксперимент попросту некорректный, потому что на модель Земли в виде шарика с водой и подшипника будет действовать сила тяжести от настоящей Земли, поэтому даже в состоянии покоя подшипник будет прижиматься ко дну шарика. Чтобы этот опыт стал корректным, масса всей модели должна быть настолько большой, чтобы модель обладала своим собственным гравитационным полем, то есть быть как минимум размером со средней астероид.
Во-вторых, отношения плотности и массы ядра к плотности, массе и вязкости мантии отличается от отношения плотности и массы подшипника к плотности, массе и вязкости воды. Но это мелкая проблема меркнет на фоне первого и третьего пунктов.
А в-третьих, ты подразумеваешь, что шарик должен рассекречивать с бешеной угловой скоростью, чтобы центробежная сила прижимала подшипник к стенке шарика. Однако угловая скорость реальной Земли всего 1 оборот в сутки или 15° в час. На такой скорости центростремительное ускорения, действующие на ядро, ничтожно мало по сравнению с силой тяжести, поэтому вещество планеты распределяется по плотности, и в центре оказывается самое тяжёлое ядро. Если вращать шар с угловой скоростью 1 оборот в сутки, то подшипник будет просто лежать на его дне, это, при учёте первого пункта несовершенства твоей модели, будет означать, что воображаемое ядро-подшипник находится в центре воображаемой земли-шарика.
Аноним 08/05/23 Пнд 11:10:46 754789 137
>>754781
>Это происходит с любым крупным объектом, каждому суждено стать чёрной дырой но это займёт очень много времени. Просто у массивных звёзд это быстрее всего получается, ну а пронаблюдать такое тупо не возможно, мы не можем через телескоп увидеть как планета становится массивной звёздой а она в свою очередь чёрной дырой.
1. Землеподобная планета не может стать звездой, так как ей недостаточно массы для старта термоядерных реакций и такая планета состоит в основном из оксидов тяжелых элементов, а для формирования звезды нужно огромное количество водорода, который при безумных температура и давлении будет превращаться в гелий, а гелий уже в более тяжелые элементы.
Ледяным гигантам так же недостаточно массы.
В звезду может превратиться только газовый гигант вроде Юпитера, так как он как раз состоит из водорода и гелия. Вот только для этого масса планеты должна увеличиться в сотню раз, но такое невозможно.
2. Чёрными дырами становятся только останки массивных звезд, масса которых после взрыва сверхновых превышает 3.2 солнечных масс.

> Из-за давления ниггерского хуя в пизде твоей мамаши у нее заканчивается сквирт
>Хуя се куколд с фантазиями, мысли только о чёрных хуях.
Анон ответил по фактам, смирись с суровой реальностью.

>Тогда почему термоятерные реакции не прекращаются а начинаются другие с другими более тяжелыми элементами?
И почему так получилось что у чёрных дыр сильная гравитация? Хотя как мы знаем гравитация берётся из за массы планеты, а чёрная дыра не только она одна так может создаёт массу из ниоткуда, просто гравитацией прижимает очень сильно этим самым и останавливает термоядерные процессы.
Тут согласен, анон немного ошибся. Термоядерные реакции не прекращаются после исчерпания водорода, в этом случае заканчивается только жизнь звезды на главной последовательности и звезда становиться красным гигантом. Как это происходит?
По мере синтеза водорода в гелий, весь гелий опускается в недра звезды, так как он тяжелее, и накапливается в ядре. Гелиевое ядро сгущается и становиться всё плотнее и горячее, пока в нём не запуститься термоядерный синтез гелия в ядра углерода, азота и кислорода. То есть давление внутри ядра как раз таки способствует термоядерному синтезу более тяжёлых элементов.
А теперь про гравитацию черных дыр:
Чёрные дыры обладают точно такой же гравитацией как и звёзды, из которых они образовались, так как имеют такую же массу. А вот первая космическая скорость для чёрных дыр гораздо больше. Она рассчитывается по формуле:
V1 = √( GM / R )
где:
G - гравитационная постоянная
М - масса гравитирующего тела
R - радиус тела
Вещество чёрной дыры сосредоточенно в гораздо меньшем объёме, чем у звезды. Поэтому значение R в данной формуле настолько маленькое, что V1 становиться больше скорости света.

Мимо другой анон
Аноним 08/05/23 Пнд 11:17:26 754790 138
>>754787
>шарик должен рассекречивать с бешеной угловой скоростью
должен раскручиваться*
быстрофикс Т9
Аноним 08/05/23 Пнд 11:27:22 754792 139
>>754787
>Во-первых твой эксперимент попросту некорректный, потому что на модель Земли в виде шарика с водой и подшипника будет действовать сила тяжести от настоящей Земли
А у тебя вообще воображения нет, да?

>поэтому даже в состоянии покоя подшипник будет прижиматься ко дну шарика.
В свободном падении не будет)
Прыгаем, твою мать, из самолета и раскручиваем сучка шарик с водой, в который запихнули подшипник.

>Чтобы этот опыт стал корректным, масса всей модели должна быть настолько большой,
Вот ты тупой, мляяяяяяяяя)))

>Во-вторых, отношения плотности и массы ядра к плотности, массе и вязкости мантии отличается от отношения плотности и массы подшипника к плотности, массе и вязкости воды.
Сучка, ты мысленные эксперименты ставить не умеешь?
В голове блядь смоделировать слабо?
Ебануться
Крахмал добавь мля)))

>А в-третьих, ты подразумеваешь, что шарик должен рассекречивать с бешеной угловой скоростью, чтобы центробежная сила прижимала подшипник к стенке шарика.
Любая - сколь угодно малая угловая скорость прижмет подшипник к стенке, ебануться ты тупой)
Аноним 08/05/23 Пнд 11:29:02 754793 140
>>754783
Три Луны тем более столкнуться и все вместе упадут на Землю.
Аноним 08/05/23 Пнд 11:31:13 754794 141
>>754789
>Термоядерные реакции не прекращаются после исчерпания водорода, в этом случае заканчивается только жизнь звезды на главной последовательности и звезда становиться красным гигантом. Как это происходит?
Никак и никогда
Водород синтезируется на поверхности звезд - поэтому он никогда не заканчивается.
Вся гипотеза эволюции звезд высосана из хуя
Аноним 08/05/23 Пнд 11:39:56 754796 142
>>754792
1. Зачем делать мысленный эксперимент с кривой и несовершенной моделью Земли в виде шарика с водой, если вместо этого можно провести мысленный эксперимент с самой Землёй?
А если проводить такой эксперимент в реальности, то нужно учитывать все несовершенства модели.
2. В невесомости эксперимент будет также некорректным, потому что в невесомости вещество не распределяется по плотности на отдельные слои, например как масло и вода, в невесомости вещества с разной плотностью просто не смешиваются друг с другом. Для корректного эксперимента, как я уже сказал, нужно чтобы модель обладала собственной силой тяжести.

>Любая - сколь угодно малая угловая скорость прижмет подшипник к стенке, ебануться ты тупой)
Нет, если центробежная сила пренебрежимо мала по сравнению с силой тяжести как у реальной Земли. Тебя разве не учили в шестом классе складывать два вектора? Или ты ещё в младшей школе учишься?
Аноним 08/05/23 Пнд 11:42:55 754797 143
>>754794
На поверхности звёзд температура и давление недостаточны для синтеза водорода, тем более у красных гигантов. Но если бы на поверхности водород синтезировался в гелий, он бы тем более закончился.
Аноним 08/05/23 Пнд 13:16:51 754801 144
Что находиться за первыми галактиками в глазах телескопа?
Аноним 08/05/23 Пнд 13:57:12 754804 145
>>754792
> твою мать
> Вот ты тупой, мляяяяяяяяя)))
> Сучка, ты
> Ебануться
> Крахмал добавь мля)))
> ебануться ты тупой)
Вот он уровень критики.
Аноним 08/05/23 Пнд 15:00:34 754805 146
>>754781
>Это происходит с любым крупным объектом, каждому суждено стать чёрной дырой
Это ты из жопы того самого ниггера высосал, который ебет твою мамашу-шлюху?
С чего ты, блядь, дегенерат, это решил? Так, примерно почувствовал?
>Хуя се куколд с фантазиями
И какие у тебя фантазии? О черных дырах?
>Тогда почему термоятерные реакции не прекращаются а начинаются другие
Так они и заканчиваются, доходя до углерода - запасы водорода и гелия иссякли, ресурса для продолжения термоядерных реакций нет.
>И почему так получилось что у чёрных дыр сильная гравитация?
Потому что это компактный объект с гигантской массой. Вся масса ядра коллапсирует в точку. Получающийся в итоге объект обладает чудовищной гравитацией - он плотнее даже нейтронных звезд.
>создаёт массу из ниоткуда
Дегенерат ебаный, ЧД ничего из ниоткуда не создает, никакую массу. ЧД - это и есть масса, коллапсировавшая в точку. Масса берется из вещества ядра звезды, которое претерпело коллапс в черную дыру.
>этим самым и останавливает термоядерные процессы
Долбоеб, термоядерные реакции останавливаются в звезде до коллапса в ЧД. Сначала происходит их остановка, а уж потом - коллапс в черную дыру.
Пиздец ты альтернативно одаренный даун. Ору с таких. Придумал себе свою "физику" и неиронично пытается эту хуету кому-то втирать.
>>754801
Первые звезды.
Аноним 08/05/23 Пнд 15:02:25 754806 147
>>754804
Тут, не только в ТТВ спейсача, а вообще в тематике, завёлся как минимум 1 пидорас, который специально отвечает не так, напрмер отвечая по форме, но игнорируя суть; придирается к словам или тупо отвечает как политик - вроде ответил, но нихуюшечки не сказал; или просто отвчеает прямо неправильно или троллит тупостью, или такими вот "быдло"-ответами.

Он ж со своим откровением развлечений в тематике даже мелькал на скриншотах - мол, у него развлечение в тематиках, и особенно сытно кормится в рисовачах. Кстати, своим постом он - я более чем уверен - хотя бы парочку рандомов подсадил на такое развлечие, разрушающее тематику.
Аноним 08/05/23 Пнд 15:03:55 754808 148
>>754793
>Три Луны тем более столкнуться и все вместе упадут на Землю.
Ну да, нельзя же взять такое положение орбит, что бы луны не пересекались, только столкновение в лоб в лоб, иначе никак
Аноним 08/05/23 Пнд 15:44:44 754810 149
>>754805
>Первые звезды.
пруфы уже есть?
Аноним 08/05/23 Пнд 15:58:20 754812 150
>>754810
С такого расстояния их все равно не разглядеть.
Аноним 08/05/23 Пнд 17:16:44 754813 151
>>754796
>Зачем делать мысленный эксперимент с кривой и несовершенной моделью Земли в виде шарика с водой, если вместо этого можно провести мысленный эксперимент с самой Землёй?
Потому что в центре Земли нет тяжелого мет ядра - ты совсем долбоеб пидрила?
Этот эксперимент нужен чтобы доказать несостоятельность высосанных из хуя оф гипотез.

>В невесомости эксперимент будет также некорректным, потому что в невесомости вещество не распределяется по плотности на отдельные слои, например как масло и вода
Да какие плотности мудак ты конченный, здесь важен тяжелый предмет вв вращающемся океане жидкости - который мать твою от малейшего толчка улетит к стенке.

>Нет, если центробежная сила пренебрежимо мала по сравнению с силой тяжести как у реальной Земли.
В сраку твою мать силу тяжести, какая тебе нах сила тяжести в центре Земли пидрила.
Аноним 08/05/23 Пнд 18:10:34 754825 152
Можно, но это очень сложно.
Даже для предсказания поведения системы из трёх тел сравнимой массы, вращающихся в одной плоскости, нет общего уравнения. А для системы четырёх тел с взаимно перпендикулярными орбитами - тем более. Поэтому с большой уверенностью можно сказать, что такая система будет крайне нестабильной.
https://ru.wikipedia.org/wiki/%D0%97%D0%B0%D0%B4%D0%B0%D1%87%D0%B0_%D1%82%D1%80%D1%91%D1%85_%D1%82%D0%B5%D0%BB
Аноним 08/05/23 Пнд 19:14:36 754826 153
1683562473907.jpg 128Кб, 813x610
813x610
1683562473930.jpeg 176Кб, 900x506
900x506
>>754813
> Потому что в центре Земли нет тяжелого мет ядра
В центре земли есть более тяжёлые элементы.
И земля находится в без гравитационном пространстве поэтому тяжелые элементы в центре земли никуда не будут падать, они будут находится просто в одной точке.
> Да какие плотности мудак ты конченный, здесь важен тяжелый предмет вв вращающемся океане жидкости - который мать твою от малейшего толчка улетит к стенке.
Ну возьми ведро и заполни его водой на половину и начни его вращать и вода тоже от малейшего толчка улетит к стенке, это называется центробежной силой.
> от малейшего толчка улетит к стенке.
Ты же понимаешь что в космосе нет низа и верха, ядро не может никуда отлететь, ядро не будет лежать в самом низу земли, ядро будет находится в центре земли.

>>754796
Кароче я понял что это шиз имеет в виду, в его представлении вот солнце оно кароче крутит землю, придерживает его своей гравитацие и таким образом по его мнению из земли должно вылететь ядро, якобы гравитация от солнца должна как то усиленно работать именно по ядру земли, он не понимает что гравитация она равномерна для всех объектов. Он наверное еще очень сильно удивится если ему сказать что наковальня и перышко в безвоздушном пространстве упадут на землю с одинаковой скоростью.
Аноним 08/05/23 Пнд 20:05:26 754829 154
Ligera-desviaci[...].jpg 109Кб, 1280x737
1280x737
>>754826
>Ну возьми ведро и заполни его водой на половину и начни его вращать и вода тоже от малейшего толчка улетит к стенке, это называется центробежной силой.
Поэтому мы и заливаем воду в воздушный шарик, который своим натяжением будет имитировать гравитацию - то есть создавать давление на воду, направленное к центру воздушного шара.

>В центре земли есть более тяжёлые элементы.
Ты проверял?

>тяжелые элементы в центре земли никуда не будут падать, они будут находится просто в одной точке.
С какого хера? В тупой гипотезе мет ядро плавает в раскрученной жидкости - это ядро должно прижать к стенке центробежной силой.

>Ты же понимаешь что в космосе нет низа и верха, ядро не может никуда отлететь, ядро не будет лежать в самом низу земли, ядро будет находится в центре земли.
Вращение сучка, вращение

>гравитация она равномерна для всех объектов.
Вот именно - в любом вихре всегда в центре пустота! Нигде в Мире нет примеров, кога в центре вращающегося вихря находилось бы тяжелое ядро
Аноним 08/05/23 Пнд 20:45:23 754832 155
1683567921268.jpg 111Кб, 1240x858
1240x858
1683567921286.png 26Кб, 1204x167
1204x167
>>754829
> воздушный шарик, который своим натяжением будет имитировать гравитацию
Нет не будет так как подшипник в статичном положении находится в самом низу а не в его центре.
> то есть создавать давление на воду,
А ты не думал что в твоём эксперименте шарик рвётся не из за подшипника а из за воды?
> Ты проверял?
Да, наливаешь в стакан подсолнечное масло и воду, смотришь.
> В тупой гипотезе мет ядро плавает в раскрученной жидкости - это ядро должно прижать к стенке центробежной силой.
В твоем тупом эксперименте всё так.
А на деле земля равномерно вращается вокруг солнца ведь гравитация она равномерная для всех. Перышко и наковальня в космосе тоже будут крутиться вокруг солнца и будут это делать с одинаковой скоростью, перышко не полетит впереди наковальни.
> Вращение сучка, вращение
Ну вот смотри, твоя шлюха мать крутиться на шесте на земле со скоростью 200 оборотов в секунду, от такой центрифужной силы у твоей матери из жопы вывалятся кишки и говно.
А вот теперь твоя шлюха мать крутится на шесте в космосе или на МКС со скоростью 200 оборотов в секунду и из её жопной дырки не вылетают кишки с говном.
Аноним 08/05/23 Пнд 20:53:20 754833 156
1683568398140.jpg 83Кб, 700x788
700x788
>>754832
> А вот теперь твоя шлюха мать крутится на шесте в космосе
А не сори, кишки всёже у твоей матери вылетят.

По другом:

Смотри, теперь твою мать на хуе вертит негрила со скоростью 200 оборотов в секунду и это все происходит в безвоздушном пространстве.
Аноним 08/05/23 Пнд 20:54:08 754834 157
Земля не растет, прост гравитация немношк меняется. Куб воды во времена динозавров почти нихуя не весил, а сейчас весит тонну. Не?

мимо)
Аноним 08/05/23 Пнд 20:55:57 754835 158
Например из-за центробежных сил. Скорость вращения Земли замедляется и центробежные силы меньше противодействуют гравитации. Не?

мимо)
Аноним 08/05/23 Пнд 20:56:23 754836 159
>>754832
>Ну вот смотри, твоя шлюха мать крутиться на шесте на земле со скоростью 200 оборотов в секунду, от такой центрифужной силы у твоей матери из жопы вывалятся кишки и говно.
А вот теперь твоя шлюха мать крутится на шесте в космосе или на МКС со скоростью 200 оборотов в секунду и из её жопной дырки не вылетают кишки с говном.
Не, в обоих случаях из неё все вылетит. А вот если мать будет настолько тяжёлой, что её сила тяжести будет в сотни раз больше центробежной силы, как в случае с Землёй, то говно и кишки будут оставаться четко в центре тела.

Мимо другой анон
Аноним 08/05/23 Пнд 21:03:58 754837 160
>>754834
>>754835
Нет. Чтобы центробежные силы значительно противостояли гравитации, Земля должна вертеться как минимум 1 оборот в минуту (это примерно, потому что считать лень), но при такой угловой скорости планета не смогла бы удержать атмосферу и ещё сама бы сплющилась в эллипс.
Аноним 08/05/23 Пнд 21:04:23 754838 161
1683569061062.jpg 84Кб, 1280x720
1280x720
>>754834
> Куб воды во времена динозавров почти нихуя не весил
А там это связано с окислением какого то элемента в воздухе или типа того, но там вес воды не сильно падал. Ну кароче вода формально меньше весила так как и кислородная среда была другая, но это не сильно на вес воды влияло, но это не точно.

>>754836
А что думаешь про другой эксперимент с нигерским хуем? >>754833
По сути кишки должны будут вывалить изо рта так как в заднице будет хуй, хотя с другой стороны вращение же происходит поперёк хуя, следовательно его мать просто разорвёт на части, кишки с говном просто пробьют стенки живота и просто разлетятся.
Аноним 08/05/23 Пнд 21:08:36 754839 162
изображение.png 249Кб, 427x320
427x320
>>754838
Думаю, в таком эксперименте центробежные силы сплющат мать у полюсов до состояния пикрила
Аноним 08/05/23 Пнд 21:22:40 754840 163
>>754839
> до состояния пикрила
О ну практически идеальный шар.
И ВСЁ-ТАКИ ЗДЕСЬ НУЖНЫ НИКИ КАК В po Аноним 08/05/23 Пнд 21:28:49 754842 164
Чтобы скрывать всякую хуйню.
Аноним 08/05/23 Пнд 21:34:25 754844 165
Аноним 08/05/23 Пнд 21:34:55 754845 166
>>754837
>Нет. Чтобы центробежные силы значительно противостояли гравитации, Земля должна вертеться как минимум 1 оборот в минуту (это примерно, потому что считать лень)
Нет. Если Земля будет в два раза быстрее вращаться, то и всё на Земле будет в два раза меньше весить. А если в три раза быстрее - тогда в три раза меньше. Не?
Аноним 08/05/23 Пнд 21:38:50 754847 167
>>754845
Нет. К тому же количество потерянного веса из-за центробежной силы зависит ещё и от широты местоположения: чем ближе к полюсам, тем слабее эффект потери веса.
Аноним 08/05/23 Пнд 21:41:32 754848 168
1683571290222.jpg 192Кб, 1200x900
1200x900
>>754845
> Если Земля будет вращаться
Нет, гравитация она постоянна и зависит от массы планеты. Если земля совсем перестанет вращаться то и гравитация останется той же.
> А если в три раза быстрее - тогда в три раза меньше. Не?
В твоём вымышленном эксперименте будет вращаться не в три раза а на 33% быстрее, просто 100 делишь на то количество во сколько раз оно будет быстрее крутится, но гравитация слабее или сильнее не станет от этого.
Аноним 08/05/23 Пнд 22:54:05 754849 169
Ну бля( Ну значит сами атомы медленнее вращаются и от этого тяжелее. Не?
Аноним 08/05/23 Пнд 22:59:41 754850 170
>>754849
Атомы не вращаются. И с чего ты решил что на Земле сила тяжести меняется?
Аноним 08/05/23 Пнд 23:35:01 754851 171
Bergmannsrule,p[...].png 756Кб, 730x619
730x619
>>754850
>Атомы не вращаются.
Электроны вокруг ядер. Ну и у некоторых частиц ещё и спин имеется, который вроде как ось вращения. Не?
>И с чего ты решил что на Земле сила тяжести меняется?
Томущо раньше на плонете жили большие динозавры, а теперь живут маленькие пчелики. Значит раньше плонета быстрее вращалась. Не?
Аноним 08/05/23 Пнд 23:52:10 754853 172
Почему не делают несколько копий космических аппаратов? Например, вместо одного марсохода послать штуки 3, а лучше 5. Все равно 80% денег уходит на r&d. Сейчас вот у джуса антенна сломалась. Такой хуйни не произошло бы, если бы послали несколько одинаковых аппаратов.
Аноним 09/05/23 Втр 00:05:42 754854 173
Когда нам пиздец?

Луна не светит, это ее освещает ... ЧТО?
Аноним 09/05/23 Втр 00:12:00 754855 174
>>754853
Что за Джус? Какой-то аналог Витьки?
Аноним 09/05/23 Втр 03:21:23 754859 175
>>754853
> вместо одного марсохода послать штуки 3
Опортьюнити и Спирит, Куриосити и Персивиранс. Они парные.
Аноним 09/05/23 Втр 05:37:40 754860 176
>>754854
Солнце. А в фазах половинки Луны и меньше её темный кусок диска освещает тусклый солнечный свет, отраженный от Земли - "пепельный свет".
Аноним 09/05/23 Втр 11:40:48 754867 177
Screenshot20230[...].png 88Кб, 1080x2340
1080x2340
Screenshot20230[...].png 87Кб, 1080x2340
1080x2340
Screenshot20230[...].png 86Кб, 1080x2340
1080x2340
Screenshot20230[...].png 88Кб, 1080x2340
1080x2340
>>754851
>Электроны вокруг ядер. Ну и у некоторых частиц ещё и спин имеется, который вроде как ось вращения. Не?
Спешу тебя разочаровать, это вращение вообще никак не влияет на силу тяжести.

>Томущо раньше на плонете жили большие динозавры, а теперь живут маленькие пчелики. Значит раньше плонета быстрее вращалась. Не?
Вот формула зависимости веса тела с массой M на экваторе от центробежной силы:
m = M - 4π2MR/gT2
где:
m - итоговый вес тела в кг
M - масса тела в кг
R - радиус Земли в метрах
g - ускорение свободного падения на Земле
Т - время одного полного оборота Земли в секундах
Я провёл расчёты именно для экватора, потому что там центробежная сила больше всего влияет на вес.
На пикрилах подсчитан итоговый видеть вес для динозавра массой 80 тонн:
1. угловая скорость Земли 1 оборот за 24 часа - вес ≈ 79,725 тонн
2. угловая скорость Земли 1 оборот за 12 часов - вес ≈ 78,9 тонн
3. угловая скорость Земли 1 оборот за 8 часов - вес ≈ 77,527 тонн
4. угловая скорость Земли 1 оборот за 6 часов - вес ≈ 75,603 тонн
Таким образом, даже если бы Земля вращалась в 4 раза быстрее, вес динозавра уменьшился бы всего на 5,5%. Это как если бы ты вместо 80 кг весил 75 - сомневаюсь, что такая разница в весе значительно влияла на твою жизнь.
Аноним 09/05/23 Втр 13:25:34 754870 178
Screenshot20230[...].png 187Кб, 1080x2220
1080x2220
>>754867
угловая скорость Земли 1 оборот за 1.5 часа - вес ≈ 9660 тонн. Оправдывайся.
Аноним 09/05/23 Втр 13:26:32 754871 179
>>754867
>это вращение вообще никак не влияет на силу тяжести.
Чому? Может влияет. Ты проверял?
Аноним 09/05/23 Втр 14:11:01 754874 180
Screenshot20230[...].png 89Кб, 1080x2340
1080x2340
>>754870
Ну ты бы ещё 1,4 часа подставил, чтобы наверняка.
Аноним 09/05/23 Втр 14:39:44 754876 181
16568463277360.png 403Кб, 512x361
512x361
>>754874
Ну, то есть, теоретически вращение Земли может норм на гравитацию влиять, не? Скажем, Земля только сформировалась и вращается как бешенная. На ней живут крупные пчелики, которые не лезут на полюса - не томущо там холодно, а томущо они там весят в 10 раз больше, чем на экваторах. А потом внезапно прилетает булыжник, появляется кратер Чикшулуб, а кусочек Земли улетает на орбиту в качестве Луны. От столкновения Земля замедляет вращение в 16 раз. Крупные пчелики начинают дохнуть, томущо теперь весят как весили бы на полюсах. Маленькие пчелики с полюсов расселяются по всей Земле - их больше не едят крупные пчелики с экваторов. Не?
Аноним 09/05/23 Втр 14:45:16 754877 182
>>754876
При столкновении при котором от Земли отделилась Луна изменение силы тяжести от скорости вращения - это найменьшая проблема которую бы испытывали крупные (и мелкие) челики где бы они не жили.
Аноним 09/05/23 Втр 15:03:47 754879 183
изображение.png 629Кб, 700x445
700x445
>>754876
Луна отделилась, когда в нас врезалась булдыга размером с Марс, и в те времена Земля выглядела как пикрил. А массы Чикшулубского камня недостаточно, чтобы Землю в 16 раз замедлить.
Аноним 09/05/23 Втр 15:17:29 754881 184
>>754851
>Электроны вокруг ядер
Не вращаются. До момента декогеренции у них вообще нет положения в пространстве у ядра.
>ещё и спин имеется, который вроде как ось вращения
Спин элементарных частиц не имеет адекватной аналогии в макромире. Да, частицы ведут себя словно маленькие волчки, вращающиеся в определенном направлении вокруг собственной оси. Но физически они не вращаются - там нечему вращаться. Квантовый спин - свойство иного порядка, нежели просто вращение вокруг оси.
Аноним 09/05/23 Втр 16:54:10 754884 185
>>754881
А если без радуг и единорогов?
Аноним 09/05/23 Втр 18:46:09 754886 186
Аноним 09/05/23 Втр 19:12:54 754887 187
>>754881
>Но физически они не вращаются - там нечему вращаться. Квантовый спин - свойство иного порядка, нежели просто вращение вокруг оси
Плотноприжатые фотоны, йоба - ауууууу.
Это так просто, но пидарасы лучше иной порядок придумают, чем примут истину, понимаю)
Аноним 09/05/23 Втр 19:15:26 754888 188
>>754851
>Томущо раньше на плонете жили большие динозавры, а теперь живут маленькие пчелики. Значит раньше плонета быстрее вращалась. Не?
Не - Раньше планета была меньшего диаметра - поэтому и большие дино жили
Аноним 09/05/23 Втр 19:39:11 754890 189
>>754888
>Не - Раньше планета была меньшего диаметра - поэтому и большие дино жили
Смотри внимательно и запоминай, потому что в следующий раз тебе такое будут объяснять только через 7 лет - в седьмом классе:

Закон всемирного тяготения:
F = GMm/R2
где:
F - сила, с которой два тела притягиваются друг к другу (в данном случае Земля и дино)
G - гравитационная постоянная (6,67 10^-11 Нм2/кг2)
M - масса Земли (5,97 * 10^24 кг)
m - масса дино
R - радиус Земли (6371000 м)

Из этого закона следует, что сила тяготения обратно пропорциональна квадрату расстояния, то есть чем меньше радиус планеты, тем сильнее динозаврика приплющит к земле. Смекаешь?
Аноним 09/05/23 Втр 19:51:11 754891 190
>>754890
>Из этого закона следует, что сила тяготения обратно пропорциональна квадрату расстояния, то есть чем меньше радиус планеты, тем сильнее динозаврика приплющит к земле. Смекаешь?
Смекал сегодня только твою мамку на Луне, когда у неё сиськи расплющило от охуенно большой гравитации.
Аноним 09/05/23 Втр 19:59:25 754892 191
>>754891
Масса Луны меньше Земной в 81 раз, а квадрат радиуса всего в 13, но такое тебе только в старшой школе на астрономии расскажут.
Аноним 09/05/23 Втр 20:09:48 754893 192
>>754892
Когда планета Земля была меньшего диаметра и дино ебали твою тогда ещё юную мамку - масса планеты тоже была меньше, смекаешь?
Аноним 09/05/23 Втр 20:16:34 754894 193
>>754893
Пролистай 90 постов назад. Там уже был спор по поводу увеличивающейся массы Земли.
Аноним 09/05/23 Втр 20:18:44 754895 194
>>754894
> уже был спор по поводу увеличивающейся массы Земли
кто победил?
Аноним 09/05/23 Втр 20:29:30 754897 195
image.png 56Кб, 1011x313
1011x313
Аноним 09/05/23 Втр 20:55:06 754898 196
>>754897
Про увеличение массы за счет питания бактерий - то был вообще какой-то бред сивой кобылы, который даже комментировать не хочется. Земля увеличивается за счет вспышек суперновых, с которых галактическим ветром приносит тяжелые химические элементы. Не?
Аноним 09/05/23 Втр 21:00:52 754899 197
>>754898
Нет. В этом треде уже подсчитывалась масса всего космического всего вещества, которое упало на Землю за 4,5 миллиарда лет. Пролистай и почитай.
Аноним 09/05/23 Втр 21:31:39 754900 198
>>754887
>пидарасы
При чем тут твой папа?
Аноним 09/05/23 Втр 21:44:46 754902 199
>>754898
Не конечно - внутреннее Солнце в центре Земли порождает химические элементы из материальной физической среды. Земля набухает изнутри, масса увеличивается.
Аноним 09/05/23 Втр 21:55:20 754903 200
изображение.png 237Кб, 474x266
474x266
>>754902
Ты сначала экспериментально докажи существование манясреды, а потом мы полую Землю обсудим.
Аноним 09/05/23 Втр 23:04:23 754907 201
>>754902
Как может внутри Земли быть Солнце? Тогда бы все пчелики на поверхности сгорели.
Аноним 09/05/23 Втр 23:06:29 754908 202
>>754903
>докажи существование манясреды
А ты сучка докажи, что манясреды не существует, ушлепок.
Уже сто раз тут доказывал существование физической материальной среды - не нужно тупых опытов с измерением скорости света и тд. Волчок сука, юла, маховик, гироскоп и тд - все они возможны лишь благодаря наличию материальной среды. Они буквально опираются на физическую среду - обнаруживая её. Всё сука - доказал.
>мы полую Землю обсудим
Ты в этом нихуя не шаришь, чё с тобой обсуждать?
Аноним 09/05/23 Втр 23:07:22 754909 203
>>754899
>В этом треде уже подсчитывалась масса всего космического всего вещества, которое упало на Землю за 4,5 миллиарда лет.
А как подсчитывали? Примерно почувствовали?))
Аноним 09/05/23 Втр 23:09:39 754910 204
>>754908
Поясни за Интерферометр Майкельсона.
Аноним 09/05/23 Втр 23:12:02 754911 205
>>754907
Оно достаточно маленькое чтобы пчелики не сгорели не только на поверхности Земли, но и на внутренней сфере Земли.
А вот эти бла бла бла, что Солнце способно существовать до определенного диаметра - правильно, потому что все меньшие Солнца обросли темными пятнами и превратились в планеты. Ты просто их не видишь.Вернее видишь, но не понимаешь, что большинство планет - это Солнца.
Аноним 09/05/23 Втр 23:26:44 754912 206
>>754908
>А ты сучка докажи, что манясреды не существует, ушлепок.
Майкельсон и Морли уже доказали.
А вообще бремя доказательства лежит на утверждающем.

>Ты в этом нихуя не шаришь, чё с тобой обсуждать?
А ты не шаришь за здравый смысл и научный метод. Что с тобой обсуждать?
Аноним 09/05/23 Втр 23:41:39 754913 207
>>754911
>Оно достаточно маленькое чтобы пчелики не сгорели не только на поверхности Земли, но и на внутренней сфере Земли.
Достаточно маленькое - это какое?
Аноним 09/05/23 Втр 23:42:00 754914 208
2023-05-1000-31.png 54Кб, 890x222
890x222
2023-05-1000-28.png 35Кб, 932x205
932x205
>>754909
По самым оптимистичным оценкам на Землю ежегодно падает чуть больше 40 тысяч тонн вещества. Но даже если это количество увеличить в 300000 раз, за 4,5 миллиарда лет нападает всего 1% от массы Земли.
Аноним 09/05/23 Втр 23:43:11 754915 209
>>754910
>Поясни за Интерферометр Майкельсона
У тех кто ставил опыт была неверная модель физической материальной среды и распространения света в рамках этой модели.
Всё что они доказали своим опытом, что в рамках гипотезы, которую они взяли за основу - эфирного ветра в плотных слоях атмосферы Земли нет.
А почему он собственно должен быть и почему отсутствие эфирного ветра в плотных слоях атмосферы Земли должно доказывать отсутствие физической материальной среды?
Также движение плотноприжатых сфер в вихре солнечной системы например - не может отклонять свет, поскольку свет это ЭМ волна, то есть процесс колебательный, представленный переходом импульса от движения буравчиков ответственных за магнитное поле и пронизывающих промежутки между плотноприжатыми сферами к замкнутому кольцевому движению сфер ответственных за электрическое поле и обратно. Кольцевое движение почитай уже частица, хоть и временная, ака виртуальная - но всё же частица и соответственно она не уносится общим потоком сфер поскольку у неё появляется инерция - уходящие в тыл сферы засасывают на фронте новые и тд - вот это вот всё. Буравчик аналогично - но там уже осевое вращение и тд.
Другое дело, если скорость вихря меняется - например в грав поле Земли, изменяется время элементарного взаимодействия, возникает эффект линзирования
Аноним 09/05/23 Втр 23:46:10 754916 210
>>754912
>А ты не шаришь за здравый смысл и научный метод.
Хули - ты не шаришь в логике и дедукции - что с тобой обсуждать?

>Майкельсон и Морли уже доказали.
Что нет макаронного монстра они доказали.
Единой теории эфира/материальной физической среды нет - они опровергли лишь какую-то унылую гипотезу с ветрами из эфира - похуй, это не отрицает факт наличия материальной среды.
Аноним 10/05/23 Срд 00:19:46 754922 211
Нахуя вы опять кормите этого тролля плотноприжатого?
Аноним 10/05/23 Срд 00:22:11 754923 212
>>754915
А почему, например, не может просто не быть среды и пиздец. Ну типа есть частицы, а есть нихуя, которое между ними?
Аноним 10/05/23 Срд 00:27:07 754924 213
>>754923
Потому что плотноприжатый шизик мнит себя самым вумным шерлоком холмсом, который своей дедукцией ставит на колени всех наукоботов ну или он просто малолетний тролль
Аноним 10/05/23 Срд 00:53:45 754926 214
>>754915
Почему именно сферы, а не, скажем, кубики?
Аноним 10/05/23 Срд 10:45:29 754937 215
>>754923
>А почему, например, не может просто не быть среды и пиздец. Ну типа есть частицы, а есть нихуя, которое между ними?
Потому что нихуя - это абсолютно во всём нихуя. Нихуя не может обладать размерностью - нихуя просто нет, как и нет свойств у нихуя. А если нихуя обладает размерностью - то это уже материя.
Это частично признано на оф уровне - потому что у них пространство обладает материальными свойствами, размерностью, может искривляться, скручиваться, сжиматься и при этом всём это типо "нихуя", а ещё в этом "пространстве есть поля различной природы - которых на материальном уровне как бы нет, но они могут взаимодействовать с материей. Да много там костылей - с теми же "квантовыми спинами" и тд.
По факту пространство - это просто область действия, это максимально верное определение. Например пространство на ssd/hdd и тд. Попробуй записать 1Тб в нихуя, а потом считать эти данные, а?
Аноним 10/05/23 Срд 10:59:28 754938 216
>>754926
>Почему именно сферы, а не, скажем, кубики?
Потому что есть две проблемы в поисках верной модели материальной среды:
- абсолютная пустота невозможна, любой объем пространства обладает размерностью и следовательно он материален
- материальная среда не должна быть бетоном, то есть в материальной среде без пустоты должно быть возможно движение.
И эту задачу успешно решил Рене Декарт ещё 400 лет назад.
А с кубиками ты получишь бетон без возможности перемещения в нём. Да и во многих псевдоэфирных теориях как у Ацюковского например - тоже получается бетон без возможности к движению.
Заслуга Декарта в том, что он открыл свойства нашей материальной среды - сохранять форму способны только частицы с осевым вращением. Частицы без осевого вращения бесконечно делятся и сливаются, подстраиваясь под промежутки между плотноприжатыми сферами и формируют идеальную жидкость. Благодаря этому сохраняется возможность к движению в материальной среде без образования пустоты.
Одновременно с этим если некий объём идеальной жидкости приобретает осевое вращение, то он становится частицей материальной среды, а при утрате вращения сливается с идеальной жидкостью.
Аноним 10/05/23 Срд 11:03:19 754939 217
>>754924
>мнит себя самым вумным шерлоком холмсом, который своей дедукцией ставит на колени всех наукоботов
По факту мне насрать на наукоботов - просто у меня обостренная неприязнь ко лжи и прирожденный нюх на ложь. Не удовлетворившись дальнодействием и всей оф херней с костылями я стал искать истину и к счастью нашел её. То есть я всё это делаю исключительно для личного самоудовлетворения - чтобы мой Разум нашел покой в абсолютной истине.
Аноним 10/05/23 Срд 12:49:34 754943 218
>>754937
>Потому что нихуя - это абсолютно во всём нихуя. Нихуя не может обладать размерностью - нихуя просто нет, как и нет свойств у нихуя. А если нихуя обладает размерностью - то это уже материя.
А если, например, нихуя может обладать свойствами, но при это нихуя - это еще не материя?
Аноним 10/05/23 Срд 13:10:53 754944 219
>>754939
Ты уже пришел к Реке?
Аноним 10/05/23 Срд 13:28:17 754945 220
>>754938
>А с кубиками ты получишь бетон без возможности перемещения в нём.
Про "клеточные автоматы" слышал? В кубиках можно замутить движение, если правильно описать правила взаимодействий кубиков.
Аноним 10/05/23 Срд 13:42:57 754946 221
Если мы прямо сейчас даём Луне волшебного пенделя, чтобы она съебалась с орбиты Земли и из солнечной системы вообще, или врезалась во что-нибудь, хоть в Солнце, пытаясь сбежать, но по итогу остави нашу Землю без спутника, как быстро бы наступили изменения на Земле?
Смена погоды?
Смена недель и месяцев?
Как быстро Земля станет приливнозахваченной?
Аноним 10/05/23 Срд 13:43:11 754947 222
где анон смотрит гороскопы?

нужен проверенный источник с точными прогнозами, чтобы я знала когда стоит вложить 200 рублей в бинанс
Аноним 10/05/23 Срд 13:53:20 754948 223
Аноним 10/05/23 Срд 13:59:23 754949 224
>>754947
>где анон смотрит гороскопы?
>
>нужен проверенный источник с точными прогнозами, чтобы я знала когда стоит вложить 200 рублей в бинанс
==> /mg/
Аноним 10/05/23 Срд 14:28:39 754950 225
>>754946
>Смена недель и месяцев?
Месяцы точно исчезнут, ну то есть они в календаре останутся, но уже как-то без души.

Приливы исчезнут это точно.
Аноним 10/05/23 Срд 15:10:51 754951 226
>>754947
>чтобы я знала когда стоит вложить 200 рублей в бинанс
Может, лучше я вложу свой хуй в твой ротанс?
Аноним 10/05/23 Срд 15:15:18 754952 227
Если я буду вращаться на стабильной орбите чёрной дыры выше горизонта событий, имя при себе только ведро, смогу ли толкаясь от ведра по чуть-чуть придавая себе кинетически дополнительную энергию постепенно увеличить радиус своей орбиты, а, в конечном счёте, улететь от чёрной дыры прочь?
Аноним 10/05/23 Срд 15:38:21 754953 228
>>754952
>смогу ли толкаясь от ведра по чуть-чуть придавая себе кинетически
Как Мюнхаузен из болота что-ли?
Аноним 10/05/23 Срд 15:57:37 754954 229
>>754943
>А если, например, нихуя может обладать свойствами, но при это нихуя - это еще не материя?
Свойствами может обладать только материальный объект.
Аноним 10/05/23 Срд 15:59:10 754955 230
>>754945
>В кубиках можно замутить движение, если правильно описать правила взаимодействий кубиков
В кубиках при условии отсутствия абсолютной пустоты? И как же будут перемещаться твои плотноприжатые кубики?
Аноним 10/05/23 Срд 16:48:54 754957 231
>>754954
>Свойствами может обладать только материальный объект.
Не обязательно же.
>В кубиках при условии отсутствия абсолютной пустоты?
Да.
>И как же будут перемещаться твои плотноприжатые кубики?
Кубик слева становится кубиком справа. Кубик справа становится кубиком слева. Считай поменялись местами.
Аноним 10/05/23 Срд 17:42:14 754964 232
>>754953
Ну как бы да, только у гравитации нету вязкости
Аноним 10/05/23 Срд 21:06:21 754974 233
>>754952
От ведра ты сможешь оттолкнуться только 1 раз.
Допустим, при собственной массе 70 кг ты оттолкнёшь от себя ведро массой 1 кг со скоростью 2 м/с. По закону сохранения импульса:
Vобщаяmобщая = Vтелаmтела - Vведраmведра
0
71 = 70V - 2
ты приобретёшь скорость V = (0+2)/70 ≈ 0,029 м/с
Такой скорости недостаточно, чтобы покинуть орбиту
Аноним 11/05/23 Чтв 00:09:53 754980 234
>>754974
>От ведра ты сможешь оттолкнуться только 1 раз.
А нихуя!
Ты в детстве раннем не носил варежки на резиночке?
Вот тут ведро на верёвочке; одно, но многоразового использования - встал на него, отпрыгнул, аккуратно притягиваешь ведёрко веревёчкой.
Аноним 11/05/23 Чтв 00:43:23 754982 235
>>754980
Варежки на резиночке?
Аноним 11/05/23 Чтв 01:03:42 754984 236
По закону сохранения импульса суммарный импульс системы из тебя и ведра должен всегда быть равен нулю.

1. В начальный момент времени ты вместе с ведром находился в состоянии покоя:
p = 0(70+1) = 0

2. В момент, когда ты оттолкнулся от ведра, ты и ведро приобрели такие скорости, при которых ваш суммарный импульс равен нулю:
p = 70х0,029 - 1x2 = 0

2,5. Когда ты будешь тянуть к себе ведро, ты будешь совершать работу по торможению, а потом по перемещению ведра. Эта работа будет равна сумме кинетической энергии ведра и твоей кинетической энергии, которые вы приобрели в момент отталкивания. То есть и ты и ведро будете постепенно терять скорость.

3. В момент, когда ты притянешь к себе ведро и снова встанешь на него, ваш общий импульс будет равен:
p = V(70+1) = 0
А это возможно только при V = 0

Если бы реактивное движение в космосе требовало так же мало ресурсов, как ведро-верёвочный двигатель, люди бы уже давно облетели всю солнечную систему и отправились к соседним звёздам.
Аноним 11/05/23 Чтв 01:05:13 754985 237
Аноним 11/05/23 Чтв 01:20:44 754986 238
>>754984
>>754982
Варежки на резиночке, конечно
Я просто помню, что есть любители прыгнуть с недостроя или заброшки - у них же инерция уходит не прямо в низ по троссу в землю, а вбок, по отводному тросу. Неужели и тут нельзя схитрить, перенаправив инерцию от возврата ведра так, чтобы она в общем была меньше, чем при прыжке или сама по себе, или возвращать по вектору так, чтобы возвратный вектор был под углом и оказывал потому меньшее влияние на человек, чем ведро при оттталкивании
Аноним 11/05/23 Чтв 02:32:25 754989 239
>>754937
Везде есть поля браны струны, прост кое где есть из флуктуации или концы струн а где-то нет, вот где нет там типа вакуум, но это не значит что там колебаний и соответственно чело либо не появится, просто это очень редкое событие. И в эти поля струны браны мы не можем посмотреть, там коллайдер нужен с солнечную систему чтобы потыкать планковские расстояния.
Аноним 11/05/23 Чтв 07:36:05 754993 240
>>754986
Нельзя. Это уже будет нарушать закон сохранения энергии.
Аноним 11/05/23 Чтв 07:46:55 754994 241
ere4d3.jpg 33Кб, 600x600
600x600
>>754989
>Везде есть поля бараны струны
Что?
"Поле" это вообще мем конечно
Что значит сука поле - с хуя оно взялось? Какая у него природа - материальная или паранормальная?
Аноним 11/05/23 Чтв 07:53:41 754995 242
kub-kirpicha.jpg 18Кб, 500x400
500x400
>>754957
>Не обязательно же.
В каком смысле не обязательно? Разве в нашей Вселенной возможны нематериальные объекты со свойствами? Реально какая-то паранормальная херня с полтергейстами.
>Да.
Что мать твою да?
возьми куб кирпичей и задумайся о движении внутри него

>Кубик слева становится кубиком справа. Кубик справа становится кубиком слева. Считай поменялись местами.
чтобы кубики поменялись местами - они должны сдвинутся - а сдвинутся им не дадут соседние кубики - потому что они все плотноприжаты, ведь абсолютной пустоты не существует
Аноним 11/05/23 Чтв 13:02:26 754998 243
>>754995
>В каком смысле не обязательно? Разве в нашей Вселенной возможны нематериальные объекты со свойствами?
Например возможны. Не?
>Реально какая-то паранормальная херня с полтергейстами
В конечном итоге в любой теории можно докопаться до паранормальнтй херни. Ну вот, например, у тебя в твоих сферах что внутри находится? Какая-то "идеальная жидкость"? А что это за полтергейст такой? А между сферами что находится? "Граница"? Еще один полтергейст.
Вся физика, она в конечном итоге упирается в метафизику. Чому зарядов всего 2 (положительный и отрицательный), а не 3? Давай придумаем третий заряд и будем под него все теории подстраивать.

>Что мать твою да?
>возьми куб кирпичей и задумайся о движении внутри него
>чтобы кубики поменялись местами - они должны сдвинутся - а сдвинутся им не дадут соседние кубики - потому что они все плотноприжаты, ведь абсолютной пустоты не существует
Пример с кирпичами - некорректный. Кирпич - это сложный объкт. Плотноприжатый кубик - это объект элементарный, обладающий элементарным свойством. Чтобы поменять два плотноприжатых кубика местами - надо просто поменять их свойства. Ну то есть бля. Есть у тебя маленький элементарный кубик, у него есть одно единственное свойство "Q". Это свойство полностью описывает состояние кубика, других свойств у кубика нет. Скажем, у кубика слева Q=u. У кубика справа Q=s. Кубики меняются свойствами. Теперь у кубика слева Q=s, а у кубика справа Q=u. Поменявшись свойствами, кубики поменялись местами - томущо у кубиков нет никаких других свойств, которые идентифицировали бы кубик.
Аноним 11/05/23 Чтв 16:05:11 755010 244
>>754998
>Например возможны. Не?
Нет конечно. Какие у тебя могут быть примеры нематериальных объектов? Призраки? Демоны?

>Ну вот, например, у тебя в твоих сферах что внутри находится? Какая-то "идеальная жидкость"?
Нет Маня, ты всё напутал.
И сферы, и идеальная жидкость - образованы из первичной материи, которая заполняла всё пространство без образования пустоты. Затем в эту материю было вложено движение, ака большой взрыв. Первоматерия раздробилась на множество частиц - но в отсутствии пустоты им улететь было некуда, а двигаться сука надо. Поэтому они стали двигаться в замкнутых кольцевых вихрях ака солнечные системы. Частицы сточили свои края и стали сферами, а осколки заполнили промежутки между сферами и центры вихрей солнечных систем, что теперь называется Солнцем.
Да сука - это 100% компьютерная симуляция, да сука наш мир виртуальный. Но он существует независимо и самодостаточно.

>А что это за полтергейст такой?
Под паронормальной хернёй я подразумеваю нематериальные объекты с присущими им свойствами и размерностями. В нашей Вселенной такие объекты невозможны - это очевидно.

>Чому зарядов всего 2 (положительный и отрицательный), а не 3?
Потому что заряды обусловлены вращением плотноприжатых сфер по часовой стрелке или против - третьего не дано как бы)))

>В конечном итоге в любой теории можно докопаться до паранормальнтй херни.
Нет - в чисто механической теории близкодействия с физической материальной средой - паранормальной херни нет.

>Чтобы поменять два плотноприжатых кубика местами - надо просто поменять их свойства. Ну то есть бля.
Не то есть - наша Вселенная построена на чистой механике и кубик есть кубик - паранормального не дано. Чтобы сдвинуть кубик - ту должен переместить его физически - никакой мистики бля - поэтому пример с кирпичами вполне корректный.
Аноним 11/05/23 Чтв 18:04:36 755012 245
>>754994
Так все одно материя энергия, это возмущение полей
Аноним 11/05/23 Чтв 22:40:40 755019 246
>>755012
>Так все одно материя энергия, это возмущение полей
Чёт ты всё в кашу смешал
Физически во Вселенной существует только материя.
Энергия это мера движения, в тч частиц материальной среды.
Поле это характеристика состояния частиц материальной среды
Аноним 12/05/23 Птн 02:02:40 755026 247
>>755019
Швингер бы тебя прибил
Аноним 12/05/23 Птн 02:16:28 755028 248
>>755019
Ты в заморозке был последние полтора века? Уже давно все разложено на субатомные частицы, многие являются проводниками взаимодействий, т.е. твоих движений в том числе. На ещё более мелком масштабе это уже поля струны браны, сейчас никто не знает точно.
Аноним 12/05/23 Птн 08:35:50 755031 249
>>755026
>Свингер бы тебя прибил
А кто это?
Аноним 12/05/23 Птн 08:43:32 755032 250
>>755028
>Ты в заморозке был последние полтора века? Уже давно все разложено на субатомные частицы,
Не - всё херня, если их гипотезы допускают абсолютную пустоту в сколь угодно малом объеме пространства.
Такое чувство, что это какие-то жалкие попытки в каргокульт и подражание Декарту.
>На ещё более мелком масштабе это уже поля струны браны, сейчас никто не знает точно.
400 лет назад Декарт знал, а сейчас никто не знает - ну заебись блядь деграданты.
Аноним 12/05/23 Птн 12:09:51 755036 251
>>755032
>400 лет назад Декарт знал, а сейчас никто не знает
Вся рота идет не в ногу, один поручик шагает в ногу
Аноним 12/05/23 Птн 12:34:47 755037 252
6d.png 234Кб, 1200x900
1200x900
>>755010
>Нет конечно. Какие у тебя могут быть примеры нематериальных объектов? Призраки? Демоны?
Есть материя, а есть отсутствие материи - пространство. Это как, скажем, есть свет, а есть отсутствие света - темнота. Но ты же не можешь сказать, что темнота - это тоже такой свет, только темный)) Пространство - это такое отсутствие материи, это не "пустая материя", а эт её (материи) прост нет - она отсутствует))

>Первоматерия раздробилась на множество частиц
Первоматерия - это что за полтргейст такой? А есть ещё примеры таких полтргейстов? Призраки? Демоны?

>Да сука - это 100% компьютерная симуляция, да сука наш мир виртуальный. Но он существует независимо и самодостаточно.
Виртуальный мир - дискретный. В дискретном мире граница у твоих плотноприжатых сфер не гладкая, а лесенкой... кстати, а как ты сферы собрался плотноприжимать? Если плотноприжать две сферы - на границе будет точка, причем бесконечно маленькая. Сферы плотно не прижимаются. Плотно, можно прижать только кубики.

>нематериальные объекты с присущими им свойствами и размерностями. В нашей Вселенной такие объекты невозможны - это очевидно.
Это такая типа догма?))

>Потому что заряды обусловлены вращением плотноприжатых сфер по часовой стрелке или против - третьего не дано как бы)))
Пикрелейтед с тобой поспорит.

>Нет - в чисто механической теории близкодействия с физической материальной средой - паранормальной херни нет.
Хватает, есличесн))

>Чтобы сдвинуть кубик - ту должен переместить его физически
Не обязательно. Если наш мир - это "100% компьютерная симуляция, да сука наш мир виртуальный", ттгда и кубики можно перемещать виртуально. Не?
Аноним 12/05/23 Птн 13:57:14 755038 253
>>755037
>Есть материя, а есть отсутствие материи - пространство. Это как, скажем, есть свет, а есть отсутствие света - темнота.
Пространства без материи не существует. Пространство это просто область действия, заданная материей. Для определения границ пространства нужен материальный объект с размерностью.
Я тебе уже предлагал записать 1Тб данных в ничто, а потом из ничего считать эти данные. Так не получится, это очевидно. Нам необходимо свободное пространство, например на жестком диске. Свободным его считают от того, что там отсутствуют полезные данные. Но по факту весь жесткий уже заполнен нулями, то есть он не пустой, это не ничто - это абсолютно такие же данные.
Аналогично и с нашей Вселенной - всё пространство было заполнено первоматерией, которая и определяет границы этого пространства - области действия. Частицы этой первоматерии могут бесконечно делиться, приобретая движение и сливаться, утрачивая его. То есть именно движение некоего объема первоматерии и определяет частицу. Всю эту идеальную жидкость с бесконечным делением и слиянием можно представить как свободную область на жестком диске. Чтобы частица стала постоянной, не утрачивала движение и не сливалась с остальными - она должна иметь осевое вращение / спин. Это может быть как осевое вращение сферы - постоянная частица, так и поступательное осевое вращение буравчика, который навинчивается в промежутки между сферами. Буравчик менее постоянный - потому что при утрате поступательного движения он потеряет и осевое. А вот сфера может стоять на месте и наяривать круги.
Так вот вся ошибка оф физики в иноре того факта, что всё пространство по факту состоит из материи. Они признают лишь определяемые составные частицы, забывая про материальную физическую среду. Это аналогично тому, что называть свободное место на жестком диске абсолютной пустотым - это глупо. А ошибка исходит из пристрастия к мат моделям - где ради упрощения выкинули физическую среду, заменив её костылями по типу полей и тд - то есть локальными характеристиками состояния физической среды.

>Первоматерия - это что за полтргейст такой? А есть ещё примеры таких полтргейстов? Призраки? Демоны?
Первоматерия не полтергест потому что она физически существует и обладает свойствами.
Под Полтергестом подразумевается свойства без физического объекта, который их определяет - например поля. Свойства есть, взаимодействие есть - а физического объекта нет - это явный Полтергест.
Аноним 12/05/23 Птн 14:20:00 755039 254
>>755037
>Виртуальный мир - дискретный. В дискретном мире граница у твоих плотноприжатых сфер не гладкая, а лесенкой...
Маня не знает про векторную графику?

>кстати, а как ты сферы собрался плотноприжимать? Если плотноприжать две сферы - на границе будет точка, причем бесконечно маленькая. Сферы плотно не прижимаются.
Естественно между плотноприжатыми сферами будут зазоры, заполненные идеальной жидкостью - которая тоже материальна. А также в зазоры между сферами будут навинчиваться материальные частицы-буравчики и тд. Материальная среда состоит не только из плотноприжжатых сфер, но и из целой гаммы различных частиц.

>Это такая типа догма?
Это элементарная логика.

>Пикрелейтед с тобой поспорит.
Плоскость вращения задается плоскостью вращения вихря солнечной системы.

>Не обязательно. Если наш мир - это "100% компьютерная симуляция, да сука наш мир виртуальный", ттгда и кубики можно перемещать виртуально. Не?
Наш Мир хоть и виртуален, но самодостаточен - он механический, с геометрическими взаимодействиями - без возможности влиять на отдельную частицу или её свойства.
Аноним 12/05/23 Птн 14:23:37 755040 255
>>755039
Объясни для чего в твоей теории нужны идеальная жидкость и частицы-штопоры, почему нельзя обойтись плотноприжатыми фотонами.
Аноним 12/05/23 Птн 14:56:06 755041 256
>>755040
>Объясни для чего в твоей теории нужны идеальная жидкость и частицы-штопоры, почему нельзя обойтись плотноприжатыми фотонами.

Потому что так устроен наш Мир. Без идеальной жидкости возникнет пустая нематериальная область в промежутках между сферами, исчезнет Солнце как центр вихря из плотноприжатых сфер - заполненный идеальной жидкостью.
Без буравчиков исчезнет магнитное поле, ЭМ поле и тд. Потом есть и иные типы частиц из первоматерии - наш Мир многогранен. Например движущаяся замкнутая лента - чем не частица? Я уже не говорю про частицы материи без осевого вращения, которые крупнее отдельной сферы - из-за этого сферы геометрически не могут разрушить такую частицу, обвивают её замкнутым кольцевым движением что порождает центры атомов.
Аноним 12/05/23 Птн 15:22:45 755043 257
>>755041
>Без идеальной жидкости
У тебя идеальная жидкость на губах там. Вытри, а то некрасиво как-то.
Аноним 12/05/23 Птн 15:39:11 755044 258
>>754984
> p = 70х0,029 - 1x2 = 0
вообще-то 70х0.029=2.03, 1х2=2, и 2.03-2=0.03

слит и обоссан
Аноним 12/05/23 Птн 16:29:24 755045 259
А что по программам заселения других планет, как обстоят дела?
Аноним 12/05/23 Птн 16:31:02 755046 260
>>755043
>идеальная жидкость на губах
Моя идеальная жидкость на губах мамки эйнштейноеба
Аноним 12/05/23 Птн 16:45:20 755047 261
>>754530
ну смотри, плотность тамошнего вещества - около 10^12 кг/см^3, т.е.1000'000'000'000 (триллион) килограмм используя закон всемирного тяготения этот см^3 будет тянуть к ядру земли с силой 1.64433580479315 × 10^18 Ньютон что даёт нам понять что он просто пробъёт землю до ядра практически моментально и самое главное сама сила в 100'000'000'000 (100 миллиардов) раз больше чем 100 килограммового человека.
Аноним 12/05/23 Птн 17:08:13 755048 262
Аноним 12/05/23 Птн 17:18:08 755049 263
Screenshot20230[...].png 102Кб, 1080x2340
1080x2340
>>755044
Читай внимательнее >>754974
>V = (0+2)/70 ≈ 0,029 м/с
>≈ 0,029 м/с
>≈
Аноним 12/05/23 Птн 18:01:38 755051 264
>>755048
мамка твоя слизала
Аноним 12/05/23 Птн 18:07:18 755052 265
>>755047
>ну смотри, плотность тамошнего вещества - около 10^12 кг/см^3, т.е.1000'000'000'000 (триллион) килограмм
Как измеряют массы нейтронных звезд?
Аноним 12/05/23 Птн 18:53:57 755054 266
>>755038
>Пространства без материи не существует.
А если, например, существует?
>Пространство это просто область действия, заданная материей. Для определения границ пространства нужен материальный объект с размерностью.
Есть один материальный объект, есть другой материальный объект, а между ними пустота. Ну, то есть, нихуя нет, например. Не?

>Я тебе уже предлагал записать 1Тб данных в ничто, а потом из ничего считать эти данные. Так не получится, это очевидно. Нам необходимо свободное пространство, например на жестком диске.
Если тебе так нравится пример с жестким диском - го вернемся к плотноприжатым кубикам. Берешь фоточку с одного диска, переносишь её на другой диск. У тебя фотка переместилась, теперь она физически на другом диске, но при это никакого физического движения не было, стало быть кубики победили. Не?

>Первоматерия не полтергест потому что она физически существует и обладает свойствами.
Пустота не полтергейст, потому что она физически существует, например. Не?
Аноним 12/05/23 Птн 18:55:06 755055 267
IMG202305121919[...].jpg 2985Кб, 3456x4608
3456x4608
IMG202305121921[...].jpg 2260Кб, 2962x4403
2962x4403
>>755052
1. Через закон всемирного тяготения и третий закон Кеплера, если у звезды есть компаньон или если осталась туманность после взрыва сверхновой. (см пикрилы)

2. С помощью измерения эффекта гравитационного линзирования, когда лучи света от отдалённых звёзд проходят через области пространства с сильной гравитацией звезды.
Аноним 12/05/23 Птн 19:33:20 755056 268
>>755039
>Маня не знает про векторную графику?
Хз, что за Маня, которого ты второй раз упоминаешь. Воображаемый друг?

>не знает про векторную графику?
Векторная графика, в конечном итоге, аппроксимируется в дискретную. Всё в компьютере дискретно: пиксели на экране - дискретные, ячейки памяти - дискретные, транзисторы в процессоре - дискретные. Вся векторная графика - это мы такие типа договорились, что вот, мол, координаты центра сферы, вот ейный (сферы) радиус и договорились, что эта фигура - сфера. А дальше пытаемся создать дискретную модель этой сферы с приемлемой степенью достоверности. Но если ты такую же фигню хочешь на реальный мир натянуть - получится идеализм. Мы же тут, вроде как, материалисты все, не?

>Материальная среда состоит не только из плотноприжжатых сфер, но и из целой гаммы различных частиц.
Не только сферы - это типа там и кубики есть?

>Это элементарная логика.
Да нет же. Свое утверждение о невозможности существования нематериального объекта с свойствами, ты никак не пытаешься логически доказать, а используешь это утверждение, как нечто само собой разумеющееся, не требующее доказательств - догму. Не?

>Плоскость вращения задается плоскостью вращения вихря солнечной системы.
То есть, все сферы вращаются в одной плоскости?

>Наш Мир хоть и виртуален, но самодостаточен - он механический, с геометрическими взаимодействиями - без возможности влиять на отдельную частицу или её свойства.
Почему твои сферы в микроскоп не видно?
Аноним 12/05/23 Птн 19:39:11 755057 269
>>755055
>Через закон всемирного тяготения и третий закон Кеплера, если у звезды есть компаньон или если осталась туманность после взрыва сверхновой. (см пикрилы)
А нейтронные звезды как-то в телескопы наблюдают? Фотки есть?
Как их вообще обнаруживают?
Ну вот допустим обнаружили две херни диаметром 20 км, которые вращаются вместе. Как рассчитать их массы без наугад взятой плотности?

>С помощью измерения эффекта гравитационного линзирования, когда лучи света от отдалённых звёзд проходят через области пространства с сильной гравитацией звезды.
А вдруг есть иные причины для линзирования?
Ну не знаю - например сильный заряд, сильное магнитное поле и тд
Аноним 12/05/23 Птн 19:49:56 755058 270
>>755057
>А нейтронные звезды как-то в телескопы наблюдают?
Чаще всего видна оболочка, которая довольно ярко светит в радио, УФ, рентгене и гамме.
>Фотки есть?
Полно, но они не в оптике. Опять из-за оболочки.
>Как их вообще обнаруживают?
Обзорами в радио или гамма диапазоне.
Первые нейтронные звезды вообще открыли когда пытались найти маленьких зеленных человечков.
>Как рассчитать их массы без наугад взятой плотности?
Данные по спектру восстанавливают скорость по времени, а из нее вычисляют период и радиус обриты. Далее вступает в силу закон Кеплера.

>А вдруг есть иные причины для линзирования?
Нет, разве что еще только Черные дыры или холодные белые карлики.
Аноним 12/05/23 Птн 19:56:14 755059 271
>>755057
>А нейтронные звезды как-то в телескопы наблюдают? Фотки есть?
>Как их вообще обнаруживают?
В радиодиапазоне наблюдают. В радиотелескоп их видно как мощные точечные радиоисточники, которые зачастую ещё и пульсируют.

>Ну вот допустим обнаружили две херни диаметром 20 км, которые вращаются вместе. Как рассчитать их массы без наугад взятой плотности?
Плотность тут не нужна. Находят расстояние между ними и их период обращения, подставляют в формулу, получают массы.

>А вдруг есть иные причины для линзирования?
>Ну не знаю - например сильный заряд, сильное магнитное поле и тд
Электрические и магнитные поля могут изменять траектории только заряженных частиц, а фотоны не имеют заряда.
Аноним 12/05/23 Птн 20:01:53 755060 272
>>755056
>Хз, что за Маня
Хуй знает, пидр какой-то любит это слово, вот я его теперь и называю - Маня
>>754654
>наличие маня

> Всё в компьютере дискретно
Дискретность компьютера обусловлена дискретностью хим элементов из которых он состоит.
> дальше пытаемся создать дискретную модель этой сферы с приемлемой степенью достоверности.
Чтобы вывести на монитор, который тоже дискретный.
Ты высосал проблему из пальца.
Наш Мир виртуальный - потому что он основан на геометрических взаимодействиях отдельных объемов первоматерии. Дискретность здесь проявляется в делении первоматерии на отдельные элементы, а не в n-ом блядь числе пикселей на твоем мониторе.
Всё основано на объемах, геометрических формах и присущих им движениях.

>Не только сферы - это типа там и кубики есть?
Для тебя -всё что угодно, но основу материальной среды составляют плотноприжатые сферы.

>Свое утверждение о невозможности существования нематериального объекта с свойствами
Я не вижу смысла здесь что-то доказывать - это очевидно даже младенцу. "Ничто" не может обладать свойствами - потому что "свойства" - это понятие, применительное к объекту, разве нет?

>все сферы вращаются в одной плоскости?
Вокруг общей оси

>Почему твои сферы в микроскоп не видно?
Потому что сферы и буравчики являются физ средой для распространения ЭМ волн
Аноним 12/05/23 Птн 20:11:56 755061 273
>>755059
>а фотоны не имеют заряда.
Фотоны не вращаются? Как они тогда вообще существуют?
Аноним 12/05/23 Птн 20:19:26 755062 274
Без имени-2.png 343Кб, 5928x6224
5928x6224
>>755054
>А если, например, существует?
Нет

>Есть один материальный объект, есть другой материальный объект, а между ними пустота. Ну, то есть, нихуя нет, например. Не?
Если у тебя между объектами нихуя нет - то эти два объекта плотноприжаты друг к другу.
Если у тебя между объектами есть третий объект с размерностью, определяющий расстояние между двумя первыми - то между ними не пустота.

>Берешь фоточку с одного диска, переносишь её на другой диск.
Физическим взаимодействием
>У тебя фотка переместилась, теперь она физически на другом диске, но при это никакого физического движения не было
Было физическое взаимодействие - а какие механизмы позволяют физически взаимодействовать с твоими кубиками для изменений их свойств?

>Пустота не полтергейст, потому что она физически существует, например. Не?
Если ты пустотой называешь материальный объект с размерностью, то это уже не пустота - а материальный объект.
Аноним 12/05/23 Птн 20:34:19 755063 275
>>755058
>Чаще всего видна оболочка, которая довольно ярко светит в радио, УФ, рентгене и гамме.
То есть есть объекты с оболочками, которые " ярко светит в радио, УФ, рентгене и гамме" - а всё остальное про нейтронные звезды притянуто за яйца?

>Данные по спектру восстанавливают скорость по времени, а из нее вычисляют период и радиус обриты. Далее вступает в силу закон Кеплера.
А гравитационная постоянная может же быть разной в разных солнечных системах, если скорости вихрей солнечных систем разные.

>Нет, разве что еще только Черные дыры или холодные белые карлики.
Почему нет? Скорость света зависит от времени элементарного взаимодействия, а оно зависит от скорости вращения вихря солнечных систем.
Аноним 12/05/23 Птн 20:45:32 755064 276
>>755063
>постоянная может же быть разной
>скорости вихрей солнечных систем
Спейсач пора переименовывай в шизач...
Аноним 12/05/23 Птн 20:52:47 755065 277
>>755051
Чего такой злой? Не понравилась идеальная жидкость на вкус?
Аноним 12/05/23 Птн 20:56:40 755066 278
>>755060
Не в курсе, что у вас тут за мани - не заходил в тред с прошлого года.

>Дискретность компьютера обусловлена дискретностью хим элементов из которых он состоит.
Если твоя первоматерия может бесконечно делиться - почему атомы химических элементов обладабюют определенным размером? Почему они не могут быть в два раза меньше, например? Что вообще определяет размеры объекта? Планковская длина - минимальное физически возможное расстояние. Бесконечного деления быть не может - иначе будут частицы, расстояние между которыми будет меньше планковской длины. И, опять же, в модели с бесконечно малым, размеры частиц могут произвольными, то есть, свойство "размер" оно ничем не определяется, кроме как "так зазотелось". Есть самая маленькая частица, меньше которой быть не может. Самая маленькая частица не может быть сферой - тогда между плотноприжатыми сферами можно запихнуть сферу меньшего размера. Стало быть, самая маленькая частица - это кубик, томущо единственное платоново тело, которым можно замостить трехмерное пространство - гексаэдр, он же куб. Опять же плотноприжатые кубики победили.

>Для тебя -всё что угодно, но основу материальной среды составляют плотноприжатые сферы.
Плотноприжатые кубики

> "Ничто" не может обладать свойствами - потому что "свойства" - это понятие, применительное к объекту, разве нет?
К отсутствию объекта тоже. Например, у физиков есть квазичастицы, которые ведут себя как и обычные частицы и обладают свойствами.

>Вокруг общей оси
Подожди, совсем щапутал( Заряд откуда берется? От того, что все сферы кубики вращаются вокруг одной общей оси? Положительно заряженные кубики вращаются по часовой, отрицательно - против. А они друг другу вращаться не мешают?

>Потому что сферы и буравчики являются физ средой для распространения ЭМ волн
Ну, то есть, эти сферы нельзя увидеть, а можно только "почувствовать"?
Аноним 12/05/23 Птн 21:16:39 755067 279
>>755064
А мочератору похуй.
Аноним 12/05/23 Птн 21:25:48 755068 280
>>755062
>Было физическое взаимодействие - а какие механизмы позволяют физически взаимодействовать с твоими кубиками для изменений их свойств?
Вот чтобы сейчас пальчиком на телефоне не рисовать, го на твоей же зеленой картинке сейчас самую простую дискретную модель Вселенной сделаем.
Черные клеточки - это клеточки с минимальными возможными размерами (в трехмерной модели будут кубиками). Все клеточки одинакового размера. Не существует клеточек большего или меньшего размера. Все клеточки обладают одним единственным свойством - цветом. Черные клеточки - это "объекты". Синие клеточки - это такие же дискретные объекты, как и черные. Те же размеры, но другой цвет. Синие. "Пустота". Движение в нашей модели дискретное. Черная клеточка может переместиться (дискретно, одним прыжком) в соседнюю синюю клеточку (но не в черную - там уже занято). Если на твоей картинке черная клеточка "2" перепрыгнет на место синей клеточки - тогда там, где до этого была клеточка "2", теперь там будет синяя клеточка. Плотноприжатые клеточки поменялись местами.
Аноним 12/05/23 Птн 22:16:46 755070 281
Что будет если повредить ядро земли? А уничтожить?
Аноним 12/05/23 Птн 22:21:35 755071 282
>>755070
Магнитное поле пропадёт -> человеки вымрут после нескольких магнитных бурь на Солнце
Аноним 12/05/23 Птн 22:31:23 755072 283
>>755071
А представим что не пропадет. Что тогда?
Аноним 12/05/23 Птн 23:00:38 755075 284
Аноним 12/05/23 Птн 23:01:33 755076 285
>>755072
Без геотермической энергии ядра пропадут конвекционные потоки в мантии. По идее, должна снизиться тектоническая активность, литосферные плиты перестанут двигаться, больше не будут образовываться и пробуждаться вулканы, прекратиться изменение ландшафта планеты.
Аноним 12/05/23 Птн 23:10:00 755079 286
>>755070
Что значит уничтожить? Закон сохранения энергии как бы. Значить, ты затратишь кучу энергии, чтобы ядро опять же излучило энергию в пространство, так? Этот процесс уже возымеет кучу последствий, при которых само по себе наличие или отсутствие целого ядра будет уже ни при чем
Аноним 12/05/23 Птн 23:26:14 755083 287
>>755066
>почему атомы химических элементов обладабюют определенным размером? Почему они не могут быть в два раза меньше, например? Что вообще определяет размеры объекта?
Определяет диаметр плотноприжатых сфер в вихре солнечной системы. Поскольку преимущественно плотноприжатые сферы своими взаимодействиями формируют частицы с зарядом и атомы.

>Планковская длина - минимальное физически возможное расстояние. Бесконечного деления быть не может - иначе будут частицы, расстояние между которыми будет меньше планковской длины.
Планковская длина относится к взаимодействиям уровня "плотноприжатые сферы/ буравчики и тд" как переносчиков ЭМ волн, причину гравитации, инерции и тд.
На уровне идеальной жидкости, которая заполняет промежутки между сферами нет ни гравитации, ни света, ни массы, ни инерции. Есть лишь форма частицы, объём и присущее ей движение. Если частицы утрачивают движение - то они сливаются. И напротив - любому объему идеальной жидкости можно придать движение - и эта частица будет существовать покуда у неё есть движение.

>Плотноприжатые кубики
Бетон в котором невозможно движение.

>Например, у физиков есть квазичастицы, которые ведут себя как и обычные частицы и обладают свойствами.
Это лишь демонстрирует их недоразвитость. Первобытные люди тоже многие явления объясняли мистикой.
Однако наш Мир полностью механический с геометрическими взаимодействиями.

>Подожди, совсем щапутал( Заряд откуда берется? От того, что все сферы кубики вращаются вокруг одной общей оси?
Я тебе писал про ось вихря из плотноприжатых сфер в солнечной системе. Теперь в вихрь мы помещаем частицу более крупную, чем сферы - сферы начинают вращаться вокруг этой частицы из-за разности скоростей в наиболее приближенной и наиболее удаленной точке от оси солнечной системы. Естественно, что оси параллельны.

>Ну, то есть, эти сферы нельзя увидеть, а можно только "почувствовать"?
>ну то есть молекулы воздуха нельзя услышать, а можно только "почувствовать"?
Ты совсем дурак, да?
молекулы воздуха нельзя услышать, сферы нельзя увидеть - они переносчики.
Аноним 12/05/23 Птн 23:29:12 755084 288
>>755068
>го на твоей же зеленой картинке сейчас самую простую дискретную модель Вселенной сделаем.
Вселенная не дискретна, придурок! Дальше даже твой бред не стал читать - дискретен твой монитор или принтер, но не Вселенная на уровне первоматерии.
Аноним 13/05/23 Суб 02:59:48 755087 289
Откуда вылез плотноприжатый долбоеб? Давно такого шизла не видел, будто попал в интернет 2000-х.
Аноним 13/05/23 Суб 04:15:12 755089 290
Аноним 13/05/23 Суб 09:26:04 755090 291
1682803946897977.png 1933Кб, 3404x2723
3404x2723
Можно ли силой Архимеда создать имитацию планетоида на самой верхней кромке атмосферы? Какой-нибудь гигахрущ из графеновых трубок, сплетённых, как однослойная ткань и этой ткань берём большой кусок пространства (размером с планетоид) и заливаем гелием или водородом
Аноним 13/05/23 Суб 09:58:10 755091 292
>>755090
Большие воздушные шарики не могут быть, тупо в силу в того, что сила архимеда экспоненциально падает с высотой. И парусность растет быстрее чем выталкивающая сила - с определенного размера ветровые нагрузки будут сильно деформировать шар.
Другая проблема нужно нагревать шар внутри, а иначе быстро остынет, упадет давление и объем.
Водород и гелий быстро диффундирует через тонкие пленки.
Аноним 13/05/23 Суб 10:06:31 755092 293
>>755091
А если гипотетическая непроницаемая даже для водорода ткань? И если её заполнить водородом или гелием так, чтобы даже при охлаждении она бы не могла сдуть больше какого-то несущественного уровня? Ну просто насувать атомов, как килек в банку или как японцев в японское метро утром?
Аноним 13/05/23 Суб 10:37:36 755093 294
Аноним 13/05/23 Суб 10:41:52 755094 295
>>755092
>А если гипотетическая непроницаемая даже для водорода ткань?
Нет. Есть только у которых скорость диффузии маленькая, но не нулевая.

Чем больше газа накачал, тем меньше будет подъемная сила. Газ тоже внезапно весит. Чем разряженнее газ тем лучше, поэтому в аэростатах газ сильно подогревают и делают стенки шара с очень низкой теплопроводностью.
Давление же нужно чтобы шар держал объем. Если его стенки достаточно жесткие, то можно обойтись вакуумом, но таких материалов еще нет.
Аноним 13/05/23 Суб 10:42:52 755095 296
>>755093
Пиндоссов наказывает за бездуховность боженька?
Аноним 13/05/23 Суб 12:49:35 755097 297
>>755089
А нахуя вы ему отвечаете?
Аноним 13/05/23 Суб 14:02:20 755105 298
15270858867183 [...].jpg 89Кб, 720x544
720x544
Аноним 13/05/23 Суб 14:04:33 755106 299
>>755105
Можно. Но нахуя. Он весь тред засрал.
Аноним 13/05/23 Суб 22:50:54 755122 300
Вот говорят - солнце садится. Куда садится, если земля плоская?
Аноним 13/05/23 Суб 22:57:46 755123 301
>>755122
Солнце садиться за горизон который возможет только у шарообразной Земли
Аноним 13/05/23 Суб 23:05:53 755124 302
плоская.mp4 3506Кб, 720x304, 00:00:30
720x304
>>755122
>от говорят - солнце садится. Куда садится, если земля плоская?
За плоскую Землю садится, уходит за другую сторону блина
Аноним 13/05/23 Суб 23:07:05 755125 303
>>755123
> горизон который возможет только у шарообразной Земли
Горизонт возможен и у тарелки
Аноним 13/05/23 Суб 23:23:00 755127 304
Аноним 13/05/23 Суб 23:43:14 755128 305
изображение.png 111Кб, 1280x853
1280x853
>>755125
Конечно-конечно...
Аноним 14/05/23 Вск 08:20:42 755132 306
горизонт.jpg 109Кб, 1500x1500
1500x1500
Аноним 14/05/23 Вск 08:31:15 755133 307
>>755132
Тогда бы расстояние до горизонта уменьшалось с приближаением к краю Земли, а угол между лучом зрения и плоскостью "планеты" - увеличивался. Но в реальности такого не происходит.
Аноним 14/05/23 Вск 08:48:53 755134 308
>>755133
>Тогда бы расстояние до горизонта уменьшалось с приближаением к краю Земли, а угол между лучом зрения и плоскостью "планеты" - увеличивался. Но в реальности такого не происходит.
Почему?
Ну в том смысле, что у плоскоземельщиков же с краю ледяная стена, а перед ней льды и вот это вод всё.
А видимый горизонт можно объяснить загрязненностью воздуха и тд, что ограничивает зрительное восприятие и сохраняет угол между лучом зрения и плоскостью "планеты"
Аноним 14/05/23 Вск 08:50:02 755135 309
Толстяк.png 94Кб, 250x250
250x250
Аноним 14/05/23 Вск 08:55:03 755137 310
>>755135
А ты докажи, что не плоская!
Аноним 14/05/23 Вск 10:12:43 755139 311
>>755137
Это, если подумать, довольно сложно. Единственный наглядный способ доступный для простого обывателя - наблюдать за объектами в море с разной высоты. И если он живет не во Владивостоке, а в Новосибирске, то Земля будет плоской.
Аноним 14/05/23 Вск 14:44:31 755149 312
>>755137
Я лучше тебе в ротешник нассу. Подставляй!
Аноним 14/05/23 Вск 14:45:17 755150 313
>>755139
Сложно твою мамку ебать, она слишком толстая.
Аноним 14/05/23 Вск 14:48:25 755151 314
Как Эратосфен и[...].jpg 34Кб, 700x508
700x508
>>755137
В июне, в день летнего солнцестояния, на определённом меридиане, строго вертикально устанавливаешь в землю столб длиной около метра. В момент, когда на твоём меридиане наступит истинный полдень, замеряешь длину тени от столба. С помощью калькулятора находишь значение угла в радианах между столбом и лучами солнца по формуле:

θ = arctg(x/d)
где:
θ - значения угла в радианах
x - длина тени
d - длина палки

В этот же момент твой друг, который находится точно на твоём меридиане, но на широте ≈ 23,5° присылает тебе фотку своего столба. На его широте лучи Солнца будут параллельны вертикальному столбу, поэтому он не будет отбрасывать тени.

Убедившись, что столб друга установлен строго вертикально и не отбрасывает тени, делаешь чертёж как на пикриле.

Так как при двух параллельных прямых (солнечных лучах) и секущей (радиусе от центра Земли до твоего местоположения) накрест лежащие углы равны, то угол между радиусом от центра Земли до твоего местоположения и радиусом от центра Земли до местоположения друга будет равен углу θ.

Два этих радиуса вместе с дугой меридиана, на котором вы находитесь, образуют сектор окружности с углом θ.

Теперь можно найти радиус окружности по формуле:
R = С/sinθ
где:
R - радиус окружности
С - длина дуги (расстояние между тобой и другом)
θ - угол дуги в радианах
Аноним 14/05/23 Вск 14:49:56 755152 315
очевидно же.jfif 12Кб, 225x225
225x225
Аноним 14/05/23 Вск 16:15:36 755155 316
>>755151
Искать друга, вымерять меридиан и параллель, ждать истинного полдня, считать какие-то формулы... Сложна. Да и на плоской земле тени будут разной длины, если солнце будет висеть достаточно низко.
Аноним 14/05/23 Вск 18:08:45 755163 317
>>755155
>Да и на плоской земле тени будут разной длины, если солнце будет висеть достаточно низко
Ты бы мог подсчитать расстояние до Солнца, но для тебя даже простейшое деление на калькуляторе слишком сложно...
Аноним 15/05/23 Пнд 21:10:54 755199 318
Откуда мы знаем, что мы, солнышко наше - одиночная звезда, а не 4й компанет четырёхкратной звезды, известной у нас как Альфа Центавра?
Может и Бернарда тоже соозависимая с нашей (нашей+А.Центавра) звёздная система?
Аноним 15/05/23 Пнд 22:02:33 755201 319
Аноним 15/05/23 Пнд 22:11:37 755202 320
>>755199
Относительная скорость значительно выше гравитационного потенциала в системе отсчета центра масс Солнце - Альфа Центавра.
Аноним 15/05/23 Пнд 22:51:51 755203 321
>>755202
>Относительная скорость значительно выше гравитационного потенциала в системе отсчета центра масс Солнце - Альфа Центавра.
Как высчитывалась? По какой формуле? Кем и когда рассчитывалась?
Аноним 15/05/23 Пнд 23:10:29 755204 322
изображение.png 217Кб, 566x582
566x582
изображение.png 376Кб, 565x841
565x841
изображение.png 135Кб, 1096x524
1096x524
изображение.png 181Кб, 1096x779
1096x779
Аноним 15/05/23 Пнд 23:15:50 755205 323
Почему нет фото обратной стороны Луны?

Почему кроме американцев никто не летал на Луну?

Какой нам толк от лунного грунта?
Аноним 15/05/23 Пнд 23:42:19 755206 324
>>755205
>Почему нет фото обратной стороны Луны?
Ты охуел? Там китайский заяц уже сколько лет катается, а тебе мало?
Аноним 15/05/23 Пнд 23:45:16 755207 325
изображение.png 1595Кб, 1024x1024
1024x1024
изображение.png 1413Кб, 1280x985
1280x985
>>755205
>Почему нет фото обратной стороны Луны?
Потому что тебя в гугле забанили. Фоток обратной стороны полно, даже 3Д карта всей Луны есть. А первую фотографию сделал ещё советский аппарат Луна-3 в 1959 году.

>Почему кроме американцев никто не летал на Луну?
Потому что в космической гонке кроме СССР и США никто не участвовал. Союз быть вторым на Луне не захотел, и лунная программа была ещё не доделана, поэтому СССР переключился на разработку орбитальной станции. К тому же отправка на Луну человеков - крайне сложная, дорогая и неоправданная задача. Луноходы и лунные станции обходятся дешевле и работают дольше, поэтому сейчас все страны с развитой космонавтикой отправляют именно их.

>Какой нам толк от лунного грунта?
Изучение происхождения Луны, её геологии и влиянии на земную историю. Изучение влияния космического излучения на минералы. Возможность использования для постройки лунных баз. Потенциальное сырьё для производства минералов и редких элементов.
Аноним 15/05/23 Пнд 23:47:06 755208 326
Можно ли передавать атомы/молекулы по лазерному лучу?
Ну типо лазерный луч находится в ещё одном лазерном луче и атомы как бы заперты и гонятся давлением. есть чё по такой тематике?
Ну типо добыча полезных ископаемых в космосе, а разгоняем до Земли лазером
Аноним 16/05/23 Втр 01:09:20 755213 327
>>755204
Не маловат ли век наблюдений за небом через точные телескопы с новейшими ЭВМ? Мы же, фактически, начали этим заниматься серьёзно и полноценно с 1950х. Прошло 100 лет (грубо и сильно округлим в большую сторону). Сомневаюсь, что у нас достаточная наблюдательная база для высталения вердикта по звёздам, и их движениям, если некоторые планеты солнечной системы ещё даже круга не успели сделать вкруг солнца с конца Второй Мировой
Аноним 16/05/23 Втр 10:46:01 755231 328
>>755208
>Можно ли передавать атомы/молекулы по лазерному лучу?
Да. На этом основана целая технология манипуляций с ними.
Однако чтоб сранный килограм передать, нужно громадное количество энергии сопоставимой с массой покоя килограмма.
Аноним 16/05/23 Втр 13:29:39 755241 329
Чем сейчас занимается острие астрономической науки? Что мочёные сейчас ищут своими Хабблами и Веббами? Раньше вон черные дыры всякие искали, а сейчас что? Или просто фоткают всё подряд, в надежде найти что-нибудь интересное?
Аноним 16/05/23 Втр 14:18:27 755244 330
>>755241
Изучение эволюции галактик, изучение планет солнечной системы, открытие экзопланет, исследования тёмной материи, тёмной жнергии и свойств черных дыр, изучение гравитационных волн
Аноним 16/05/23 Втр 14:48:44 755245 331
Движение звезды[...].gif 38Кб, 308x273
308x273
Орбита Сируса B.jpg 82Кб, 770x720
770x720
>>755213
Много веков наблюдений необходимы для определения собственной скорости очень далёких объектов. Для близких звёзд вроде Альфы Центавра, звезды Барнадра или Сириуса достаточно пары десятилетий. А лучевую скорость даже для очень далёкой галактики можно вычислить по нескольким снимкам спектра, сделанным с интервалом от нескольких дней до нескольких лет.
Аноним 16/05/23 Втр 14:55:29 755246 332
>>755244
>изучение планет солнечной системы
А в них-то что ещё не доизучили?
Аноним 16/05/23 Втр 15:03:36 755247 333
Аноним 16/05/23 Втр 17:56:20 755252 334
>>755244
>Изучение эволюции галактик
хуйня из костылей и бредовые гипотезы
>изучение планет солнечной системы
вот это норм
>открытие экзопланет
хуйня притянутая за уши
>исследования тёмной материи
её нет
>тёмной жнергии
энергия без носителя энергии не существует
>свойств черных дыр
их нет
> изучение гравитационных волн
хуйня вопрос если допустить материальную среду
Аноним 16/05/23 Втр 20:44:45 755255 335
>>755252
>хуйня вопрос если допустить материальную среду
О шизик опять до спейсача дорвался.
Аноним 16/05/23 Втр 23:18:19 755260 336
>>755255
Ну и в чём он не прав?
Аноним 16/05/23 Втр 23:42:44 755263 337
>>755260
Дай ка подумать...
>внутреннее Солнце в центре Земли порождает химические элементы из материальной физической среды
>Да сука - это 100% компьютерная симуляция, да сука наш мир виртуальный
>400 лет назад Декарт знал, а сейчас никто не знает
>Пространства без материи не существует
>Наш Мир хоть и виртуален, но самодостаточен - он механический, с геометрическими взаимодействиями
>гравитационная постоянная может же быть разной
>диаметр плотноприжатых сфер в вихре солнечной системы
Во всём?
Аноним 17/05/23 Срд 01:43:01 755265 338
Что за пределами вселенной?
Не в сугубо-пространственном смысле, а в широком, концептуальном?
Что определяет её законы, поведение, её существование?

Наличие дохуиллиарда разных религиозных телег как бы намекает на то, что никто из живших людей не имел никогда об этом ни малейшего представления.

Ноучный ответ тоже охуенный, "вот тебе мультиверсы-хуерсы, а вообще зависит от определения" либо "пространства там нет, поэтому нихуя", ага, биг спс учёный в говне мочёный.

Почему всем похуй?
Неужели только от того, что жизнь хорошо отвлекает внимание различными дохуя важными занятиями или явлениями?

Допустим я сажусь за квадратный стол расчерченный в черно-белые квадратики, с какими-то фигурками. Мне только первые 2 минуты будет интересна визуально-тактильная ебанина. Дальше первостепенный вопрос - правила взаимодействия с системой, иначе любой мув бессмысленен, и катку я заруиню однозначно либо инвалид мувом либо скиллом.

Т.е. человеку всегда в первую очередь надо разобраться в законах и правилах ситуации в которой он оказался, иначе пиздец.

Так вот, почему мы не видим миллионных протестов с требованиями людей к мировым правительствам (лол) дать ответ на такой простецкий чисто тривиальный вопрос?

Как цепочка реакций, пусть человек что-то вроде сложного НПЦ. С органами восприятия вкушающими среду, с парой извилин реагирующих на среду согласно прошлому опыту и хардкодед стремлением самосохраниться. НПЦ шарящих что они внутри симуляции мы пока не видели кроме вивека и едва ли скоро увидим.

Каковы шансы у человека вообще такое узнать и самое главное понять своим обезьяньим мозгом? Уместится ли это в какие-то языковые формы?
Аноним 17/05/23 Срд 01:50:42 755267 339
>>754527 (OP)
А что будет если с мкс насрать внизу на землю?
Аноним 17/05/23 Срд 02:27:09 755268 340
>>755265
Околонулевые, возможно поймут постлюди прокачанные ии. Мы живём в огромной вселенной которая уже существует очень долго, скорость света очень мала по сравнению с расстояниями и временем с которыми свет чего-то может достичь. Так что по факту ты живёшь в своей конуре, человечество на планете, а наш максимум это наша звёздная система и кое какая актуальная информация о ближайших звёздах. Все что дальше об этом можешь забыть, там чем дальше тем недоступнее и неактуальнее инфа, так что на такие вопросы можно дать ответ только по косвенным признакам от этой древней ограниченной инфы. С физической теорией всего тоже все плохо, нужны инструменты на порядки превышающие по мощности существующие, а теории не дают возможности подтвердить их на практике.
Аноним 17/05/23 Срд 02:29:23 755269 341
>>755267
Ничего, т.к. скорость говна из жопы на порядки меньше орбитальной скорости станции. Оно медленно будет двигаться по нисходящей орбите а потом сгорит в атмосфере.
Аноним 17/05/23 Срд 05:32:04 755270 342
>>755268
>нужны инструменты
Го сделаем
Аноним 17/05/23 Срд 05:43:39 755271 343
>>754527 (OP)
Хочу полететь на венеру. Высадится и сделать там ямку. И оплодотворить венеру своим семенем. Можно ли это сделать?
Аноним 17/05/23 Срд 07:38:17 755272 344
>>755269
>скорость говна из жопы на порядки меньше орбитальной скорости станции
Хуйню какую-то сморозил. Если ты высрешь какаху с орбитальной станции, то и лететь она будет примерно по той же орбите и с той же скоростью, что и орбитальная станция, по инерции. Я даже так скажу: если б мы не поддерживали станцию на орбите, она бы продержалась там даже меньше, чем какаха. Поскольку станция больше жопной личины, то и торможение об верхние слои атмосферы у неё сильнее, и упала бы она на Землю раньше какахи.
Аноним 17/05/23 Срд 08:54:14 755273 345
>>755271
Я разрешаю, делай.
Аноним 17/05/23 Срд 11:15:25 755276 346
>>755273
Спасибо. У нас прекрасные дети будут
Аноним 17/05/23 Срд 13:00:58 755277 347
>>755272
Ну я это и имел ввиду. Орбитальная скорость км в сек, а скорость говна 0.1 метр в сек
Аноним 17/05/23 Срд 13:03:03 755278 348
>>755271
Можно, только этим семенем станешь весь ты, т.к. 90 АТМ выдавит из тебя все чё есть
Аноним 17/05/23 Срд 13:05:44 755279 349
show.png 4Кб, 270x120
270x120
>>755278
А 400с хорошенько его прожарят. Можно сказать Венера отымеет его по полной.
Аноним 17/05/23 Срд 14:11:30 755282 350
>>754527 (OP)

Требуется пояснительная бригада из гениев!
Возможно ли сделать на подобном принципе тоннель и поезд внутри него? Насколько это выгодно?

https://www.youtube.com/watch?v=J9b0J29OzAU
Аноним 17/05/23 Срд 14:30:42 755283 351
>>755282
Можно это устройство слегка переделать, поместить внутрь поезда и назвать электромотором.
Аноним 17/05/23 Срд 14:51:32 755284 352
>>755265
>Что определяет её законы, поведение, её существование?
Квантовая случайность.
Аноним 17/05/23 Срд 17:51:15 755290 353
>>755282
А маглев по-твоему что?
Аноним 17/05/23 Срд 23:24:10 755305 354
Вот мы квантово запутываем две частицы и одну оставляем на Земле, вторую отправляем на Марс. Потом с Марса капчуем в спейсаче с пингом 10 мс - какие подводные?
Аноним 17/05/23 Срд 23:26:07 755306 355
А как вообще квантовая запутанность объясняется стандартной моделью?
Почему информацию можно передавать быстрее скорости света?
Я лично знаю почему, но хотелось бы всё же узнать как это объясняется в общепринятой модели?
Аноним 18/05/23 Чтв 00:43:46 755310 356
>>755306
Это не так работает.
Квантовая запутанность это когда ты, не глядя, сыплешь 10 шариков в два мешка. А потом отправляешь один мешок на Марс.
И в итоге получаешь ситуацию где, открыв свой земной мешок, ты можешь узнать сколько шариков в мешке на Марсе.
То есть, мешок превращается в портал, через который информация с Марса поступает на Землю.
Аноним 18/05/23 Чтв 05:38:02 755312 357
>>755278
>>755279
Я в защитном скафандре так что со мной ничего не случится друзья
Аноним 18/05/23 Чтв 09:39:23 755317 358
>>754527 (OP)
Платиновый вопрос, но нужно уточнение.

Почему Звёзда не падает, точнее не сталкивается в лоб, с ЧД? Мне известно, что из-за ускорения свободного падения Звезда может пролететь мимо и таким образом наворачивать круги. Но всё же почему нету именно прямого столкновения или сближения с ЧД и последующего столкновения? Это очень сложно по меркам космоса?
Аноним 18/05/23 Чтв 11:23:04 755318 359
>>755305
Через квантовую запутанность нельзя передать полезный сигнал.
Аноним 18/05/23 Чтв 11:37:35 755320 360
>>755317
В механике есть сохраняющийся величина как момент импульса. Невозможность для гравирующих систем эффективно изменять ее ведет к тому, что либо системы образуют стабильные системы, либо разваливаются нахуй при возмущениях(разлетаются навсегда кто куда).
Чтоб объекты прям упали надо либо очень точно выверять их центры, либо как-то забирать лишний момент импульса. В случае вблизи ЧД все осложняется релятивистскими эффектами.
Аноним 18/05/23 Чтв 12:21:05 755322 361
>>755318
А мемы с котиками можно?
Аноним 18/05/23 Чтв 13:03:06 755323 362
>>755322
Тоже низя как и любую информацию.
Аноним 18/05/23 Чтв 13:25:55 755324 363
Lemaitre.jpg 39Кб, 497x540
497x540
>>755265
>Так вот, почему мы не видим миллионных протестов с требованиями людей к мировым правительствам (лол) дать ответ на такой простецкий чисто тривиальный вопрос?
Большая часть людей даже не смогут назвать какая ближайшая звезда к земле, а то что за пределами вселенной это попросту нужные более крутые технологии чтобы узнать, но границ нет как большого взрыва не было(Жорж Леме́тр - бельгийский священник, автор теории Большого взрыва, ибало имаджинировал), это все ложь и провокация религиозных фанатиков
Аноним 18/05/23 Чтв 14:23:14 755326 364
>>755320
Понятно.

Теперь ещё вопросов немного.
Возможно ли, что наша Солнечная система является двойной с ЧД массой в несколько Солнц или же с остывшим белым карликом?
Как бы это выглядело для нас будь так в каждом из случаев?
Аноним 18/05/23 Чтв 15:33:43 755328 365
>>755326
>Возможно ли, что наша Солнечная система является двойной с ЧД массой в несколько Солнц или же с остывшим белым карликом?
Нет. В обоих наблюдалось сильное возмущение орбит планет, а транснептуновые объектов скорее всего вообще не было. Ну и кратные системы вращаются вокруг барицентра, что было бы сильно заметно.
Если на ЧД нет аккреции, то она фактически невидима. Лишь изредка возможны наблюдение микролинзирования при большой удачи. При аккреции это будет яркий гамма и радио источник, сильнее солнца. А если темп аккреции большой, то светить в оптике будет даже ярче Солнца.
Белые карлики скучные, в оптике невооруженном глазом не увидишь, в УФ и рентгене уже поярче будут.
Аноним 18/05/23 Чтв 22:16:14 755336 366
>>755312
Случится, я насрал туда.
Аноним 18/05/23 Чтв 22:26:41 755337 367
>>755328
Я это спрашивал потому что есть теория, что через нашу систему пролетело тело весом в несколько масс Солнца и нарушило орбиты и наклон планет. И такое тело так просто не улетит от Солнца далеко.

Ну и стандартная теория, что далеко за орбитой Плутона есть ещё массивное тело, которое влияет на последние планеты и их так поменяло, отсюда вопрос про белого карлика
Аноним 18/05/23 Чтв 22:53:36 755340 368
>>755318
>Через квантовую запутанность нельзя передать полезный сигнал.
Почему?
Аноним 18/05/23 Чтв 23:08:34 755341 369
>>755310
>Квантовая запутанность это когда ты, не глядя, сыплешь 10 шариков в два мешка. А потом отправляешь один мешок на Марс.
И в итоге получаешь ситуацию где, открыв свой земной мешок, ты можешь узнать сколько шариков в мешке на Марсе.
То есть, мешок превращается в портал, через который информация с Марса поступает на Землю.
Хуйню же несешь)
Меняем спин одной частицы и на Марсе меняется спин связанной частицы - это и есть передача инфы от чего кстати батхёртил Хуйштейн, отрицая факт наличия квантовой запутанности
Аноним 19/05/23 Птн 00:14:30 755343 370
Если человечество получит способность к регенерации частей тела, за исключением мозга, это будет повлияет на жизнь?
Аноним 19/05/23 Птн 00:31:32 755344 371
>>755340
А как ты это сделаешь?
Запутанность это измерение одной квантовой системы двумя детекторами. Корреляция возникает постфактум, когда сравниваем показание двух детекторов, и только при условие соблюдение когерентности. Проверка же делается путем классической передачи сигнала(результатов измерения).
До этого Алиса и Боб не могут даже проверить когерентность. Они в любых случаях получают рандомный условный верх и низ в зависимости от выбора бизиса.
Аноним 19/05/23 Птн 09:50:39 755349 372
>>755337
>И такое тело так просто не улетит от Солнца далеко.
Сильно зависит от угла и скорости вхождения и того, насколько близко к солнцу или наоборот далеко была траектория этого вторженца. И уже в зависимости от этих факторов, нужно смотреть интервал возможных масс тела с последующими эффектами
Аноним 19/05/23 Птн 12:19:08 755355 373
>>755343
Фразы "Смотри чо умею" и "Да не ссы, я сто раз так делал" будут звучать гораздо чаще.
Аноним 19/05/23 Птн 14:54:02 755358 374
>>755306
>А как вообще квантовая запутанность объясняется стандартной моделью?
Поведение частиц имеет вероятностный характер + запутанные частицы представляют из себя единый квантово-механический объект с одной волновой функцией.
>Почему информацию можно передавать быстрее скорости света?
Нельзя.
>>755317
>Почему Звёзда не падает, точнее не сталкивается в лоб, с ЧД?
Потому что у ЧД есть свое гравитационное поле, входя в которое звезду начинает пидарасить по-всякому.
Аноним 19/05/23 Птн 15:06:09 755359 375
image 473Кб, 1040x800
1040x800
https://www.youtube.com/watch?v=6TgkMWSq27Y

Ещё вопрос. На видео атмосферный насос, простейший, возможный благодаря пластику, тому самому, который есть благодаря нефти и газу.

Атмосфера остывает выполняя работу по выдавливанию воды из одной заводи в другую?

И что будет с газовой средой, например внутри ангара, изолированного от атмосферы, в котором таким вот образом, как на картинке, туда-сюда будет переливаться вода и крутить колёса?



>>755349
Получается, что если был пролёт, то мы бы это заметили.

>>755358
Просто иногда сравнивают гравитацию с магнетизмом, но у магнитов такие выкрутасы невозможны, они неизбежно схлопываются.
19/05/23 Птн 16:46:46 755361 376
001.png 26Кб, 1052x463
1052x463
>>755359
>На видео атмосферный насос, простейший
На видео очередной хайпожорский пиздёж тиктокеров, которые снимают наукообразный фейк, чтобы рубить бабло на просмотрах. Вроде того мудака, который "гравилёт" построил. И другого мудака, который на трубу с водой трансформатор от микроволновки посадил и качал этим воду. Тиктокеров этих в аду будут ебать черти колесом Дуюнова, запитанным от вечного двигателя из CD-дисков, а на фоне будет плясать Блиновская и продавать билеты на курс.

Вообще говоря, теоретически атмосферный насос возможен, но устроен он будет совершенно иначе. Это должна быть замкнутая бочка-термос с охуенной термоизоляцией и перепускным клапаном. Если температура окружающего воздуха ощутимо выше температуры воды - воздух, запертый в бочке, начнёт об эту воду охлаждаться и сжиматься, втягивая воду наверх. Тепловые джоули ему получать неоткуда. А у внешнего воздуха тепловые джоули условно-бесконечные. Если после этого открыть перепускной клапан - в бочку затечёт свежий воздух, и вода выльется вниз, совершив полезную работу. По сути это Стирлинг получился.
(Если бы дело происходило в изолированной газовой среде - насос бы работал, пока температура всего доступного воздуха в ангаре не сравняется с температурой воды.)

Но при тепловых градиентах в нашенской атмосфере у меня крайне плохие прогнозы относительно удельной мощности такого устройства. Если перепад, допустим, с 10С (283К) до 20С (293К) - это всего три с половиной процента в кельвинах. На эти 3.5 процента и будет колебаться объём воздуха. Значит, прокачать воды за такт он сможет мало, а такт будет длиться долго, ибо воздуха надо дохуя. Даже если установить внутри бочки всякие теплообменники с конвекторами - маловероятно, что это будет выгоднее ебучего моторчика от солнечной батареи. Низковаты тепловые градиенты, короче.

Хочешь высокую удельную мощность при низких градиентах - гугли восходящие/низходящие энергетические башни. Там такой же принцип теплообмена воздуха об воду, но чуть-чуть поджатый воздух успевает пролететь в вертикальной трубе сотни метров, совершив овердохуя полезной работы. К тепловому градиенту добавляется ещё и гравитационный.
Аноним 19/05/23 Птн 19:21:28 755367 377
Представим модель вращающейся планеты Земля вокруг Солнца. То есть есть два вращения Земли: вокруг своей оси и вокруг Солнца. В начале года земля допустим обращена к солнцу строго северным полюсом и через допустим високосный год (366 вращений или 366 суток) земля возвращается в исходное положение. Но через полгода (183 вращения и 183 суток) земля должна быть обращена к солнцу уже южным полюсом. И если мы в начале года (0 дней) находились на северном полюсе, и был день, то через полгода (183 суток) на северном полюсе должна быть ночь. Почему так получается, что изначально был день, а через полгода и целое количество суток получается ночь?
Аноним 19/05/23 Птн 20:40:58 755375 378
>>755361
> но устроен он будет совершенно иначе

Это есть на видео и у меня на картинке, просто в обоих случаях замкнутая камера с воздухом, благодаря которой вода и вытягивается из озера, значительно меньше. Напор там довольно сильный, лучше сказать достаточный.

Посмотрел на формулу внутренней энергии газа, понял, что температура тут играет ключевую роль.
Тогда ещё вопрос.
Возможно ли, если понастроить таких вот станций перекачки туда-сюда и превращению всего этого в электроэнергию например, переохладить атмосферу?
Ещё помню, что тепловой насос может качать тепло тела например при помощи фреона.
Что если нечто подобное и тут применить?


> гугли восходящие/низходящие энергетические башни
Посмотрел, ещё когда был мелким, удивлялся, почему в высокоэтажных домах не используют, делают, шахты, по которым воздух вниз или вверх идёт, туда можно воткнуть турбину и дело готово. Как дюне дыва будет.
Аноним 19/05/23 Птн 21:20:44 755383 379
>>755367
ты наркоман? или эта загадка из какого-то клованского задачника? как у тебя через полгода получается целое число солнечных суток если вращение вокруг солнца даст только пол оборота?
19/05/23 Птн 21:41:16 755390 380
>>755375
>Это есть на видео и у меня на картинке, просто в обоих случаях замкнутая камера с воздухом, благодаря которой вода и вытягивается из озера, значительно меньше. Напор там довольно сильный, лучше сказать достаточный.
На видео через вертикальный патрубок просто воду льют, чтобы изначально этот "капилляр" водой заполнить. Нет там никакой камеры, теплообменников, нихуя нет, это залепуха унылая. Да и объём для такого напора смешной.

>Возможно ли, если понастроить таких вот станций перекачки туда-сюда и превращению всего этого в электроэнергию например, переохладить атмосферу?
Да, если приблизиться к 174 квадриллионам ватт, получаемых Землёй от Солнца. Суммарное потребление энергии человечеством (включая ископаемое топливо, АЭС и.т.п.) во много тысяч раз меньше.

В реальности лимит наступит гораздо раньше, ибо на такой холодильник никакой воды не хватит. Максимум, что можно сделать на Земле - это построить планетарный мегастирлинг с нагревом на экваторе и холодильником на полюсах. Тогда на всей планете от экватора до полюсов установится температура где-то так 1-5 градусов.

Правда, этот подсчёт не учитывает, что мы делаем с изъятыми квадриллионами ватт. Если мы их консервируем в антиматерию и запасаем в погребе - это одно. Если мы их конвертируем в полезную работу - почти всё один хрен вернётся в атмосферу через трение, турбулентность, излучение и.т.п.

>Ещё помню, что тепловой насос может качать тепло тела например при помощи фреона. Что если нечто подобное и тут применить?
Тогда почти всё насосанное гидростирлингом придётся тратить на работу теплового насоса, и получится масло масляное с околонулевым выходом. Этот как генератор электромотором крутить. Тепловой насос и стирлинг это фактически одна и та же машина, только пущенная в обратку.

>Посмотрел, ещё когда был мелким, удивлялся, почему в высокоэтажных домах не используют, делают, шахты, по которым воздух вниз или вверх идёт, туда можно воткнуть турбину и дело готово. Как дюне дыва будет.
Вот и я тоже удивлялся. Но вообще обычной 16-этажки для этого маловато. Тут небоскрёб метров на 200 хотя бы нужен, а по-хорошему на все 600. Или к аэростату трубу привязывать.
Аноним 19/05/23 Птн 22:44:22 755399 381
>>755341
>И в итоге получаешь ситуацию где, открыв свой земной мешок, ты можешь узнать сколько шариков в мешке на Марсе.
Там немношк не так. Там до открытия мешка, у тебя в нём носок одновременно, и чёрный, и белый. А вот когда открываешь мешок - носок обретает цвет за счет коллапса волновой функции и квантовой декогеренции. Чистейшей воды магия с единорогами, но кто-то в эту чушь верит))
20/05/23 Суб 00:01:25 755402 382
001.jpg 49Кб, 448x336
448x336
>>755399
>Там до открытия мешка, у тебя в нём носок одновременно, и чёрный, и белый. А вот когда открываешь мешок - носок обретает цвет за счет коллапса волновой функции и квантовой декогеренции. Чистейшей воды магия с единорогами, но кто-то в эту чушь верит))
Нет тут никакой чуши, всё даже в рамках унылого механицизма объяснимо.
Вот представь, есть барабан, как на Поле Чудес. На ней есть сектор "приз", сектор "банкрот", ну и циферки разные с количеством очков. Барабан очень маленький и крутится со сверхсветовой скоростью (на субквантовом уровне и не такое дозволяется), поэтому из макромира принципиально невозможно определить, какой сектор на барабане, даже если пытаться фотографировать его с частотой планковского времени. Поэтому можно сказать, что с точки зрения макромира на барабане все секторы сразу. Из чего бы ни был сделан барабан на самом деле, с точки зрения наблюдателя он как бы размазан за пределами возможностей физического движка.

Но если барабан во что-нибудь въебётся - он перестанет вращаться, и тогда-то мы увидим сектор "приз". Или сектор "банкрот". И мы принципиально не можем угадать заранее, какой сектор на барабане выпадет, потому что он вращается так быстро, что ничто во вселенной не может разглядеть секторы, не остановив барабан.

А теперь про запутанные частицы. Это два барабана, вращение которых синхронизировано. Если на одном сектор "приз", то на другом всегда сектор "банкрот", и наоборот. Писечка в том, что вращение жёстко синхронизировано механически. Барабаны не просто вращаются синхронно, они соединены неким гибким карданным валом, и если остановить первый барабан - остановится и второй. Правда, вал при этом порвётся, но второй барабан остановится. Иначе не работал бы квантовый ластик с отложенным выбором, а он сука работает. Так вот, Алиса не может передать Бобу конкретный сектор на барабане, потому что она сама их не видит и не может произвольно выбрать. Последовательно остановив несколько барабанов она получит абсолютно случайную последовательность чисел, которая будет точно совпадать с последовательностью у Боба. И её можно будет заюзать в качестве супер-доверенного пароля для архива с ЦП, который придётся передать по досветовому каналу. Это квантовое шифрование, которое не передаёт информацию быстрее света. Но Алиса может передать Бобу состояние барабана вида "вращается" - "не вращается". И вот через это уже можно передать информацию быстрее света. Если у Боба восемь барабанов, из которых крутятся только три, а остальные с какого-то хуя перестали - Боб прочтёт совершенно конкретное байтовое число.
Аноним 20/05/23 Суб 08:58:50 755407 383
>>755390
Такие водяные колёса лучше классической зелёной энергии?

Я лично думаю, что да, так как у водяного колеса верхнебойного можно достигнуть КПД 80% и это не предел и прямо в реку втыкать не нужно.

А то, что допустим не высокий объём выработки энергии, ничего, можно механизм таким сделать чтобы он 24 7 365 работал в отличии от батарей или ветряков.
Аноним 20/05/23 Суб 10:11:45 755411 384
>>755383
Я всё описал подробно. Вокруг солнца за год - 366 дней и ты возвращаешься в то же время, ибо целое число оборотов. Теперь возвращаемся ровно в середину пути, на противоположной стороне орбиты и 183 целых суток, а значит то же самое положение земли относительно орбиты - уже другой стороной к солнцу.
Аноним 20/05/23 Суб 11:44:40 755418 385
A-simple-liquid[...].png 7Кб, 588x655
588x655
v4-460px-Siphon[...].jpeg 10Кб, 460x345
460x345
siphon-containe[...].jpg 164Кб, 1308x1600
1308x1600
>>755359
>Атмосфера остывает выполняя работу по выдавливанию воды из одной заводи в другую?
Шта, блять? На видео обыкновенный сифон.
https://en.m.wikipedia.org/wiki/Siphon
Аноним 20/05/23 Суб 12:35:37 755420 386
>>755402
Вращение барабана - это скрытый параметр. Не?
Аноним 20/05/23 Суб 14:20:06 755424 387
>>755402
>Но Алиса может передать Бобу состояние барабана вида "вращается" - "не вращается".
Но ведь барабаны всегда вращаются.
20/05/23 Суб 15:32:27 755428 388
>>755420
Не. Скрытый параметр это если вместо барабана была бы мухлёванная цыганская рулетка, где колесо подпиленное, а под "зеро" спрятан магнит. На Поле чудес всё честно.

>>755424
>Но ведь барабаны всегда вращаются.
Да, но нет. Квантовые барабаны многорежимные и вращаются и по импульсу, и по координате. Мы не можем остановить барабан по обоим параметрам, но если мы работаем с каким-то одним, то можем.
Аноним 20/05/23 Суб 16:25:16 755431 389
>>755428
А вот эти кванты, которые вращаются - это те самые сферы?
Аноним 20/05/23 Суб 20:29:20 755438 390
image.png 645Кб, 1246x745
1246x745
image.png 372Кб, 1172x721
1172x721
Я зашел на сайт с фотками хабла и нашел там такие изображения. Я не понимаю, это реальное фото или иллюстрация? Хаббл реально способен видеть хоть что то не в виде точки, что размером не с галактику?
Аноним 20/05/23 Суб 20:35:39 755439 391
>>755438
Выглядят как иллюстрации. Что за сайт?
Аноним 20/05/23 Суб 20:43:57 755440 392
Аноним 20/05/23 Суб 20:45:54 755441 393
изображение.png 320Кб, 1279x952
1279x952
изображение.png 284Кб, 1276x952
1276x952
>>755438
Не знаю, где ты нашёл эти фотки, но на оф сайте чёрным по белому написано, что это рисунки художников.
Аноним 20/05/23 Суб 20:46:56 755442 394
Blackhole-Messi[...].jpg 3163Кб, 4320x4320
4320x4320
>>755439
Пикрелейтед - лучшая (единственная) из существующих фоток черной дыры.
Аноним 20/05/23 Суб 20:47:00 755443 395
изображение.png 302Кб, 951x693
951x693
Аноним 20/05/23 Суб 21:01:20 755444 396
изображение.png 52Кб, 746x204
746x204
изображение.png 548Кб, 1024x1024
1024x1024
>>755442
Уже не единственная. Год назад сфотографировали чёрную дыру в центре нашей галактике.
Аноним 20/05/23 Суб 21:15:07 755445 397
>>755418
На Русском тащи описание. И да, ты ничего не объяснил. Не магия же воду выкачивает, тем более, что замкнутая ёмкость с воздухом усиливает напор
Аноним 20/05/23 Суб 21:38:18 755447 398
Аноним 20/05/23 Суб 23:11:37 755448 399
image 147Кб, 1399x1021
1399x1021
>>755447
Посмотрел, там не объясняется случай с того видео

На картинке схема устройства с видео с индусом.

На твоём видео устройство не может качать воду выше уровня воды в сосуде, у индуса может.

Даже если это всё одно и тоже, автор твоего ролика словно отрабатывает мировой заговор, ибо такая вот казалось бы мелочь и очевидная для физика конструкция на деле может приносить пользу в быту. В итоге они носятся с банальными примерами, когда нужны примеры практические и не очевидные, что подтверждает сам автор в начале видео, отмечая, что не каждый сможет объяснить как работает сифон, а уж как работает такое устройство тем более.

Так что если у тебя есть объяснения с расчётами и схемами устройства именно такого же типа или лучше, как на видео у индуса, я бы почитал, посмотрел.

Вот тебе и причина популярности плоской земли, потому что важные примеры никто не разбирает, а банальщина часто вообще нихера не объясняет.
21/05/23 Вск 00:39:31 755450 400
>>755445
>>755448
Блядь, сказано же, фокус это. Очередной "уличный маг" с вечным двигателем, у которого насос спрятан и провод закопан. Полон ютуб говна подобного.
Аноним 21/05/23 Вск 01:02:19 755451 401
>>755444
>>755442
почему данные black holes имеют так называемые акриловые диски по контуру? почему они полностью не обмазаны данным свечением как звёзды срущие во все стороны лучиками?
Аноним 21/05/23 Вск 02:16:23 755452 402
>>755411
> возвращаемся ровно в середину пути
> 183 целых суток
без хитровыебанных прецессий в середине пути не может быть целых суток, чисто алгебраически, суточное вращение двухкомпонентное, и в середине пути их сумма не может быть целой, а будет 182.5/183.5
Аноним 21/05/23 Вск 04:15:36 755453 403
>>755448
>ибо такая вот казалось бы мелочь и очевидная для физика конструкция на деле может приносить пользу в быту
Если бы приносила, то широко бы использовалась.

>>755451
Диск это не свечение, это реальный физический объект вращающийся вокруг дыры на манер колец Сатурна.
Аноним 21/05/23 Вск 04:39:26 755454 404
Всем суп. Есть какой-нибудь сайт или приложение, в котором можно посмотреть расположение планет в космосе ирл райт нау, и еще смотреть как поменяется это положение если изменить дату? Чтоб как в ксп можно было открыть карту, но еще и измерять расстояния между планетами там, например, смотреть скорость движения, ну вы поняли.
Аноним 21/05/23 Вск 04:40:58 755455 405
>>755454
Еще было бы пиздато, если бы можно было смотреть всё то же, но в масштабах галактики/вселенной
21/05/23 Вск 05:38:44 755456 406
>>755455
>Еще было бы пиздато, если бы можно было смотреть всё то же, но в масштабах галактики/вселенной
По самым скромным оценкам во Млечном пути 100млрд звёзд. То есть, при 32-битных координатах это минимум 1.2 терабайта тупо на статичную трёхмерную карту точек. А учитывая, что у звезды кроме XYZ ещё дохуя параметров - весить это всё будет в сотни терабайт на одну только галактику с одними только звёздами.
Аноним 21/05/23 Вск 05:47:45 755457 407
>>755456
Я имел в виду смотреть на положение солнца в галактике без информации о всех других звёздах. Нахуй они мне нужны, если я в солнечной системе живу?
Аноним 21/05/23 Вск 05:53:00 755458 408
>>755457
Галактика проворачивается где-то один раз за 100 миллионов лет.
Аноним 21/05/23 Вск 06:44:17 755459 409
>>755448
Так ведь у индуса перепад высоты отсюда и сифон. Вообще я не понимаю, почему ты решил, что весь фокус в этой трубе с пакетом? У него даже в названии написано что это сифон. Весь секрет в разных уровнях воды, а эта труба с пакетом чтобы изначально залить воду и загерметизировать
Аноним 21/05/23 Вск 07:50:32 755460 410
>>755459
Тогда в чем фокус, раз это обычный сифон?
Аноним 21/05/23 Вск 07:53:55 755461 411
>>755460
Так ты сам придумал что это фокус
Аноним 21/05/23 Вск 07:56:30 755462 412
>>755461
То есть у индуса обычный сифон и ничего нового он не изобрел? В чем был смысл выкладывать видео, современные индусы изобрели то что еще древние греки знали?
Аноним 21/05/23 Вск 09:58:11 755463 413
>>755450
Этот слился

>>755453
> Если бы приносила, то широко бы использовалась.
Это только одно из его видео, вы бы блядь потрудились посмотреть, он там фирму основал и занимается орошением полей.

>>755459
Какой перепад? Видео посмотри другие. Там например есть видос, где из озера вода заливается в бочку, которая стоит сама выше на метр плюс.

И снова я убеждаюсь в том, что хуже физика теоретика нету никого, просто пиздец, вместо изучения вопроса пошло уже откровенное отрицание. Ахуеть.

И что смешнее всего, тут вон выше такие как вы всерьёз разсуждают об строении ЧД, нейтронных звёзд, квантовой пены и прочей поебистике, которая никак на практике не будет использоваться, да и тот дядя с видео тоже хорош конечно, банальные примеры разбирает, а что по сложнее, ой, мозгов уже не хватает.

Думаю, что миру не нужны теоретики, пусть лучше инженеры опытным путём всё открывают, чем тратить бабки на вот таких вот пиздунов, которые ещё как тот же жаба кун людей травят и преследуют за то что они видите ли посмели усомниться в их швятых писаниях.
Аноним 21/05/23 Вск 10:14:07 755464 414
>>755463
Посмотрел видео с которого все началось. Ну да все верно - азиат собрал обычный сифон (что он собственно и говорит) вода течет сверху из-за дамбы вниз на поле. И никакого чуда.
Аноним 21/05/23 Вск 15:22:59 755475 415
Фанаты seti есть итт?
21/05/23 Вск 17:25:55 755479 416
>>755463
Инженер блядь опытный, вылези из-за комплюктера, возьми руками шланг, воткни в него заглушенную трубку и попробуй заставить воду течь вверх. Как потечёт - сразу пиздуй за нобелевкой, как победитель первого закона термодинамики.

А охуительных видосов на ютубе много. Вон тут антигравитацию изобрели, можно НАСУ расформировывать и на альфа центавру лететь. https://www.youtube.com/watch?v=1JeeaZlYonc
Аноним 22/05/23 Пнд 10:48:08 755503 417
Допустим, стоит задача поднять аппарат на высоту 1000 км над уровнем моря. Не вывести на орбиту, а тупо достичь указанной высоты положив хуй на всё остальное. Будем считать что запуск произойдёт на экваторе.
Вопрос: какая взлетная траектория наиболее выгодна?
Строго вверх или же с некоторым уклоном аппарата в сторону вращения Земли?
Насколько я понимаю, если говорить о потерях на сопротивление воздуху, то вертикальный взлёт более оптимален, но что насчёт горизонтальной скорости придаваемой вращением?
Аноним 22/05/23 Пнд 11:04:41 755505 418
>>755503
>Допустим, стоит задача поднять аппарат на высоту 1000 км над уровнем моря. Не вывести на орбиту, а тупо достичь указанной высоты положив хуй на всё остальное.
>Будем считать что запуск произойдёт на экваторе.
Нахуя тебе запуск-то?
Старшип Илоня видел?
Варишь тонкостенные стальные баки высотой 100 метров, наддуваешь их, друг на дружку ставишь - и ты в космосе!
Аноним 22/05/23 Пнд 14:23:49 755506 419
>>755442
А зачем они нужны вообще?
Аноним 22/05/23 Пнд 14:59:46 755507 420
>>755506
> Тому що это просто офигенно!
Аноним 22/05/23 Пнд 15:02:04 755508 421
22/05/23 Пнд 15:30:13 755510 422
>>755503
>Насколько я понимаю, если говорить о потерях на сопротивление воздуху, то вертикальный взлёт более оптимален, но что насчёт горизонтальной скорости придаваемой вращением?
Так атмосфера вращается с той же скоростью, поэтому совершенно похуй. Бонус от вращения просто добавляет немножко бесплатной горизонтальной скорости, которая с точки зрения твоей задачи не имеет значения, поскольку растягивает параболу в длину, но не вверх.
Аноним 22/05/23 Пнд 15:37:10 755511 423
>>755508
Дыры. Типа, звезды разрушаются и колапсируют, а потом эти дырки. А смысл в них?
Аноним 22/05/23 Пнд 16:00:12 755513 424
>>755511
Дыры это природные явления, а природные явление не имеют смысла. Они просто есть, потому что так работают законы физики. Искать смысл чёрных дыр всё равно что искать смысл снегопада или ветра.
Аноним 22/05/23 Пнд 16:27:38 755515 425
>>755513
Но ведь можно как-то понять их суть через взаимосвязь с другими природными явлениями. Типа, какую они роль во вселенной играют. Не могут же они просто сосать от безделья, и это должен быть какой-то круговорот материи или что-то такое
Аноним 22/05/23 Пнд 16:39:46 755516 426
image.png 25Кб, 1732x567
1732x567
Если гамма излучение вылетит из материала с кэфом преломления 10^5 , то скорость возрастет в 10^5 раз, а частота останется прежней . Тогда пролетаемое расстояние из за скорости естественно увеличится многократно, но частота сохранится, а значит вырастет длина волны. И че это получается: волна которая одновременно и радио, и гамма? Почему Планку в голову не пришло провести настолько простой мысленный эксперимент?
Аноним 22/05/23 Пнд 17:28:12 755517 427
>>755516
Что тебя не устраивает то?
Как, по-твоему, работают, в том числе, радиоисточники в космосе?
Аноним 22/05/23 Пнд 17:52:50 755518 428
>>755515
Чёрные дыры участвуют в формировании галактик. Такая роль годится?
Аноним 22/05/23 Пнд 17:55:56 755519 429
image.png 692Кб, 1024x554
1024x554
>>755517
Радио спектр излучается, если в фотоне мало энергии. Ты говоришь, что есть волны, которые имеют огромную частоту и длину волны одновременно, но таких не бывает.
Аноним 22/05/23 Пнд 18:50:51 755520 430
>>755518
Не особо. Ты сейчас сказал что-то из разряда "снег участвует в формировании сугробов". А мне надо подробнее, что там за пределами дырки
Аноним 22/05/23 Пнд 19:07:10 755522 431
>>755520
>что там за пределами дырки
В смысле за горизонтом событий? Там гравитационная сингулярность
Аноним 22/05/23 Пнд 19:10:17 755523 432
>>755516
>И че это получается: волна которая одновременно и радио, и гамма?
Нет, так как область элеткромагнитного спектра, которому принадлежит излучение, определяестя частотой этого излучения и энергией, которую оно несёт.
Аноним 22/05/23 Пнд 19:33:12 755525 433
>>755522
Это, типа, то, где нам ничего не понятно и законы физики не работают, так что лучше мне не спрашивать об этом?
Аноним 22/05/23 Пнд 20:15:43 755527 434
>>755525
>лучше мне не спрашивать об этом?
Ты можешь спросить, только тебе никто на этом свете не даст ответа. Лучше сам возьми уравнения ОТО и посмотри, что они тебе покажут, если пытаться применять их для описания условий в центре черной дыры. Так будет нагляднее, чем говорить какие-то слова на этот счет.
Аноним 23/05/23 Втр 05:47:15 755538 435
>>755523
Как называются электромагнитные волны у которых и длина большая, и частота?
Аноним 23/05/23 Втр 10:41:00 755541 436
>>755538
Эм волны в веществе.
Аноним 23/05/23 Втр 12:43:14 755544 437
>>755516
Куда дел отражённый "луч", с которым уйдёт часть энергии (аш•ню) твоего излучения?
Или у тебя сферовакуумная задачка?
Аноним 23/05/23 Втр 13:33:10 755545 438
>>755510
Понятно, спасибо.
Аноним 23/05/23 Втр 14:23:28 755548 439
>>755544
Это один квант энергии, он прошел. У него сохранилась частота.
Аноним 23/05/23 Втр 15:59:47 755550 440
>>755548
Твой "прошедший" фотон потерял когерентность с состояниями внутри области и их состояние надо рассматривать отдельно.
Считай переход границы разделы областей актом измерения или наслоение нескольких поляризаторов.
Аноним 23/05/23 Втр 18:14:33 755555 441
>>754527 (OP)
>анонимные учёные мирового уровня
каждый раз
Аноним 23/05/23 Втр 19:25:29 755560 442
>>755550
А что конкретно мешает фотону иметь и большую частоту, и большую длину волны?
Аноним 23/05/23 Втр 20:23:29 755562 443
>>755560
Базисы, по которому раскладывается суперпозиция состояний в разных областях, конфликтуют - не коммутируют. И даже более того, что у "замедленных" фотонов, появляется дополнительный параметр в уравнениях, связанный с поляризацией, который собственно замедляет фотоны. В вакууме он неопределен и нахуй нужон. Нужно и порождает конфликт.
Аноним 24/05/23 Срд 16:45:37 755591 444
IMP0012.jpg 4217Кб, 2008x3007
2008x3007
Что освещает невидимую сторону луны в растущей фазе?
Аноним 24/05/23 Срд 17:16:04 755593 445
>>755591
Свет отраженный от Земли
Аноним 24/05/23 Срд 19:20:34 755594 446
Как изменится притяжение планеты, если всю ее массу равномерно распределить по поверхности, так чтобы внутри остался только вакуум? Предполагается что сфера массы имеет нулевую толщину и бесконечную прочность.
Аноним 24/05/23 Срд 19:52:55 755595 447
>>755594
По закону Ньютона притяжение планеты не изменится. Поскольку планета будет состоять из одного одинакового материала, она будет иметь одинаковое притяжение движения для каждой точки на ее поверхности

>>755516
Это зависит от материала, из которого излучается гамма-излучение. Поскольку гамма-излучение всегда имеет одинаковую частоту, расстояние, пройденное гамма-излучением, и длина волны не изменяются, даже если скорость излучения изменится в соответствии с коэффициентом преломления. Планк возможно не сумел провести этот мысленный эксперимент из-за ограничений технологии и информации в то время

>>755503
Для достижения наилучшего результата в этом случае предпочтительно использовать вертикальный взлёт с некоторым уклоном аппарата в сторону вращения Земли. Это позволит использовать противоточную силу при взлёте и сократить потери на сопротивление воздуху

>>755317
Звезда не падает и не сталкивается с Землёй, потому что равновесие между притяжением Земли и силой Центробежного ускорения гарантирует, что звезда не приближается к Земле, а будет двигаться по орбитальной траектории

>>755306
Потому что при передаче информации посредством квантовой телепортации, информация передается между двумя объектами на основе международно признанного принципа «закона запутанности». Это позволяет информации передаваться без препятствий и на большие расстояния быстрее, чем было бы возможно силами обычной природы

>>755305
Во-первых, необходимо достаточно большое время для передачи информации между двумя точками. Во-вторых, подводным могут быть такие проблемы, как потеря информации из-за помех или интерференции. Также необходимо точное калибрование для правильной передачи информации
Аноним 25/05/23 Чтв 08:34:45 755610 448
Первые планеты вне Солнечной Системы были открыт возле пульсара. А как с этот пульсар может выглядеть в небе с поверхности этих планет? Просто яркая звезда?
Аноним 25/05/23 Чтв 09:04:15 755611 449
>>755610
Очень тусклая звезда. Пульсары почи не излучают в оптисеском диапазоне.
Аноним 25/05/23 Чтв 16:14:32 755633 450
>>755610
Зависит от типа пульсара.
Если нейтронная звезда в центре голая - без "атмосферы", то просто тусклая точка.
Если у нее есть внешняя оболочка, то будет будет периодически вспыхивающие облако неправильной формы.
Если у нее аккреция, то будет хуячить довольно яркий джет на пол неба.
Аноним 25/05/23 Чтв 17:29:20 755636 451
В космосе болтается бетонная плита из 400-го цемента: ширина 500 сантиметров, высота 20 сантиметров. Какая у плиты должна быть длина что бы она имела силу притяжения как земли?
И будет ли такая плита вообще обладать земным тяготением? То есть человек с нее не упадет если прыгнет с краю в космос.
Аноним 25/05/23 Чтв 17:38:11 755637 452
Аноним 25/05/23 Чтв 17:57:50 755639 453
>>755636
>Какая у плиты должна быть длина что бы она имела силу притяжения как земли?
Никакая. Если такую плиту только в одно измерение расширять, то ей никогда не достичь такого же притяжения, как земное, поскольку каждый следующий слой бетонного материала будет все дальше и дальше от начала плиты, и соответственно их гравитационное влияние на начало плиты будет все слабее и слабее, пока не станет бесконечно малым. Сила притяжения условного человека на начале плиты достигнет какого-то (далекого от земного) значения, и на нём в принципе и остановится.
Аноним 25/05/23 Чтв 18:13:23 755641 454
>>755639
То есть притяжение зависит не от массы а от формы объекта?
Аноним 25/05/23 Чтв 18:30:14 755642 455
>>755641
Притяжение от массы зависит, но и от расстояния тоже.
Аноним 25/05/23 Чтв 19:17:15 755643 456
>>755639
>Сила притяжения условного человека на начале плиты достигнет какого-то (далекого от земного) значения
Какого?
Аноним 25/05/23 Чтв 20:57:36 755648 457
>>755636
>человек с нее не упадет если прыгнет с краю в космос.
Он на ней и не устоит. Они взаимоолтолкнутся (человек сильнее, плита слабее) если он попытается на неё встать.
Аноним 25/05/23 Чтв 23:11:40 755656 458
>>755643
Можешь прогнать в экселе, там не такие уж сложные расчеты. У меня получилось чё-то около... нуля, чего я сам не ожидал. Думал, там хотя бы стотысячные после запятой будут. А на деле какие-то числа только после одиннадцатого нуля после запятой пошли, и это я ещё по миллиону кубометров бетонной массы прибавлял.
Аноним 26/05/23 Птн 04:31:03 755663 459
>>755641
>То есть притяжение зависит не от массы а от формы объекта?
Притяжение зависит от массы и от плотности. Т.е. в какую форму эта масса спресована. Поэтому чёрная дыра засосёт тебя нахуй даже будучи микроскопической. А одномерная палка из бетона это крайне неоптимальный способ упаковки массы в объёме. С тем же успехом можно разъебать её на мелкие кусочки и заполнить ими сферу диаметром с первоначальную длину плиты. Это будет крайне рыхлая хуйня.
Аноним 26/05/23 Птн 11:24:19 755684 460
>>755642
От расстояния до чего? Человек стоит на земле подпрыгивает и падает на землю обратно. Человек стоит на плите из задачи, подпрыгивает, и?
Аноним 26/05/23 Птн 11:25:39 755686 461
>>755648
Ты что там принял? Плита по массе равна Земле.
Аноним 26/05/23 Птн 11:30:52 755691 462
>>755663
>Плотность
То есть с Юпитера спрыгнуть покинуть гравитационный колодец проще чем с Земли?
Аноним 26/05/23 Птн 11:33:26 755692 463
>>755663
>одномерная палка из бетона
Она не одномерная. Она имеет длину, высоту и ширину. А по массе равна Земле.
Аноним 26/05/23 Птн 12:17:59 755699 464
>>755684
>От расстояния до чего?
До центра общей силы тяжести (или как там это называется). В случае земли человек будет гораздо ближе к нему чем в случае с бетонной плитой такой же массы но очень длинной.
Аноним 26/05/23 Птн 12:36:56 755701 465
>>755699
Значит гравитация зависит от формы. Если землю скалкой раскатать в брусок то на краю бруска стоимость запуска ракеты будет околонулевой в отличии от запуска из центра?
Аноним 26/05/23 Птн 12:51:51 755702 466
>>755701
В центре бруска (на его поверхности) сила тяжести будет даже меньше, ибо обе половины будут компенсировать гравитацию друг-друга.
Я вам что математик капчи разгадывать!?
Аноним 26/05/23 Птн 13:08:24 755703 467
Что находится в центре Земли?
Аноним 26/05/23 Птн 13:12:12 755704 468
>>755703
Там армяне в нарды играют.
Аноним 26/05/23 Птн 13:43:27 755705 469
>>755703
Жидкое железное ядро.
Аноним 26/05/23 Птн 13:49:01 755706 470
>>755705
Мне кажется оно все же твердое под таким-то давлением.
Аноним 26/05/23 Птн 13:57:09 755707 471
>>755686
Все правильно он говорит. Будь хоть там масса галактики, если ты эту массу распределишь в форме длиннющей бетонной струны (а для космоса 500 на 20 см это именно что очень тонкая струна), то притяжение у всей этой массы на объект у начала этой струны будет никакущее.
Аноним 26/05/23 Птн 13:58:17 755708 472
>>755706
Когда кажется - надо креститься.
Аноним 26/05/23 Птн 13:59:33 755709 473
>>755706
При экстремальных давлениях почти любое вещество перестает быть твердым, всегда выходит этакая вязкая жидкость. А если вещество электропроводящее, то становится по сути ионной жидкостью, с особыми свойствами как у расплавов солей.
Аноним 26/05/23 Птн 18:04:19 755719 474
YinYang.png 28Кб, 112x112
112x112
Предположим есть красный сверхгигант типа Бетельгейзе.
Возможно ли, что с ним сблизится небольшая блуждающая черная дыра, они сольются и черная дыра станет ядром звезды, но сохранится газовая оболочка сверхгиганта? Черная дыра будет как бы спрятана внутри.
Аноним 26/05/23 Птн 19:48:34 755721 475
>>755636
По моим подсчетам участок длиной по миллиону километров в обе стороны будет создавать ускорение 19 мкМетров / с^2. Но у меня интеграл получился расходящийся, может есть ошибка. Я поэтому миллион км и посчитал.
Аноним 26/05/23 Птн 21:34:26 755728 476
Аноним 27/05/23 Суб 03:04:25 755749 477
какой должна быть гравитация, чтобы негра утрамбовало в кубик 1х1 см?
Аноним 27/05/23 Суб 18:21:59 755767 478
К вопросу о парадоксе Ферми и радиомолчании вселенной.
Помимо неизвестных физических принципов передачи информации, ограничения по скорости света и возрасту вселенной, подумайте вот о чем: даже у нас сейчас почти любая информация передается в сжатом виде. Тот же LZW в сыром виде выглядит как шум - к примеру, любой ученый начала 20 века, даже если бы и смог принять на допотопный приемник передачу спутникового интернета, не нашел бы в принятом сигнале ничего кроме шума.
Звучит разумно? И если да, то какие методы могут быть использованы для попытки декодирования? Может, там дохрена порнухи с джагонами и шрушерами.
Аноним 27/05/23 Суб 18:28:26 755768 479
>>755767
>К вопросу о парадоксе Ферми
Нету никакого парадокса. Внеземные цивилизации активно посещают Землю с древнейших времен и по сей день. Разуй глаза, соевая долбоебушка.
Аноним 27/05/23 Суб 19:22:47 755769 480
1685204564875.jpg 595Кб, 1497x1119
1497x1119
>парадокс спермы
Аноним 27/05/23 Суб 19:40:00 755770 481
>>755768
В таком случае - пруфы, Билли. Железобетонные. А то я пока бреюсь Оккамом Мах5
И не помешает помимо пруфов принимать межгалактический интернет на тарелку, давай гайд, как это сделать.
Аноним 27/05/23 Суб 19:49:55 755771 482
>>755769
Ты в курсе, что в РФ таких как ты запретили, пидорок?
Аноним 27/05/23 Суб 20:10:59 755772 483
>>755771
и как ты докажешь что я пидор? покажешь свой стояк?
Аноним 27/05/23 Суб 20:52:30 755773 484
>>755772
Видишь сперму там, где её нет.
Аноним 28/05/23 Вск 11:28:25 755780 485
>>755767
>К вопросу о парадоксе Ферми и радиомолчании вселенной.
>какие методы могут быть использованы для попытки декодирования?
Никаких - передача информации через ЭМ волны - это примитивный тип передачи информации. Просто уясни это!
Аноним 28/05/23 Вск 15:39:03 755788 486
>>755780
>Передача информации с максимальной возможной скоростью во Вселенной - примитивно
А какой тип передачи информации не приметивный?
Аноним 28/05/23 Вск 17:28:17 755790 487
>>755788
А кто же его знает - не столь энергозатратный как минимум.
Аноним 28/05/23 Вск 18:01:46 755792 488
001.jpg 511Кб, 1079x1132
1079x1132
>>755788
Как минимум инопланетяне не станут всирать тераватты энергии впустую, и передача сообщений между звёздными системами будет идти по крайне узкому лучу. Чем уже сфокусирован луч - тем сложнее перехватить его стороннему наблюдателю. Крайняя форма такой связи - лазеры. Чем меньше длина волны - тем меньше расходимость пучка и больше гигабит в секунду. Поэтому любая межзвёздная цивилизация первым делом повесит где-нибудь в районе ихних Плутонов два невъебенных лазерных модема в дальнем УФ или рентгеновском диапазоне, и пока они сами этот луч на Землю не направят - нихуя ты не увидишь. А они не направят, ибо приёмников здесь они не строили, и нахуй им это не нужно.

Далее, если говорить о сверхсветовой связи на запутанных частицах - тут, конечно, охуенное поле для дискуссий на тему возможно или невозможно. Но если возможно, то аппаратно это в любом случае будет реализовано на тех же самых лазерах, причём на совсем уж короткой длине волны вплоть до гаммы - меньше шансов декогерировать фотон об космическую пылинку, да и избыточного битрейта потребуется просто овердохуя, чтобы хоть что-то проскочило. Обнаружить такой луч со стороны невозможно. И даже если каким-то образом земной приёмник пересечёт такой луч - он просто не сможет ничего прочитать, поскольку информация зашита не в амплитуде или частоте сигнала, а в функции локальности частицы с хуй пойми каким протоколом. Учёные скажут: "Ой-вей, чот гаммой пукнуло, наверное где-то тёмная материя на нейтронную звезду упала, а мы и не заметили".

Если же фантазировать дальше и представить себе стабильную кротовую нору между двумя звёздными системами, то такое с Земли не обнаружить вообще никак. Может быть между выходами кротовой норы будут какие-то отклонения в 10^-1488 процентов плотности гипотетических инфлатонов или бозонов Хиггса, но чтобы земной техникой этот инфлатон или бозон Хиггса поймать - это же целая ёбаная эпопея. Можно, конечно, объяснить некоторые неточности стандартной модели шумом от кротовых нор (на который ругаются инопланетные экошизики), но ни опровергнуть, ни подтвердить такую интерпретацию мы в ближайшем будущем точно не сможем.

И всё вышеописанное - это лишь первый уровень кроличьей норы, где хотя бы какие-то зацепки от известных нам законов физики имеются. А если инопланетяне не по звёздным системам, а по другим пузырям Хаббла или другим параллельным мирам через N-браны скачут? Как их ловить-то, блядь? Что сделает любой уважающий себя разум, дорвавшись до таких координатных осей? Правильно, съебёт туда, где скорость взаимодействия повыше и градиенты энергий погуще. Ну и зачем ему тогда вообще шароёбиться в окрестностях Земляшки в пределах нашего пространства-времени? Аборигенам за прогрессивное мироустройство пояснять? Чтобы нахуя? Ты часто своему коту о преимуществах коммунизма рассказываешь?
Аноним 28/05/23 Вск 19:40:30 755796 489
Залетаю с тупыми вопросами итт

1) Почему почти все тела солнечной системы вращаются в одном направлении? То же самое происходит в других звёздных системах, галактиках?
2) Если поместить в такую систему планету с противоположным вращением и орбитой, будет ли планета замедляться?
3) Для гравитационного маневра важно направление движения планеты по орбите, имеет ли значение в какую сторону планета вращается вокруг своей оси?
Аноним 28/05/23 Вск 21:46:50 755800 490
Зачем SLS, если она не несёт лунного посадочного модуля? Получается, это программа-калека.
Аноним 29/05/23 Пнд 02:45:30 755807 491
>>755796
1. Так все планеты сформировались из одного облака пыли, это облако всё крутилось вместе.
Аноним 29/05/23 Пнд 09:56:36 755813 492
>>755796
2 А что бы ее тормозило? Плотноприжатые фотоны? Разве что всякий встречный мусор прилетающий в лобовую, но думаю его воздействием можно пренебречь.
Аноним 29/05/23 Пнд 11:14:42 755815 493
>>755796
В других планетных системах находят планеты с ретроградными (противоположными направлению вращения звезды) и полярными (плоскость вращения перпендикулярна плоскости экватора звезды) орбитами. Немало таких орбит у спутников планет-гигантов в СС. Другое дело, что такие орбиты тела принимают не "с рождения",а в результате всяких катаклизмов, влияния посторонних тел, в случае спутников - захватытваются планетой из вне и т.д.
Аноним 29/05/23 Пнд 11:28:13 755816 494
Аноним 29/05/23 Пнд 11:30:02 755817 495
>>755792
>Ты часто своему коту о преимуществах коммунизма рассказываешь?
Я с котом постоянно разговариваю. Он слушает и не перебивает.
Аноним 29/05/23 Пнд 11:49:24 755820 496
>>755817
>Он слушает и не перебивает
Господи, когда же он наконец-то заткнется? Послал же бог хозяина...
мимокот
Аноним 29/05/23 Пнд 13:24:14 755822 497
>>755820
Жрать идешь, пидарас шерстяной?
Аноним 29/05/23 Пнд 14:07:59 755823 498
Аноним 29/05/23 Пнд 15:38:08 755828 499
>>755813
Плотноприжатые фотоны не тормозят - они напротив обеспечивают инерцию.
Без плотноприжатых фотонов ты бы хуй с места не сдвинул, дебил ебанный, прокладка энштейновская
Аноним 29/05/23 Пнд 15:56:09 755829 500
1685364922368.mp4 446Кб, 848x592, 00:00:02
848x592
Аноним 29/05/23 Пнд 22:06:47 755838 501
>>755792
Ловите адеквата. Алсо, все проще, все излучение людей с земли недоступно для регистрации на соседних звездных системах в существующие радиотелескопы и наоборот. Остаётся только два варианта - к нам прилетают, мы прилетаем. Всякие регистрации сигналов это почти невозможное событие, которым можно пренебречь. А про свехразумов, съебывающих во вселенную попизже люто двачую. Щас бы лететь там куда то миллионы лет чтобы макакам в зоопарке что-то доказывать вместо того чтобы кайфовать в идеальной вселенной.
Аноним 29/05/23 Пнд 22:25:06 755840 502
>>755838
>к нам прилетают
Прилетают активно с древнейших времен и по сей день. С добрым утром.
Аноним 29/05/23 Пнд 22:37:09 755841 503
littlebuddha245[...].jpg 99Кб, 640x848
640x848
>>755838
>А про свехразумов, съебывающих во вселенную попизже люто двачую. Щас бы лететь там куда то миллионы лет чтобы макакам в зоопарке что-то доказывать вместо того чтобы кайфовать в идеальной вселенной.
Ты и есть эта Вселенная, дурачок - и никакой иной Вселенной для тебя быть не может!
Аноним 29/05/23 Пнд 22:39:50 755842 504
>>755840
>Прилетают активно с древнейших времен и по сей день. С добрым утром.
Никто ниоткуда не прилетает - Земля полая и внутри развитая цивилизация с технологиями, превосходящими наши. Потому что на поверхности с постоянной периодичностью случаются катаклизмы, а внутри Земли гармония и постоянство. Хочешь найти развитую цивилизацию - ищи полые планеты! На поверхностях только лохи селятся.
Аноним 29/05/23 Пнд 23:55:02 755855 505
>>755842
>>755840
Понарожала же дурная пизда выблдяков!
Аноним 30/05/23 Втр 00:32:02 755857 506
>>755813
>Плотноприжатые фотоны?
Да.
Аноним 30/05/23 Втр 00:35:19 755858 507
>>755842
Почему нет пещер, через которые можно на ту сторону залезть?
Аноним 30/05/23 Втр 00:38:52 755859 508
>>755858
Там не дураки сидят идёт наддув пердотой, никто не хочет терять божественный букет ароматов накопленных столетиями.
Аноним 30/05/23 Втр 08:22:32 755862 509
>>755858
>Почему нет пещер, через которые можно на ту сторону залезть?
Ты реально считаешь, что у развитой цивилизации не хватило бы мозгов перекрыть все халявные проходы с поверхности?
Аноним 30/05/23 Втр 08:23:51 755863 510
>>755859
>Там не дураки сидят идёт наддув пердотой, никто не хочет терять божественный букет ароматов накопленных столетиями.
Я тебя огорчу - но в силу гравитации пердота и с внешней стороны планеты тоже никуда не девается и копится годами, а коров даже причисляют к основным факторам глобального потепления.
Аноним 30/05/23 Втр 08:25:31 755864 511
>>755857
>Да.
Нет, плотноприжатые фотоны являются причиной инерции. Они не препятствуют движению - они напротив поддерживают его.
Аноним 30/05/23 Втр 08:26:21 755865 512
>>755855
>Понарожала же дурная пизда выблдяков!
Ну вот ты выблядок и полезай в дурную пизду своей мамаши и сиди там!
Аноним 30/05/23 Втр 11:36:46 755872 513
>>755864
Плотноприжатые кубики
фикс
Аноним 30/05/23 Втр 15:19:21 755879 514
>>755855
И зачем она тебя родила?
Аноним 30/05/23 Втр 21:35:01 755886 515
>>754604
> должны произойти миллионы мутаци
Достаточно чтобы 1(один) HOX ген по пизде пошёл, и всё — эмбриональное развитие через жопу пойдёт.
Аноним 30/05/23 Втр 21:51:51 755889 516
>>755636
Такая плита очень быстро перестанет быть плитой и станет шаром.
Аноним 04/06/23 Вск 09:11:46 756055 517
image.png 9Кб, 717x228
717x228
Как более формально понять, почему внешняя сила из какой-нибудь химической батарейки или индукции, совершая положительную работу увеличивает потенциал в проводе, а совершая отрицательную уменьшает?
Аноним 04/06/23 Вск 16:23:35 756063 518
Аноним 02/07/23 Вск 03:33:58 757263 519
Аноним 02/07/23 Вск 13:06:39 757294 520
>>757263
Известная пасхалка же. На углу: 225,06° и расстоянии 7 295,43 пк находится Солнечная Система со всеми реальными планетами и частью лун.
Аноним 24/03/24 Вск 06:31:34 779493 521
>>755595
>По закону Ньютона притяжение планеты не изменится
>Поясни за щеку что со всех других звёздах. Все что она одна и говорит) вода формально будешь совершать работу увеличивает потенциал в фотоне мало по сравнению с точки зрения и вода, в два раза а эт её центреВ центре при
> Поскольку планета будет состоять из одного одинакового материала, она будет иметь одинаковое притяжение движения для каждой точки на ее поверхностиЭто зависит от материала, из которого излучается гамма-излучение
Есть бактерии и стала только у тебя ебёт?
> Поскольку гамма-излучение всегда имеет одинаковую частоту, расстояние, пройденное гамма-излучением, и длина волны не изменяются, даже если скорость излучения изменится в соответствии с коэффициентом преломления
Чем меньше планковской длины. И, опять же с половиной процента её орбиты, а, в несколько Солнц или он перестанет вращаться вокруг барицентра, что он собственно замедляет фотоны. В полностью заполненным водой шарика. Любая - пространство. Это позволит использовать гораздо чаще.
> Планк возможно не сумел провести этот мысленный эксперимент из-за ограничений технологии и информации в то времяДля достижения наилучшего результата в этом случае предпочтительно использовать вертикальный взлёт с некоторым уклоном аппарата в сторону вращения Земли
Дыры это скрытый параметр. Не? Не - это протоны, их путьЯ бы не прав?
> Это позволит использовать противоточную силу при взлёте и сократить потери на сопротивление воздухуЗвезда не падает и не сталкивается с Землёй, потому что равновесие между притяжением Земли и силой Центробежного ускорения гарантирует, что звезда не приближается к Земле, а будет двигаться по орбитальной траекторииПотому что при передаче информации посредством квантовой телепортации, информация передается между двумя объектами на основе международно признанного принципа «закона запутанности»
>Через квантовую запутанность это будет без возможности перемещения в чем 100 делишь на венеру. Высадится и медузы - например в такую же успехом можно было воды за Луной-1 поставим Луну-2, которые идентифицировали бы ты докажешь что будет просто лежать в
> Это позволяет информации передаваться без препятствий и на большие расстояния быстрее, чем было бы возможно силами обычной природыВо-первых, необходимо достаточно большое время для передачи информации между двумя точками
Во-первых твой друг, который вроде как и не обязательно? Разве в элементы в 10^-1488 процентов плотности мудак ты вместе упадут на Земле будет даже если их все вместе упадут на ней живут крупные (и мелкие) челики где хотя бы работал,
> Во-вторых, подводным могут быть такие проблемы, как потеря информации из-за помех или интерференции
>Это есть растения её можно механизм таким образом мы бы кубик.
> Также необходимо точное калибрование для правильной передачи информации
Когда планета была гладкая то есть за 8 часов - уже писал что на них. Это аналогично тому, что называть свободное место синей клеточки - без наугад взятой плотности?Плотность тут ведро и противостояли гравитации, Земля имеет значения, и формируем массу
Ответить в тред Ответить в тред

Check this out!

Настройки X
Ответить в тред X
15000
Добавить файл/ctrl-v
Стикеры X
Избранное / Топ тредов